critical reasoning 101 t - amazon s3 · critical reasoning section on the gmat and gre will require...

118
© Maple Leaf International Consulting, Inc., New York New York. For exclusive use by our registered participants only. Not to be copied or reproduced without our consent. Page 1 CRITICAL REASONING 101 wo guys are looking at a glass filled to half its capacity. One guy reaches the conclusion that the “glass is half full” and the other guy reaches the conclusion that the “glass is half empty”. This story, in a nutshell, encapsulates what an ‘interpretation of evidence’ is all about. In this instance, notice that the two so-called different interpretations have the SAME significance and the interpretations will qualify as ‘logical interpretations’. On the other hand, if the person looking at the glass filled to half its capacity were to reach the conclusion that ‘someone forgot to fill the entire glass’ or that ‘someone did not finish drinking all the liquid in the glass’, the person will be ‘guilty’ of making an ILLOGICAL CONCLUSION. An ILLOGICAL CONCLUSION is one that is derived on the basis of the stated EVIDENCE and on the basis of additional information TAKEN FOR GRANTED. The additional information taken for granted is the ASSUMPTION of the argument. In the first case, the persons reaching the conclusion that ‘the glass is half full or half empty’ did not bring in additional information into the argument in order to reach a conclusion. Therefore, the persons reaching the conclusion were ‘logical’ in their interpretation of the evidence. However, in the second case, the person looking at the same evidence ‘assumed’ or ‘took for granted’ that the ‘liquid in the glass could not have evaporated or spilled out, or could not have been drunk by the drinking bird’. (Therefore, the person forgot to fill it to the brim or forgot to drink all of it.) An ARGUMENT is essentially the conclusion of a line of reasoning using an evidentiary basis and additional information either taken for granted or not considered at all. The EVIDENCE provides the CONTEXT for the Conclusion, and is critical to the conclusion. The conclusion of an argument taken alone is without context and may not shed light on the WHY of the conclusion. All information needs to be processed in terms of two questions: WHAT is being said? And Why is it being said? The conclusion answers ‘WHAT’ is being said whereas the evidence answers “WHY” the conclusion is being said. If the conclusion is that the ‘use of marijuana can cause cancer’, the basis or the context for this conclusion is that ‘an ingredient called THC inactivates the herpes virus that can trigger the uncontrolled growth of cancer cells’, we need to process both the WHAT and WHY of the argument. The test is about your ability to process information in context and to understand the question that relates to the argument in order that you pick an answer choice that is relevant to the specific scope of information that is presented to you and to the question posed to you. Do not process information in vague generalities. For example, the above argument about marijuana cannot be processed as ‘something about marijuana being bad’. Process the specifics and deal with the question on that basis. T

Upload: nguyenbao

Post on 08-May-2018

220 views

Category:

Documents


0 download

TRANSCRIPT

Page 1: CRITICAL REASONING 101 T - Amazon S3 · Critical Reasoning Section on the GMAT and GRE will require that you draw conclusions that are logically certain for a set of stated or implied

© Maple Leaf International Consulting, Inc., New York New York. For exclusive use by our registered participants only. Not to be copied or reproduced without our consent.

Page 1

CRITICAL REASONING 101

wo guys are looking at a glass filled to half its capacity. One guy reaches the conclusion that the “glass is half full” and the other guy reaches the conclusion that the “glass is half empty”. This story, in a

nutshell, encapsulates what an ‘interpretation of evidence’ is all about. In this instance, notice that the two so-called different interpretations have the SAME significance and the interpretations will qualify as ‘logical interpretations’. On the other hand, if the person looking at the glass filled to half its capacity were to reach the conclusion that ‘someone forgot to fill the entire glass’ or that ‘someone did not finish drinking all the liquid in the glass’, the person will be ‘guilty’ of making an ILLOGICAL CONCLUSION. An ILLOGICAL CONCLUSION is one that is derived on the basis of the stated EVIDENCE and on the basis of additional information TAKEN FOR GRANTED. The additional information taken for granted is the ASSUMPTION of the argument. In the first case, the persons reaching the conclusion that ‘the glass is half full or half empty’ did not bring in additional information into the argument in order to reach a conclusion. Therefore, the persons reaching the conclusion were ‘logical’ in their interpretation of the evidence. However, in the second case, the person looking at the same evidence ‘assumed’ or ‘took for granted’ that the ‘liquid in the glass could not have evaporated or spilled out, or could not have been drunk by the drinking bird’. (Therefore, the person forgot to fill it to the brim or forgot to drink all of it.) An ARGUMENT is essentially the conclusion of a line of reasoning using an evidentiary basis and additional information either taken for granted or not considered at all. The EVIDENCE provides the CONTEXT for the Conclusion, and is critical to the conclusion. The conclusion of an argument taken alone is without context and may not shed light on the WHY of the conclusion. All information needs to be processed in terms of two questions: WHAT is being said? And Why is it being said? The conclusion answers ‘WHAT’ is being said whereas the evidence answers “WHY” the conclusion is being said. If the conclusion is that the ‘use of marijuana can cause cancer’, the basis or the context for this conclusion is that ‘an ingredient called THC inactivates the herpes virus that can trigger the uncontrolled growth of cancer cells’, we need to process both the WHAT and WHY of the argument. The test is about your ability to process information in context and to understand the question that relates to the argument in order that you pick an answer choice that is relevant to the specific scope of information that is presented to you and to the question posed to you. Do not process information in vague generalities. For example, the above argument about marijuana cannot be processed as ‘something about marijuana being bad’. Process the specifics and deal with the question on that basis.

T

Page 2: CRITICAL REASONING 101 T - Amazon S3 · Critical Reasoning Section on the GMAT and GRE will require that you draw conclusions that are logically certain for a set of stated or implied

© Maple Leaf International Consulting, Inc., New York New York. For exclusive use by our registered participants only. Not to be copied or reproduced without our consent.

Page 2

LOGICAL REASONING 101

Deductive and Inductive Arguments Logical Reasoning involves two types are arguments: Deductive and Inductive. A deductive argument is an argument in if the premises (generally explicitly stated) are true, it would be impossible for the conclusion to be false. In order words, if the evidence is true, then the conclusion is necessarily true or the conclusion MUST BE TRUE. Deductive reasoning will be tested in verbal context (in the LSAT) and in the verbal and quantitative context in the other two graduate school tests – GMAT and GRE. An inductive argument is an argument in which the evidence used is such that the conclusion that is reached on the basis of the premises (explicit and implied) is merely PROBABLE, not CERTAIN. All three tests – LSAT, GRE, and GMAT – will ask you to identify arguments in which the conclusion is merely one of several probable scenarios consistent with the evidence. The Argument essays tested on all three tests are about your ability to identify OTHER equally probable conclusions that are consistent with the evidence. GMAT and GRE will also test your ability to test more than ONE PROBABLE interpretation of the evidence and to see whether the conclusion that can reached on this basis is consistent across different scenarios consistent with the evidence. Because deductive arguments (also referred to as Logical Conclusions on the three tests) are those in which the truth of the conclusion is thought to be completely guaranteed and not just made probable by the truth of the premises (usually explicit or necessarily implied in the explicit information). In such arguments, the truth of the conclusion is "contained within" the truth of the premises; i.e., the conclusion does not go beyond what the truth of the premises implicitly requires. For this reason,

Page 3: CRITICAL REASONING 101 T - Amazon S3 · Critical Reasoning Section on the GMAT and GRE will require that you draw conclusions that are logically certain for a set of stated or implied

© Maple Leaf International Consulting, Inc., New York New York. For exclusive use by our registered participants only. Not to be copied or reproduced without our consent.

Page 3

deductive arguments are usually limited to inferences that follow from literal interpretation of the evidence and from universal definitions relied upon in mathematics and other rules of formal logic. For example, the following are deductive arguments in which the conclusion is necessarily true if the premises used as evidence are true: There are 120 people in the class, taking either English or Spanish language courses, and no person takes both courses. 80 people take English language courses. Therefore, there are 40 students taking Spanish language courses. Each year, the official estimate of the available cod in the Atlantic is made on the basis of an AVERAGE of the two estimates: the official tonnage of cods caught by research vessels and the commercial tonnage of cods caught by other fishing vessels. In the past, the two tonnages agreed were within 1% of each other. However, in the past, the official tonnage went markedly down by the same amount as the commercial tonnage went up. Therefore, the official estimate remained practically unaffected. Notice that because the average is computed by adding up the two estimates and dividing the sum by 2, if one of the values goes up by about the SAME amount as the other went down, then the sum of the two tonnages should remain the same and the average will remain unaffected. The conclusion stated is a valid DEDUCTION and necessarily true. Inductive arguments or ILLOGICAL ARGUMENTS, on the other hand, can appeal to any consideration that might be thought relevant to the probability of the truth of the conclusion. Inductive arguments, therefore, can take very wide ranging forms, including arguments dealing with statistical data, generalizations from past experience, appeals to signs, evidence or authority, and causal relationships.

Page 4: CRITICAL REASONING 101 T - Amazon S3 · Critical Reasoning Section on the GMAT and GRE will require that you draw conclusions that are logically certain for a set of stated or implied

© Maple Leaf International Consulting, Inc., New York New York. For exclusive use by our registered participants only. Not to be copied or reproduced without our consent.

Page 4

To sum up, an INDUCTIVE ARGUMENT is one in which the conclusion that is drawn on the basis of a set of premises explicitly stated or implied in the line of reasoning is PROBABLE but NOT logically certain. The conclusion is but ONE of several PROBABLE interpretations of the evidence. X is an integer. Therefore, X is equal to 2. The above is an example of an INDUCTIVE argument in which the conclusion that X is equal to 2 is PROBABLE but not certain because X could also be 1, 3, 4, 5 or any other positive or negative value or zero. A DEDUCTIVE ARGUMENT is one in which the conclusion is NECESSARILY CERTAIN if the evidence is true. If X is an integer, then it is necessarily true that X is a whole number. If John Scored 160 on the LSAT, then it is necessarily true that John took the LSAT because unless the test is taken, a score may not be obtained. All three tests will require that you logically interpret evidence or any statement and draw ‘deductions’ or necessarily valid conclusions. The Reading Comprehension section on the three tests will test your ability to make logical and literal interpretation of context-specific information relevant to the question. The Logical Reasoning Section on the LSAT and the Critical Reasoning Section on the GMAT and GRE will require that you draw conclusions that are logically certain for a set of stated or implied premises. Remember that questions asking you to identify that which ‘could be’ true are about identifying probable answers consistent with the evidence. IF the evidence is that ‘at most one person recognizes everybody in the room’ it is probable that one person recognizes everybody in the room. It is equally probably that nobody recognizes anybody in the room because ‘at most’ implies equal to or less than. The questions asking you to identify that which MUST BE true for a set of premises asks you to pick an answer that is LOGICALLY CERTAIN for a set of evidence used.

Page 5: CRITICAL REASONING 101 T - Amazon S3 · Critical Reasoning Section on the GMAT and GRE will require that you draw conclusions that are logically certain for a set of stated or implied

© Maple Leaf International Consulting, Inc., New York New York. For exclusive use by our registered participants only. Not to be copied or reproduced without our consent.

Page 5

HOW IS LOGICAL REASONING TESTED ON THE GRADUATE SCHOOL ADMISSION TESTS?

LSAT LOGICAL REASONING SECTION LSAT test will include TWO scored sections of Logical Reasoning section, each section having 25-26 questions to be answered in 35 minutes. WE will discuss the different types of questions that will be tested in this section on the LSAT. It is also probable that the test will include a third LR section that may not be scored, although it is more probable that the not-scored section will be Analytical Reasoning or Reading Comprehension. LSAT will measure your logical reasoning skills throughout the entire test and in the various sections. While the Logical Reasoning Section is specifically geared to measuring your ability to critique inductive or illogical arguments, to engage in deductive reasoning using the evidence presented, and to identify parallel forms and logical explanations for the described paradoxes, the Reading Comprehension and Analytical Reasoning sections will also measure your inductive and deductive reasoning skills. Reading Comprehension is about your ability to deductively interpret context-specific information relevant to the question and to identify the main point of the whole passage. The Analytical Reasoning Section on the LSAT will ask you to draw logically certain conclusions using a set of statements provided to you and then to use the deductions to make further decisions about the presented questions. For example, if the ‘game’ in the Analytical Reasoning Section stated the following: A Storm passes through five towns and drops either rain or both rain and hail. It passes through each town exactly once. The storm passes through Oakville immediately AFTER it passes through Rockville. We would logically interpret the last statement to mean that Oakville cannot be the first town that the storm passes through and Rockville cannot be the last town that the storm passes through. We would then use this logically derived conclusion to check out valid and permissible scenarios for the storm’s path. If the ‘game’ stated that the five music stores in Ghost Town carried three genres of music: Rock, Pop, and Jazz. Each store must carry at least one genre of music. Store B carries more types of music than Store C does and Store D carries more types of music than Store B does, we would ‘logically conclude’ that Store B must carry two types of music and Store D all three types of music. Store C must carry exactly one type of music in order to make the specified statements work and to be consistent with the requirement that each store carry at least one genre of music We would then use this deduction to make further decisions about the scenarios presented.

Page 6: CRITICAL REASONING 101 T - Amazon S3 · Critical Reasoning Section on the GMAT and GRE will require that you draw conclusions that are logically certain for a set of stated or implied

© Maple Leaf International Consulting, Inc., New York New York. For exclusive use by our registered participants only. Not to be copied or reproduced without our consent.

Page 6

Analytical Reasoning Section on the LSAT will test your ability to come up with inductive reasoning and deductive reasoning. Questions asking you to identify that which COULD BE TRUE involve your ability to consider PROBABLE scenarios. Questions asking you to identify that which MUST BE TRUE test your ability to engage in DEDUCTIVE or LOGICALLY CERTAIN reasoning involving a probability of 100%. Therefore, your ability to engage in inductive and deductive reasoning will be put to the test throughout the LSAT. Get ready for the adventure. GMAT CRITICAL REASONING SECTION GMAT does not have a SEPARATE section for Critical Reasoning. The questions in this category will be mingled with other questions and tested throughout the test. Typically, GMAT test-takers will receive 12-14 questions, 10 of which are typically scored. The other 3 or 4 questions will not be scored. Logical and Critical Reasoning will also be tested in Quantitative Assessment. Some of the problem solving questions will ask you to identify an answer that MUST BE TRUE for a set of evidentiary statements provided in the question stem. The Data Sufficiency section will test your ability to deductively interpret evidence, and to identify at least two different probable interpretations of evidentiary information if the evidence allows more than one interpretation. You will then be required to check whether the decision that you will reach will remain consistent across the two or three different, valid interpretations of the evidence. If the question in Data Sufficiency asks: IF 2X = 3Y, is X > Y?, we need to proceed as follows:

1. We need to logically and deductively interpret evidence 2X = 3Y to mean that X and Y MUST HAVE THE SAME SIGN (must lie on the same side of zero; one cannot be positive and the other negative) but we must also engage in inductive reasoning involving PROBABLE SCENARIOS or WHAT IF scenarios: X and Y could be both positive or both negative or both zero. X and Y could be whole numbers or fractions in order to make the evidence work.

2. In one PROBABLE interpretation of the evidence, X = 3 and Y = 2. In this scenario, X > Y. In another, equally probable scenario, X = -3 and Y = -2. In this scenario, X is less than Y. In a third and equally valid scenario, X = 0 and Y = 0. In this scenario, X and Y are equal.

3. You will then conclude that our LOGICAL REASONING using the stem evidence as it applies to the DECISION (whether X is definitely greater than Y or definitely NOT greater than Y) does not permit a unique decision.

Page 7: CRITICAL REASONING 101 T - Amazon S3 · Critical Reasoning Section on the GMAT and GRE will require that you draw conclusions that are logically certain for a set of stated or implied

© Maple Leaf International Consulting, Inc., New York New York. For exclusive use by our registered participants only. Not to be copied or reproduced without our consent.

Page 7

4. We will, therefore, conclude that no logical determination as to the question can be made using the available evidence. We will then go on to examine the next statement or the second item of evidence, and so on.

5. Remember that Logical Reasoning will be tested throughout the GMAT. Even GMAT Sentence Correction will test your logical reasoning skills. In deciding whether ‘problems in counseling are responsible for students failing to graduate’ is better than the statement ‘problems in counseling are responsible for the failure of students to graduate’, you will decide that the latter is the more logically sound expression because the problems cannot be responsible for the students but could be responsible for the ‘failure of students’ to do something.

GRE CRITICAL REASONING SECTION GRE will test logical reasoning in Sentence Completion, which requires test-takers to logically interpret words and phrases used outside the underlined part of the sentence, and Reading Comprehension, which will test your ability to engage in DEDUCTIVE reasoning and to question INDUCTIVE arguments put forth in the passage. Logical and Critical Reasoning will also be tested in GRE Quantitative Assessment. Some of the problem solving questions will ask you to identify an answer that MUST BE TRUE for a set of evidentiary statements provided in the question stem. The QUANTITATIVE COMPATISON section will test your ability to deductively interpret evidence, and to identify at least two different probable interpretations of evidentiary information if the evidence allows more than one interpretation. You will then be required to check whether the decision that you will reach will remain consistent across the two or three different, valid interpretations of the evidence. Consider the GRE Quantitative comparison problem stated as follows: STEM EVIDENCE: (a, 3) is a point in Quadrant II of an X-Y plane. (-4, b) is a point in Quadrant III of the same X-Y plane. DECISION QUESTION: (compare the two values in the two columns and determine whether you can conclude which of the two values MUST BE greater of the two values.

COLUMN A COLUMN B a b

Page 8: CRITICAL REASONING 101 T - Amazon S3 · Critical Reasoning Section on the GMAT and GRE will require that you draw conclusions that are logically certain for a set of stated or implied

© Maple Leaf International Consulting, Inc., New York New York. For exclusive use by our registered participants only. Not to be copied or reproduced without our consent.

Page 8

PROCEDURE FOR LOGICAL REASONING IN THIS CONTEXT OF GRE QUANTITATIVE COMPARISON You are required to DEDUCTIVELY interpret the stem evidence and then to use the deduction to decide which of the two values, a and b, is greater of the two. Our understanding of coordinate geometry would lead us to conclude that a and b must be both negative because a, the x-value of the first point, is negative in QII and b, the second value of the point (-4, b) is negative in QIII. We are, therefore, comparing a negative value with another negative value, In one scenario a could be -1 and b could be -3. In this scenario, a is greater than b. In an equally plausible scenario, a could be -3 and b -1. In this alternative and equally probable scenario, a is less than b. You will, therefore, conclude that our DEDUCTIVE REASONING of the evidence does not permit a LOGICALLY CERTAIN conclusion about which of the two values is greater. We will pick option D as explained in Module II – GRE file and in the Quantitative preprogram work file. LOGICAL INTERPRETATION OF QUANTITATIVE CONCEPTS All formulas and properties of geometric shapes tested on the GMAT and GRE are valid DEDUCTIONS. If ABC is an isosceles triangle, then ABC must have two equal sides and two equal angles. All three angles of ABC must add up to 180. If 5 is the median value of a set of values, then we MUST conclude that there is an equal number of values of either side of 5. If the average and the median of a set of values are the same, then we MUST conclude that the set comprises values that have the same spacing between any two successive values. We will learn in Quantitative Modules that any set of values such that the absolute different of any two successive values is the same as that of any other two successive values, then the median and the mean must be the same. If X,Y, and Z are integers such that XYZ is an odd, positive integer, then we MUST CONCLUDE that each of the three integer values must be odd. With respect to the positioning of these values on a number line, they could all be positive or any one positive and the other two negative. WE cannot be logically certain, under the second scenario, which two of the three values is negative and which is positive. X could be positive and Y and Z are negative, or X and Y are negative and Z is positive, or X and Z are negative and Y is positive. When you try to make decisions, you will be required to use these other, equally probable scenarios and see whether your decision is consistent across different valid scenarios.

Page 9: CRITICAL REASONING 101 T - Amazon S3 · Critical Reasoning Section on the GMAT and GRE will require that you draw conclusions that are logically certain for a set of stated or implied

© Maple Leaf International Consulting, Inc., New York New York. For exclusive use by our registered participants only. Not to be copied or reproduced without our consent.

Page 9

WHY IS LOGICAL REASONING SUCH A BIG DEAL FOR GRADUATE SCHOOL ADMISSION TEST-TAKERS? No matter what career you choose to pursue – be it a nurse, a biochemist, a cytologist, an engineer, a lawyer, a consultant, or a business manager -- you will be required to maintain a healthy dose of skepticism about theories that involve single explanation that excludes alternative ones. Scientists are loath to accept one explanation for the evidence unless they have ruled out other possible explanations for the evidence. Trial lawyers are skeptical about the prosecution’s self-serving ‘single explanation’ for the evidence and usually present alternative explanations for the evidence in the course of creating a ‘reasonable doubt’ about the explanation proffered by the prosecution. Business Consultants make a living by questioning the ‘single explanation’ arguments advanced by their clients. If the client wants to improve ‘employee retention’ by offering more benefits, the consultant will want to explore other explanations for why the company has issues ‘retaining people’. The management style or prospects for growth within the organization could be the issues that the client may have overlooked. In short, logical reasoning involves searching for ‘alternative explanations’ for the evidence or identifying ‘alternative factors’ that could have led to the same outcome. Take a look at the following argument: “In a recent citywide poll, fifteen percent more residents said that they watch television programs about the visual arts than was the case in a poll conducted five years ago. During these past five years, the number of people visiting our city’s art museums has increased by a similar percentage. Since the corporate funding that supports public television, where most of the visual arts programs appear, is now being threatened with severe cuts, we can expect that attendance at our city’s art museums will also start to decrease. Thus some of the city’s funds for supporting the arts should be reallocated to public television.” You should bear in mind that the explanation that is offered in the argument may not always be explicit; it could be implied as in this argument. The argument offers ONE explanation for why the number of people visiting the city’s museums increased by the same 15%: the public television show on visual arts. What could be other explanations for the same increase observed in attendance at museums? The museums advertised or lowered the admission price or had a tie-up with the local schools. It may very well be that the 15% increase in viewership may represent an increase of, say, 1000 viewers whereas the 15% increase in the visitors to the museums may represent, say, 20,000 people, Maybe, the locals watch the television program whereas the tourists from out of town go to museums. Critical thinking is about considering alternative explanations for the same information presented as evidence.

Page 10: CRITICAL REASONING 101 T - Amazon S3 · Critical Reasoning Section on the GMAT and GRE will require that you draw conclusions that are logically certain for a set of stated or implied

© Maple Leaf International Consulting, Inc., New York New York. For exclusive use by our registered participants only. Not to be copied or reproduced without our consent.

Page 10

FACTORS, CRITICAL FACTORS, ENABLING FACTORS Factors are events or things that are causally linked to the evidence as presented in a chain of reasoning. Some of the factors that could be causally linked to admission to a law school are LSAT test scores, undergraduate GPA, motivation as demonstrated through application essays and campus visits, recommendations, and on-time applications. If the evidence is that John was denied admission to Yale Law school and the factor ‘attributed’ to the evidence was ‘his low LSAT Score of 145’, the argument implies that some of the other, equally relevant factors could not have resulted in the denial of admission to John. All the factors identified are PLAUSIBLE in equal measure and the very act of suggesting the existence of other factors that could have led to the same outcome as used in the evidence decreases the PROBABILITY of the original factor identified in the argument. CONCEPT OF PROBABILITY AS IT IS USED IN CRITIQUING THE ARGUMENTS We all know that in probability, the larger the set of values the smaller the probability that any one of those values will be randomly selected. Likewise, the more explanations and factors we can think of for the evidence, the less the probability that the explanation or factor identified in the argument will have played out. The argument seems to imply that, in the absence of other factors addressed in the argument, the probability of the one factor to which the evidence is attributed appears to be 1 or certainty. But if you can identify 4 other factors, deficiency in any of which could have led to the same denial of admission, then the original explanation becomes one out of 5 equally plausible explanations. Therefore, the probability of the original explanation drops from 1 to 1/5. Weakening of an argument is done by decreasing the probability of the original explanation or of the original factor identified in the argument. Weakening of an argument is done by identifying other factors and explanations that increase the set of factors and explanations that could have played out in producing the evidence and, in doing so, by decreasing the probability of the original explanation or of the factor. Strengthening of an argument is done by asserting that the factor or explanation identified in the argument is more probable than other factors. This is usually done by providing additional evidence or information in support of the conclusion for a stated evidence. The Assumption underlying an illogical argument attempts to make the conclusion logically certain (probability of 1) by DENYING the existence or probability of other factors.

Page 11: CRITICAL REASONING 101 T - Amazon S3 · Critical Reasoning Section on the GMAT and GRE will require that you draw conclusions that are logically certain for a set of stated or implied

© Maple Leaf International Consulting, Inc., New York New York. For exclusive use by our registered participants only. Not to be copied or reproduced without our consent.

Page 11

CONTEXT FOR INFORMATION Context is defined as the parts of a discourse that surround a word or passage and can shed light on its meaning. It is also a set of interrelated conditions in which something exists or occurs. The conclusion of an argument has to be viewed in the CONTEXT of the evidence used in the argument. In Reading Comprehension passages, the main idea has to be viewed in the context of the rest of the information presented in the passage. Context is the setting or the environment in which the conclusion exists and must be processed as such. To use an algebra analogy, context for information and the information that is provided exist as TERMS or chained values that cannot be separated. (We know that in algebra X + Y is a binomial expression in which X and Y are chained as terms and cannot be separated). The test will be about your ability to process information in context. This is done by asking two questions:

• What is being said? • Why is it being said?

If we consider a simple argument, “John scored 580 on the GMAT. Therefore, he will not be admitted to Harvard Business School’, the conclusion ‘he will not be admitted to HBS’ must be viewed in the CONTEXT of ‘John’s GMAT Score of 580’. Context for information is referred to as SCOPE of information in the following pages. No matter what the nomenclature, you need to process information in context and not deal with the two chained values (evidence and conclusion) as if they exist in isolation. If the conclusion of an argument is that the use of marijuana will cause cancer, we need to know ‘why the use of marijuana would or could cause cancer’. The evidence will provide the context or the setting for the conclusion and the answer choice for any question has to be consistent with the context or the setting for the conclusion.

Page 12: CRITICAL REASONING 101 T - Amazon S3 · Critical Reasoning Section on the GMAT and GRE will require that you draw conclusions that are logically certain for a set of stated or implied

© Maple Leaf International Consulting, Inc., New York New York. For exclusive use by our registered participants only. Not to be copied or reproduced without our consent.

Page 12

LOGICAL REASONING FUNDAMENTALS You need to be conversant with the ground rules in logical reasoning.

1. IF ALL OF X IS Y, THEN ALL OF Y IS NOT NECESSARILY X BUT AT LEAST SOME OF Y IS X.

An example of this logic is as follows: IF all Americans speak English, all those who speak English are not necessarily Americans. But at least some of those who speak English are Americans. 2. If ALL OF X IS Y, THEN, NOT X COULD BE Y.

An example of this logic is as follows: If all Mexicans are Catholics, then a person who is NOT MEXICAN could be catholic.

3. IF ALL OF X IS Y, THEN NOT Y IS NOT X.

An example of this logic is as follows: If all Business Schools require GMAT Scores for admission, then any school that does NOT REQUIRE GMAT Scores for admission cannot be a Business School.

4. IF Y WILL NOT HAPPEN UNLESS X IS PRESENT, THEN Y MAY OR

MAY NOT HAPPEN IF X IS PRESENT. (If the absence of a factor precludes an outcome, the presence of the factor may not guarantee the outcome)

An example of this logic is as follows: You cannot get into a Graduate School unless you take the standardized admission test. If you have taken the standardized admission test, you may or may not be admitted to a Graduate School if the score is not good enough or if other factors are not in order. If it is true that unless one saves money, one cannot buy a house, then the fact that someone saves money may not guarantee that the person can purchase a house. If the person does not save enough or if other emergencies crop up requiring the using the savings (such as a medical emergency), then the person may still not be able to purchase a house.

Page 13: CRITICAL REASONING 101 T - Amazon S3 · Critical Reasoning Section on the GMAT and GRE will require that you draw conclusions that are logically certain for a set of stated or implied

© Maple Leaf International Consulting, Inc., New York New York. For exclusive use by our registered participants only. Not to be copied or reproduced without our consent.

Page 13

CRITICAL FACTORS AND ENABLING FACTORS Factors, absence of which will preclude an outcome from actualizing, are called critical factors. The absence of an LSAT score will preclude admission to a Law School. But the mere presence of an LSAT score will not necessarily lead to the desired outcome if other factors that come into play once the critical threshold is met are not in order. For example, if one scored 150 on the LSAT, the mere existence of a test score will not guarantee the outcome (admission). If the score is below the school average or if other factors are not in order. Consider the following argument: “Autopsies performed on the bodies of those who had Alzheimer’s disease reveal abnormal deposits of protein in the brain and extensive brain lesions. However, autopsies performed on the bodies of those who died without Alzheimer’s disease also reveal abnormal deposits of protein in the brain but not lesions. Therefore, we can conclude that any person lives long enough will eventually suffer from Alzheimer’s disease.” The context for the conclusion that ‘anyone who lives long enough will eventually suffer from Alzheimer’s disease’ is the ‘observation of abnormal deposits of protein in the brain area of both Alzheimer and non-Alzheimer patients. Will the mere presence of protein in the brain lead to Alzheimer’s disease or do the abnormal deposits of protein require a trigger or ‘enabler’ to result in Alzheimer’s disease? If the question requires that you weaken the argument, you would be looking for a reason why the deposits of protein alone cannot produce Alzheimer’s disease in the absence of an enabling factor or a trigger. If the question required that you strengthen the argument, you would look for a reason why the mere presence of abnormal deposits of protein in the brain area alone is sufficient to produce Alzheimer’s disease. Consider another real-world example. “Unless you buy/obtain an airline ticket, you cannot travel to your destination by plane. If you bought or obtained an airline ticket, you may still get to the destination if the flight were to be canceled, or if you arrived late and missed the flight, or if you were prevented from boarding the aircraft because your name was inadvertently put on the ‘watch list’. Understand that there are several kinds of factors: factors, critical factors, and enabling factors. Some factors require the presence of other enablers to be able to produce the required outcome.

Page 14: CRITICAL REASONING 101 T - Amazon S3 · Critical Reasoning Section on the GMAT and GRE will require that you draw conclusions that are logically certain for a set of stated or implied

© Maple Leaf International Consulting, Inc., New York New York. For exclusive use by our registered participants only. Not to be copied or reproduced without our consent.

Page 14

PAY ATTENTION TO AUXILIARIES USED IN THE STATEMENTS Auxiliary verbs such as COULD, WOULD, WILL, CAN, MAY, MIGHT need to be processed. In fact, when you work on Reading Comprehension passages, you will be required to pay attention to and process these auxiliary verbs. COULD means ‘likely can’ and denotes a PROBABILITY of occurrence of an event. WOULD means ‘likely will’ and connotes a probability of occurrence of an event. “Would” also may suggest an assumption that the author is making in advancing the argument. WILL implies speculation that is based on a good degree of confidence. IF the statement is that if Perry’s faction wins the elections, the nation WILL suffer militarily and if Terry’s faction wins the elections, the nation WILL suffer economically. The use of WILL suggests speculation that is based on reasonable certainty. But the statement also implies that the nation COULD suffer economically if Perry’s faction wins, and the nation could suffer militarily if Terry’s faction wins the elections. CAN implies a potential or confidence. “Replenishment of the gene pool of the cultivated wheat CAN be accomplished through tapping into the resources of the wild wheat’ -- The statement connotes a degree of confidence in potentially enriching the gene pool of cultivated wheat through using the model of wild wheat. MAY implies an uncertainty and a smaller probability. MIGHT is a weaker ‘may’.

QUALIFIERS AND THEIR CONNOTATIONS The QUALIFIER ‘at most’ means ‘equal to or less than’ The QUALIFIER ‘at least’ means ‘equal to or greater than’ SOME means a small proportion of the total population. MOST means a significant proportion of the total population. SOME, SOME, SOME does not cover the entire population. For example, if the statement goes: ‘some are good cooks, some are gourmet cooks, and some are fast-food cooks’, you should know that you have not covered the entire population of cooks. SOME, OTHERS, THE REST cover the whole population. If the statement goes: “some are good cooks, others are gourmet cooks, and still others or the rest are fast-food cooks”, then you have covered 100% of the group of cooks.

Page 15: CRITICAL REASONING 101 T - Amazon S3 · Critical Reasoning Section on the GMAT and GRE will require that you draw conclusions that are logically certain for a set of stated or implied

© Maple Leaf International Consulting, Inc., New York New York. For exclusive use by our registered participants only. Not to be copied or reproduced without our consent.

Page 15

LOOK FOR DELIBERATE VAGUENESS OR AMBIGUITY IN THE ITEMS OF INFORMATION PROVIDED TO YOU

The test is about your ability to look for and identify words and phrases that are interchangeably used but are not synonymous. Deliberate attempts at creating ambiguities may involve dropping of a qualifier or introduction of a qualifier, or use of words that mean different things in different context.

Consider the following argument: “In a recent study, 40 handwriting analysts could not predict the occupations of 100 persons whose handwriting were analyzed Therefore, handwriting analysis is a poor way to predict the personality-types of job applicants”. Notice that the argument implies that personality types and occupations are correlated in some fashion. Consider another example: “Studies have shown that sugars, if consumed immediately after 45 minutes of continuous exercise may actually promote burning of fact and lead to weight loss. Therefore, people who drink aspartame drinks rather than soft-drinks containing sugar may be acting against their self-interest in pursuing dietary goals’. Notice that the argument refers to a study about SUGARS (which are normally fructose found in fruits and juices) and then switches to a reference about SUGAR (notice the singular case noun here). The sugar found in soft-drinks may be processed sugar (sucrose) made from sugar-cane, and may not be interchangeably used with SUGARS that occur naturally in fruits and 100% orange juice. Another ambiguity that you should pay attention to is that the argument implies that dietary goals involve weight loss. A dietary goal may not necessarily involve weight loss or, in some instances, may actually involve weight gain. You need to look for, identify, and process these elements of deliberate vagueness that are found in the information provided to you. Consider the following argument in which the word EXPLOITATION is used in two different connotations but interchangeably referred to: “Your argument that unions protect workers against exploitation misses the point that exploitation of workers is just as natural as exploitation of natural resources”. Notice that ‘exploitation of workers’ does not carry the same connotation as ‘exploitation of resources’. When workers are exploited, they are not paid well or provided benefits or good working conditions. Exploitation of resources could mean complete utilization of resources. Logical and critical reasoning is about paying attention to and processing these subtleties. As well, When you deal with the Argument essay in Analytical Writing Section, look for intended vagueness and interchangeable use of words and phrases, and question their use.

Page 16: CRITICAL REASONING 101 T - Amazon S3 · Critical Reasoning Section on the GMAT and GRE will require that you draw conclusions that are logically certain for a set of stated or implied

© Maple Leaf International Consulting, Inc., New York New York. For exclusive use by our registered participants only. Not to be copied or reproduced without our consent.

Page 16

TYPES OF QUESTIONS TESTED ON THE LSAT, GRE, and GMAT

1. ANALYSIS OF AN ILLOGICAL ARGUMENT • WEAKEN AN ILLOGICAL ARGUMENT • STRENGTHEN AN ILLOGICAL ARGUMENT • FIND THE ASSUMPTION UNDERLYING AN

ILLOGICAL ARGUMENT

2. FIND THE FLAW IN THE ARGUMENT 3. IDENTIFY THE STRUCTURE OF AN ARGUMENT AND IDENTIFY AN

ARGUMENT HAVING PARALLEL STRUCTURE 4. IDENTIFY THE PRINCIPLE THAT WILL SUPPORT THE ARGUMENT

(tested more frequently on the LSAT and less frequently on the GMAT; the principle supporting the argument has the same connotation as stare decisis or case law or precedent in legal jurisprudence. Lawyers make arguments using precedents set by prior rulings on matters similar to the argument they are making).

5. IDENTIFY A LOGICAL CONCLUSION THAT IS NECESSARILY SUPPORTED BY THE EVIDENCE PRESENTED.

6. IDENTIFY a LOGICAL EXPLANATION for the discrepancy or paradox described in the passage.

AN ILLOGICAL Argument is a conclusion that requires at least TWO statements in support of it. The EVIDENCE constitutes one statement and the assumption the other. An illogical argument may require several assumptions in support of the conclusion. We will discuss the anatomy of an illogical argument a bit later in this file. An ILLOGICAL argument can be recognized by noticing the ‘disconnect’ between the evidence and the conclusion. As Johnny Cochran, the attorney for O.J. Simpson, would say, the illogical argument involves a ‘rush to judgment’ and does not clearly lay out the reasoning leading to the conclusion. In other words, an illogical argument IMPLIES at least one statement before the conclusion, and the implied statement is referred to as the argument’s assumption.

In addition to IMPLYING something between the evidence and the conclusion, the illogical argument may also IMPLY something in the conclusion. If the conclusion of an argument is that John will not be admitted to Yale law school, the conclusion implies that John will apply or has applied for admission to Yale law school because one cannot be considered for admission unless one applies. Application is a NECESSARY precondition for admission consideration, and in order for the conclusion about the admission to be valid, the statement must imply that the person named will APPLY for admission. We will get into a more serious discussion of assumptions later on in this file.

Page 17: CRITICAL REASONING 101 T - Amazon S3 · Critical Reasoning Section on the GMAT and GRE will require that you draw conclusions that are logically certain for a set of stated or implied

© Maple Leaf International Consulting, Inc., New York New York. For exclusive use by our registered participants only. Not to be copied or reproduced without our consent.

Page 17

•ANATOMY OF AN INDUCTIVE OR ILLOGICAL ARGUMENT

All ILLOGICAL ARGUMENTS are ‘non sequiturs’ (Latin for ‘it does not follow’), in which a statement (conclusion) does not NECESSARILY follow from the preceding statement or statements. An example of a simple non-sequitur is: “I live in New York City. Therefore, I do not drive a car to work’. Notice that the conclusion ‘Therefore, I do not drive to work’ is NOT necessarily true UNLESS it is true that ‘all those who live in New York City do not drive to work’. Politicians make use of non sequiturs all the time. Consider the argument: “Democrats are bad. Therefore, Republicans are good.” Notice that the conclusion ‘Republicans are good’ is not NECESSARILY TRUE UNLESS it is true that ‘either Republicans or Democrats are good’. The ‘UNLESS STATEMENT’ that the argument relies on is the ASSUMPTION of the argument. The purpose of the UNLESS STATEMENT or the ASSUMPTION is to make the otherwise non sequitur conclusion a ‘necessarily true’. REMEMBER: IN A NON SEQUITUR, THE CONCLUSION IS NOT NECESSARILY TRUE UNLESS THE ASSUMPTION IS VALID, or the IMPLIED ASSUMPTIONS ARE TRUE OR VALID. A non sequitur may rely on more than one ‘UNLESS STATEMENT’ in order to become ‘necessarily true’. Consider the following argument: “Herbal Magic is a cough mixture that is promoted by the manufacturer as an instant cough relief. Yet, this mixture has only 60% of the market share. If it were as effective as claimed, the mixture should command a market-share close to 100%. Because it does not, (therefore) it is not effective.”

EVIDENCE PRESENTED

ADDITIONAL INFORMATION TAKEN

FOR GRANTED OR ASSUMED

CONCLUSION

Page 18: CRITICAL REASONING 101 T - Amazon S3 · Critical Reasoning Section on the GMAT and GRE will require that you draw conclusions that are logically certain for a set of stated or implied

© Maple Leaf International Consulting, Inc., New York New York. For exclusive use by our registered participants only. Not to be copied or reproduced without our consent.

Page 18

Notice that the conclusion that the ‘HERBAL MAGIC is not effective because it does not have a market share of close to 100%’ is a non sequitur and is NOT NECESSARILY TRUE UNLESS each of the following is true:

• Herbal Magic is widely distributed and carried by distributors throughout the country.

• Herbal Magic is widely KNOWN to all cold-sufferers. • Herbal Magic does not have adverse side effects that may preclude

use by some cold-sufferers. • Herbal Magic does not have a competing alternative which is equally

effective. The UNLESS STATEMENTS that must be true in order for the conclusion to be ‘not a non sequitur’ are the ASSUMPTIONS of the argument. Therefore, the non sequitur must be understood as a statement that is NOT NECESSARILY TRUE UNLESS AT LEAST ONE OTHER STATEMENT IMPLIED IS TRUE. The IMPLIED ‘unless statement’ is the ASSUMPTION. The assumption is IMPLIED and never openly stated. COUNTER-SCENARIO is a ‘MAYBE’ scenario that questions the ASSUMPTION. Consider the relationship between the assumption and the counter-scenario as presented in the table below.

ASSUMPTION COUNTER-SCENARIO

1. HERBAL MAGIC IS WIDELY KNOWN TO ALL COLD-SUFFERS. 2. HERBAL MAGIC does not have any adverse side effects that could preclude use by some cold-sufferers. 3. Herbal Magic does NOT have a competing alternative that is equally effective.

1. MAYBE, HERBAL MAGIC IS NOT WIDELY ADVERTISED AND NOT KNOWN TO ALL COLD-SUFFERERS.

1. Maybe, Herbal Magic DOES HAVE side effects that could preclude use by some cold-sufferers.

3. Maybe, Herbal Magic DOES HAVE a competing alternative that is equally effective so that at least some cold-sufferers need not use HERBAL MAGIC.

Page 19: CRITICAL REASONING 101 T - Amazon S3 · Critical Reasoning Section on the GMAT and GRE will require that you draw conclusions that are logically certain for a set of stated or implied

© Maple Leaf International Consulting, Inc., New York New York. For exclusive use by our registered participants only. Not to be copied or reproduced without our consent.

Page 19

THE FLAW IN AN ARGUMENT IS any DEFECT in the structure of the argument that detracts from the persuasiveness of the argument. The FLAW in an argument could be that

• It uses terms interchangeably when the terms are not interchangeable;

• It ignores an alternative and equally plausible explanation; • It draws a conclusion about the larger population on the basis

of a sample, without establishing that the sample is representative;

• It considers just one factor as sufficient for success or failure; FLAW arising from interchangeable use of terms Consider the following argument: The common procedure for determining whether a food additive should be banned from use is to compare its health-related benefits with its potential risks. Yellow Dye No.5, an additive used to color lemon soda, might cause allergic reactions in a few consumers. For most consumers of lemon soda, however, the coloring enhances their enjoyment of the beverage. Therefore, this particular additive should not be banned, because the benefits greatly outweigh its risks. As you can see, the argument interchangeably uses ‘health-related benefits’ and ‘enjoyment of beverage’, and treats the terms synonymously. The Flaw is, therefore, that the argument treats ‘enjoyment of beverage’ as a ‘health-specific benefit’. FLAW arising from ignoring an alternative and equally plausible explanation for the evidence Consider the following argument; “Premiums for automobile accident insurance are often higher for red cars than for cars of other colors. To justify these higher charges, insurance companies claim that, overall, a greater percentage of red cars are involved in accidents than are cars of any other color. If this claim is true, then lives could undoubtedly be saved by banning red cars from the roads altogether.” The argument advocates banning red cars from the roads altogether as a solution for the greater percentage of red cars that are involved in accidents. The alternative explanation for why a greater percent of red cars are involved in accidents is that aggressive drivers prefer to drive red cars that match their type-A or aggressive personalities. The argument ignores this alternative explanation and is, therefore, FLAWED.

Page 20: CRITICAL REASONING 101 T - Amazon S3 · Critical Reasoning Section on the GMAT and GRE will require that you draw conclusions that are logically certain for a set of stated or implied

© Maple Leaf International Consulting, Inc., New York New York. For exclusive use by our registered participants only. Not to be copied or reproduced without our consent.

Page 20

FLAW arising from consideration of a single factor as sufficient reason for the outcome as predicted The arguments having this flaw will attribute a single factor as sufficient cause for the predicted outcome and ignore the possibility that other factors may offset the benefits or the drawbacks of the factor mentioned in the argument. (We will discuss SINGLE FACTOR arguments later on in this file – these arguments assume that the effects of any single factor, good or bad, may not be offset by other factors). Consider the following argument: “If the play were successful, it would be adopted as a movie or revived at the Decade Festival. But it is not successful. We must, regrettably, conclude that neither become a movie nor revived at the Decade Festival” The argument assumes that in order for the play to be made into a movie or revived at the Decade Festival, it must satisfy the ONLY CRITERION of being successful. Maybe, there are other considerations besides success as a play for the play to be made into a movie or revived at the Decade Festival. The argument fails to recognize that the fact that the play does not satisfy one sufficient condition may not preclude the play satisfying a different but sufficient condition for adaptation as a movie or revival at the Decade Festival. Arguments having this flaw are classic ‘single factor’ arguments that ignore the possibility that an outcome may depend on one or another factor. Admission to law school may be predicated on LSAT Scores or, equally, on Legacy as a factor. FLAW in an argument can also arise from a reasoning that is predicated on a single, usually valid, criticism leading to the conclusion that the criticism justifies invalidating the whole thing under consideration (Missing the forest for trees flaw). For example, an argument that LSAT or GMAT test is unfair to those whose native language is not English is a valid criticism but the criticism itself does not invalidate the purpose of the standardized testing for Graduate School Admission. Interestingly, voters in an election are able to intuitively identify this flaw during elections: the fact that George Bush had a DUI arrest on his record did not invalidate his viable candidacy. (A counter-argument we hear is that this mantra was not repeated with Goebbelsian obsession during the 2000 election. In support of this counter-argument, people may cite the swift-boating on Kerry that worked because the technique of argumentum ad nauseam was followed to the letter so that the electors were focused on the single criticism rather than on the bigger picture).

Page 21: CRITICAL REASONING 101 T - Amazon S3 · Critical Reasoning Section on the GMAT and GRE will require that you draw conclusions that are logically certain for a set of stated or implied

© Maple Leaf International Consulting, Inc., New York New York. For exclusive use by our registered participants only. Not to be copied or reproduced without our consent.

Page 21

Consider an argument having the flaw of ‘focusing on the trees and missing out on the forest’ that was tested on the LSAT. “Extensive research shows that television advertisements affect the buying behavior of consumers. Some people conclude from this that violent television imagery sometimes causes violent behavior. But the effectiveness of television advertisements could be a result of those televised images being specifically designed to alter buying habits, whereas television violence is not designed to cause violent behavior in the audience. Therefore, we can safely conclude that violent television imagery does not cause violence.” The flaw in the argument is that it concludes that the entire first claim is invalid because the evidence on which the first conclusion is drawn does not necessarily support the first conclusion. Another flaw is that the argument assumes that if violent images are not MEANT or DESIGNED to cause violence behavior in the audience, they will not. This flaw, referred to as Columbine flaw, ignores the possibility that Some children may find violence portrayed on television or in the movies enticing and may want to ape the behavior portrayed on the screen, even though the producer did not intend for the images to have that effect. FLAW in the argument could also arise from a reasoning that because a sample shows a certain behavior, the larger population will likewise show the same characteristics. The arguments having the flaw will fail to establish that the sample under review is representative. Identifying the flaw in the argument is a matter of extreme critical reasoning. Flaw in the argument could also arise from attacking the character of the person making the argument to discredit his or her argument. Politicians resort to arguments having this flaw. If Richard Clark, a former National Security Advisor, argued that the 500 warheads discovered in Iraq during 2003 date back to pre-1990 Gulf war and could not be weaponized, his critics may counter-argue that Mr. Clark contributed to Kerry’s political campaign and lacks the credibility to make logical arguments. The fact that a person may have a character flaw does not make the person’s argument defective. That Clinton was a ‘womanizer’ did not possibly detract from his ability be a good administrator or ‘leader’ (at least , that is what you would think if you are a Clintonista). Put on your critical reasoning cap and understand the manner in which the argument is structured in order to be able to identify the flaw in the argument. ________________________________________________________________

Page 22: CRITICAL REASONING 101 T - Amazon S3 · Critical Reasoning Section on the GMAT and GRE will require that you draw conclusions that are logically certain for a set of stated or implied

© Maple Leaf International Consulting, Inc., New York New York. For exclusive use by our registered participants only. Not to be copied or reproduced without our consent.

Page 22

LET US RECAP OUR UNDERSTANDING OF THE ANATOMY OF AN ILLOGICAL ARGUMENT. The CONCLUSION of an ILLOGICAL argument is derived on the basis of the presented EVIDENCE and on the basis of the ADDITIONAL INFORMATION ASSUMED or taken for granted. Typically, the Conclusion that we reach is an INTERPRETATION of the evidence. If the interpretation involves using ‘experience’ or ‘expectation’ on top of the presented evidence, the conclusion thus reached will be ILLOGICAL. An ILLOGICAL conclusion is NOT a BAD CONCLUSION. It simply means that the conclusion cannot be accepted at face value, and must be verified. When the corporate managers make decisions or when the prosecution attorneys present a case in a court of law, they always use ‘experience’ or stereotypical ‘expectations’ in order to interpret the evidence, and arrive at ILLOGICAL decisions or conclusions. A CONSULTANT or a DEFENSE ATTORNEY must question the ASSUMPTIONS that were brought to bear on the decision or the conclusion. Experience and expectations lead one to take ‘additional information for granted’ in the context of the evidence. An ILLOGICAL ARGUMENT has the following rationale for its conclusion: “Given the evidence, the conclusion is valid BECAUSE the ADDITIONAL INFORMATION ASSUMED OR TAKEN FOR GRANTED is valid.” The GMAT, LSAT, and GRE test takers will have some experience in dealing with the assumptions in the context of the Argument Essays in Analytical Writing Assessment . The requirement was for the test takers to ‘question’ the unexplained and unexamined assumptions underlying the presented argument. You can look at an illogical argument as one in which the conclusion is one of several possible scenarios that might have played out, and one in which another person looking at the same evidentiary information will go to a different conclusion by bringing his or her own perspectives to bear on the evidence.

Page 23: CRITICAL REASONING 101 T - Amazon S3 · Critical Reasoning Section on the GMAT and GRE will require that you draw conclusions that are logically certain for a set of stated or implied

© Maple Leaf International Consulting, Inc., New York New York. For exclusive use by our registered participants only. Not to be copied or reproduced without our consent.

Page 23

For example, if the evidence is: “Tom received a GMAT score of 400 out of 800 on the GMAT”, and if 5 people are looking at this evidence, each one might make a different conclusion along the following lines:

♦ Tom will not be admitted to a leading Business School. ♦ Tom did not take a prep class. ♦ Tom is not a good test taker. ♦ Tom must have been partying all night the night before. ♦ Tom is not smart.

Notice how each person’s conclusion is driven by his or her unique set of experience and expectations. Neither one of these conclusions is necessarily valid, and each one is “illogical” or ‘non sequitur’ for that reason alone.

The upshot of this analysis is that anytime we use our experience or expectations to reach a conclusion, we are “guilty” of making an illogical conclusion.

LOGICAL CONCLUSION OR INFERENCE A logical conclusion is a necessarily true statement that is supported by a literal interpretation of the specifics of the evidence.

A LOGICAL argument is one in which the Conclusion is essentially a restatement of the argument’s evidentiary piece of information, or a statement of that which is implied but not openly stated. When you make a logical conclusion or an inference, you do not TAKE ANYTHING FOR GRANTED or use information that goes beyond that which is stated in the evidence presented to you.

A LOGICAL interpretation of the evidence presented to you

may also be about ‘interpreting’ the qualifiers used in the evidence or the ‘verbs’ and ‘nouns’ used in the evidence. A logical conclusion can also be made by interpreting information presented in a mutually exclusive context.

EVIDENCE USED

CONCLUSION REACHED BY LITERAL INTERPRETATION

OF THE EVIDENCE USED

Page 24: CRITICAL REASONING 101 T - Amazon S3 · Critical Reasoning Section on the GMAT and GRE will require that you draw conclusions that are logically certain for a set of stated or implied

© Maple Leaf International Consulting, Inc., New York New York. For exclusive use by our registered participants only. Not to be copied or reproduced without our consent.

Page 24

Reading Comprehension Section on the GMAT, LSAT, and the

GRE test your ability to make logical interpretation of the information presented in a specific context, and the procedure you will follow is identical to that you will use in the context of critical reasoning.

Let us examine the evidence about Tom’s GMAT score: “Tom received a GMAT score of 400 out of 800 on the GMAT.” If we are required to make a logical conclusion on the basis of the above evidence, the only logical restatement of the evidentiary information is that “Tom received 50% of the top score on the GMAT.” Notice that we did not pass any value judgment on Tom’s intelligence, or make any way-out predictions about what is in store for Tom when he applies to Harvard Business School. Value judgments and way-out predictions are the stuff ILLOGICAL arguments are made of. A LOGICAL conclusion, on the other hand, stays away from speculation and limits itself to a simple restatement of the evidentiary information or of what is implied but not openly stated. A logical conclusion is a statement is NECESSARILY TRUE if the evidence as presented is true. LOGICAL CONCLUSION can be obtained by identifying that which is necessarily implied between two factual statements. IF all of X is Y and if all of Y is Z, then we can logically conclude that all of X is Z, which is necessarily implied between statements. LOGICAL CONCLUSION can also be obtained by identifying that which is NECESSARILY IMPLIED in a single statement. In fact, the ASSUMPTION that an illogical argument relies on is also an identified implied information but the assumption is not characterized as a logical conclusion because one of the two statements (conclusion) permitting the inference is questionable. In an illogical argument, any inference that is drawn from the evidence constitutes LOGICAL Conclusion whereas any inference drawn from the Conclusion constitutes the Assumption or Questionable inference. In order for you to understand the distinction mentioned above, consider the following illogical argument:

Page 25: CRITICAL REASONING 101 T - Amazon S3 · Critical Reasoning Section on the GMAT and GRE will require that you draw conclusions that are logically certain for a set of stated or implied

© Maple Leaf International Consulting, Inc., New York New York. For exclusive use by our registered participants only. Not to be copied or reproduced without our consent.

Page 25

“No botanist lives long enough to watch a California red wood tree grow to full maturity. Botanists can put together the life story of a California red wood tree by going to a forest of red wood trees and observing trees in various stages of growth. Astronomers should follow the exact same principle in trying to piece together the life-story of globular clusters because no astronomer lives long enough to watch the complete development of a globular cluster.” We know that we are dealing with an illogical argument here because there is a DISCONNECT between the first part, which talks about red wood trees and botanists, and the second part, which talks about globular clusters and astronomers. The argument implies between the two statements that the globular clusters can be observed in various stages of development in order that the EXACTLY SAME PRINCIPLE that applies to the study of red wood trees can be applied to the study of globular clusters. This INFERENCE between two statements is characterized as ASSUMPTION or QUESTIONABLE INFERENCE because the conclusion is a questionable statement and sounds like an opinion or recommendation. (notice the auxiliary ‘should’ in the conclusion). If the second statement read more as a factual statement as follows: “Astronomers use the exact same principle in trying to piece together the life-story of globular clusters, because no astronomer lives long enough to watch the complete development of a globular cluster’, then the statement that is IMPLIED between the two statements, both of which are FACTUAL, becomes the LOGICAL CONCLUSION or UNQUESTIONABLE INFERENCE that is necessarily true. Let us go back to the illogical argument at the top of this page. WE can also draw inference using the evidence alone. The evidence will support the inference or logical conclusion that if a botanist visits a forest of all mature red wood trees, then he or she may not be able to piece together the life story of a California red wood tree. Notice that in order for the life story to be reconstructed, the botanist must be able to observe trees in different stages of growth; if it is not possible to do so because all trees are mature, then we must logically conclude that such a forest may not permit a botanist to come up with a good reconstruction of the life story. Because the above inference was drawn from a FACTUAL statement, it becomes a logical conclusion. In the Reading Comprehension section (that is part of all three tests – GMAT, GRE, and LSAT), you will be required draw logical conclusions by identifying that which is implied between two statements or within a statement itself.

Page 26: CRITICAL REASONING 101 T - Amazon S3 · Critical Reasoning Section on the GMAT and GRE will require that you draw conclusions that are logically certain for a set of stated or implied

© Maple Leaf International Consulting, Inc., New York New York. For exclusive use by our registered participants only. Not to be copied or reproduced without our consent.

Page 26

INFERENCE SUPPORTED BY A LITERAL INTERPRETATION OF THE SPECIFICS OF A STATEMENT. We can make logical conclusions by literally interpreting the specifics of a statement such as the verbs, qualifiers, nouns, pronouns, and information stated in an opposite context. GRE General Test tests your ability to make logical conclusion in two different sections: Reading Comprehension and Sentence Completion. Consider the following GRE sentence completion question: Which of the following logically completes the sentence? “To think that we can eradicate evil altogether is rather simplistic, for evil is a protean creature, forever assuming different __________________”

• Cultures • Forms • Countries • Ideologies • Politics.

The above sentence contains the adjective ‘protean’ that means ‘able to take on different forms and shapes’, versatile, etc. A literal interpretation of the adjective ‘protean’ will give rise to the logical conclusion that evil takes on different forms and shapes. We must choose the option 2 here. It is unlikely that GMAT and LSAT test-takers will have to deal with such highfalutin vocabulary that is normally tested on the GRE but the same principle and practice are relevant to GMAT and LSAT tests. Logical interpretation of adjectives does not have to be a fancy stuff. We can interpret simple qualifiers in order to be able to come up with a logical interpretation. Consider the following statement reproduced from a RC passage: “What makes Shaw’s very questionable diagnosis appealing is that it answers problems raised in three other interpretations of American Revolution.” Our logical interpretation of the qualifiers ‘very questionable’ and ‘appealing’ will lead us to logically conclude that ‘an analysis of doubtful plausibility can be attractive’. Notice that we literally interpreted ‘diagnosis’ as ‘analysis’, questionable as ‘doubtful’ and ‘appealing’ as attractive. This is what logical interpretation is all about.

Page 27: CRITICAL REASONING 101 T - Amazon S3 · Critical Reasoning Section on the GMAT and GRE will require that you draw conclusions that are logically certain for a set of stated or implied

© Maple Leaf International Consulting, Inc., New York New York. For exclusive use by our registered participants only. Not to be copied or reproduced without our consent.

Page 27

Let us consider another example. Make a LOGICAL INTERPRETATION OF THE FOLLOWING INFORMATION: “Bush administration’s tax cut plan will give the NEEDED STIMULUS to the American economy.” Our interpretation of the above statement involves literally interpreting the qualifier ‘needed’ and will read: “The American economy is in NEED OF incentive to grow.” Any other interpretation such as ‘Bush tax cut plan favors the super rich on the backs of the struggling middle class’ or ‘Tax cuts always stimulate economic growth’ is illogical because these interpretations go way beyond the limited scope of information presented to you. Take a look at another example of interpreting qualifiers in order to reach a logical conclusion: “Anything that exploits the musical characteristics of a language is a poem. A novel, however, does not USUALLY exploit the musical characteristics of a language.” Our logical interpretation of the evidence must involve demonstrating an understanding of the qualifier ‘usually’. Our inference or logical conclusion is that ‘a novel SOMETIMES exploits the musical characteristics of a language, and when it does it must be characterized as a poem’ (as the evidence would have us believe). Let us take a look at an example of how we can interpret verbs and nouns used in the ‘evidence’ presented to us. Consider the following example that interprets verb: “The Earth association sent out 3,000 mailers to residents of town X and claimed that it CONVERTED 3,000 individuals to its cause.” Our logical interpretation must be about literally interpreting the verb ‘convert’, which implies that that the 3,000 individuals who received the mailers did not believe in the cause of the earth association. (you cannot convert the converted. The verb ‘convert’ suggests that the mailers were not intended to preach to the choir.) Let us take a look at how we can make a logical conclusion by literally interpreting ‘nouns’ used in the evidence presented:

The most commonly made mistake when test takers are required to reach an inference or a logical conclusion on the basis of the evidence presented is that the test takers pick a choice that is not derived solely on the basis of evidence. Instead, they end up picking a choice that either explains the evidence or reaches an illogical conclusion on the basis of an assumed information.

Page 28: CRITICAL REASONING 101 T - Amazon S3 · Critical Reasoning Section on the GMAT and GRE will require that you draw conclusions that are logically certain for a set of stated or implied

© Maple Leaf International Consulting, Inc., New York New York. For exclusive use by our registered participants only. Not to be copied or reproduced without our consent.

Page 28

“The New York City apartment in which Miles Davis and his group gathered became a SEED BED for the West Coast cool jazz style.” Our literal interpretation of ‘seed bed’ is that the West Coast cool Jazz style originated or ‘took root’ in the New York City apartment in which Miles Davis and his group gathered. Let us consider an example of how we can ‘literally interpret’ information presented in a mutually exclusive context. “During the industrial revolution, the mill owners chose women to do jobs that required repetitive chores because the mill owners thought that women were more patient (than who?) with such jobs.” Our logical interpretation of the comparison conjunction is that ‘women were more patient THAN MEN with jobs requiring repetitive chores”. Consider another example of how we can interpret information presented in an opposite context: “Prior to 1995, the Harvard Business School did not require GMAT scores for admission purposes.” Our logical interpretation of the above statement is that AFTER 1995, the HBS did require the GMAT Scores for admission purposes. Again, you will be required to use the same skill in literally interpreting the evidence presented to you when you are presented with questions in Reading Comprehension. A more detailed discussion showing more examples of how you can make logical conclusions in the context of graduate school tests is available later on in this file. For now, understand the procedure for making logical conclusion on the basis of information presented to you. In sum, a LOGICAL CONCLUSION is about Literally interpreting the evidence presented to you; Literally interpreting qualifiers used in the evidence; Literally interpreting the verbs and nouns used in the evidence; Literally interpreting information presented in a mutually exclusive

context. You will be tested on this skill in two sections on the GMAT and LSAT: The critical reasoning and Reading Comprehension. The GRE will test this skill in the context of its Reading Comprehension section and Sentence Completion section. (GRE test-takers, know your vocabulary.)

Page 29: CRITICAL REASONING 101 T - Amazon S3 · Critical Reasoning Section on the GMAT and GRE will require that you draw conclusions that are logically certain for a set of stated or implied

© Maple Leaf International Consulting, Inc., New York New York. For exclusive use by our registered participants only. Not to be copied or reproduced without our consent.

Page 29

You should expect about 6 questions out of 13 questions on the GMAT Critical Reasoning Section, asking you to make logical conclusions on the basis of given evidence. Another 6 or 7 will ask you to deal with illogical conclusions in terms of WEAKENING or STRENGTHENING the conclusion in the arguments, or of IDENTIFYING THE ASSUMPTION that logically connects the evidence to the illogical conclusion stated. On the LSAT, there are two Logical Reasoning sections of 26 questions each, and you should expect about 10 to15 questions out of the 50-52 questions asking you to identify a logical conclusion.

GMAT and LSAT will also require that you demonstrate ‘comprehension’ of information presented to you in Critical Reasoning. Such ‘comprehension’ questions will ask you to recap the essence of the argument, or to find a parallel form, or identify the relationship between the two different sections of the argument. NO matter what you are expected to deal with during the test, you should bear in mind that you will be required to wear your blinkers and limit your close examination of answer choices to only those that contain the SUBJECT and the SCOPE of each argument.

COMMON MISTAKE MADE BY TEST TAKERS DEALING WITH AN ILLOGICAL ARGUMENT or with a LOGICAL CONCLUSION OR INFERENCE. When test takers analyze an ILLOGICAL ARGUMENT,

they INTERPRET the conclusion of the argument, and reach their OWN CONCLUSION first, and then begin to attack their own conclusion instead of the original conclusion of the argument. For example, if the argument’s conclusion is that ‘the continuing war effort in Iraq will hurt the U.S. economic recovery’, test takers ‘interpret’ this conclusion to mean that ‘the war in Iraq has bad consequences’ or that ‘the war effort in Iraq is unjustified’. These ‘interpretations of the argument’s conclusions’ are fraught with disastrous consequences for test takers. TAKE THE ARGUMENT’S CONCLUSION LITERALLY. Remember: You are required to ATTACK the argument’s ORIGINAL CONCLUSION in its literal form, and not your ‘own conclusion’ reached on the basis of an ‘interpretation of the argument’s original conclusion’.

Page 30: CRITICAL REASONING 101 T - Amazon S3 · Critical Reasoning Section on the GMAT and GRE will require that you draw conclusions that are logically certain for a set of stated or implied

© Maple Leaf International Consulting, Inc., New York New York. For exclusive use by our registered participants only. Not to be copied or reproduced without our consent.

Page 30

Another example of a ‘fatal interpretation of the conclusion’ is as follows: The original conclusion reads: “Small businesses will take business

away from large businesses’. You ‘interpret’ this original conclusion to mean ‘small businesses are more profitable than are the large businesses’ or ‘large businesses are less competitive than are the small businesses’.

The original conclusion reads: “The LOCATION OF THE VASE shows that there was trade between Greece and Poland”. You ‘interpret’ this conclusion to mean ‘the VASE shows that there was trade between Greece and Poland’ or ‘Greece and Poland had Free Trade agreements’ or ‘Greece and Poland have historical trading relations’. You forgot to pay attention to the SUBJECT of the conclusion, ‘the LOCATION’, and failed to take the argument’s conclusion literally.

The original conclusion reads: “The Free Trade agreements between developed and developing nations will lead to a lowering of living standards in the developed nations”. You ‘interpret’ this conclusion to mean ‘the developed nations should not trade with developing nations’ or ‘free trade agreements will lead to an improvement of living standards in the developing nations’.

If the conclusion of the argument is that ‘the use of marijuana can cause cancer’, do not interpret this statement to mean that the ‘use of marijuana is a bad thing’. Deal with the specifics of information provided to you.

The problem with these ‘broad’ interpretations of the conclusions is that you will end up picking an answer choice that might attack your interpretation but not the specifics of the original argument. You should be on guard and stay away from loosely ‘interpreting’ the argument’s conclusion. TAKE IT LITERALLY. Also, pay attention to the qualifiers, verbs, nouns used in the evidence and in the conclusion. If the conclusion reads ‘X will REPLACE Y”, and if you are required to look for a weakening answer, you should look for a reason why ‘X will NOT Replace Y”. It is not enough for you to identify a choice stating that ‘Y will continue to be present’. Similarly, if the conclusion is that ‘the LOCATION OF THE VASE shows there was trade’, and if you are required to strengthen the conclusion, you should look for a reason why the ‘location’ of the vase reveals that there was trade. DO NOT BE DISTRACTED. The tests measure your ability to remain focused on the information presented to you and to resist distraction. Arguments about babies, insurance, cars, drugs, politics, etc. allow test-takers to wander off into areas that are not relevant to the scope of information presented to them. Stay FOCUSED! Do not Drift.

Page 31: CRITICAL REASONING 101 T - Amazon S3 · Critical Reasoning Section on the GMAT and GRE will require that you draw conclusions that are logically certain for a set of stated or implied

© Maple Leaf International Consulting, Inc., New York New York. For exclusive use by our registered participants only. Not to be copied or reproduced without our consent.

Page 31

very argument will be about a “central character”, ‘hero’ if you will. We will call this “central character” the SUBJECT of the argument. The SUBJECT of the argument is the one engaged in the action

described, or about which or whom the argument is made. In the Argument, “Tom received a GMAT score of 400 out of 800 on the GMAT. Therefore, he will not be admitted to Harvard Business School”, Tom’s GMAT Score is the SUBJECT of the argument, and the argument is about the ‘effect’ of Tom’s GMAT score on the ‘admission to HBS’. What the argument is about defines the SCOPE of the argument. Bear in mind that arguments are not about a broad generic information; they deal with specifics and it is your job to identify and to pay attention to the SPECIFICS of the argument. We will also see that critical reasoning requires you to be a good QUALIFIER POLICE. Qualifiers bring in specificity to an argument and any inappropriate use of qualifiers in an answer choice must be a flag item. Politicians use ‘qualifiers’ to manipulate and to manage expectations and voter perceptions. You must be alert to the possibility that qualifiers originally used in the argument may be needlessly ‘emphasized’ using adverbs in the answer choices, or may be used as nouns instead of as qualifiers in the answer choice. The classic example is the statement made by Prime Minister Tony Blair in the British Parliament that Saddam possessed weapons (of mass destruction) that could be set off in 45 minutes. Later on, the same politician started using ‘weapons’ as an adjective, not as the noun it was in the original statement: “Saddam had weapons (of mass destruction) program”. As a critical thinking person, you should pay attention to the fact that in one statement ‘weapons of mass destruction’ was used as a noun phrase; in the other, it was used as an adjective. Because an adjective and a noun serve different purposes in a sentence, you should consider that the two statements are not synonymous. You should likewise be skeptical about adverbial phrases bringing in a needless emphasis or a ‘wiggle room’. Consider the following statement that David Kay, the weapons inspector, made after he returned from Iraq: “I do not expect that Iraq has significant stockpiles of weapons of mass destruction”. Notice that David Kay created a wiggle room by using ‘weapons’ as a qualifier phrase and made ‘significant stockpiles’ as the subject. Also, notice that the use of ‘significant’ created a wiggle room for David Kay: If anyone ever found any stockpile of weapons in Iraq, Kay could argue that it was not ‘significant’.

E

Page 32: CRITICAL REASONING 101 T - Amazon S3 · Critical Reasoning Section on the GMAT and GRE will require that you draw conclusions that are logically certain for a set of stated or implied

© Maple Leaf International Consulting, Inc., New York New York. For exclusive use by our registered participants only. Not to be copied or reproduced without our consent.

Page 32

Similarly, adverbs bring in added emphasis to a verb or an adjective and such emphasis may not be warranted. Consider the statement: “I am totally confused”. The adverb ‘totally’ adds an emphasis to the adjective ‘confused’. If you are confused, you are confused. There is no point in emphasizing the ‘degree of confusion’. Similarly, “I am slightly weak in math” is another illogical statement. There cannot be degrees of weakness. We will also have to use this logic in the context of sentence correction, and stay away from answer choices that bring in unwarranted emphasis through use of adverbial phrases. In Critical Reasoning and in Reading Comprehension, adverbs always raise a red flag and must be checked out very critically. We will discuss in the following pages why adverbs must be viewed as ‘troubling’ by you when you work on the test. We started this discussion by telling you that you must identify the subject in the evidence. You should also pay attention to the ‘subject’ of the sentence making up the conclusion. We have previously discussed why it is important to take the specifics of the conclusion literally, and not broadly interpret the conclusion. ALSO, PAY ATTENTION TO THE QUALIFIERS SUCH AS ADJECTIVES and to the VERBS used in the conclusion. Your answer choice must be relevant to the Subject and to the Verb of the sentence making up the conclusion. If the conclusion reads “the small businesses will TAKE BUSINESS AWAY from large businesses”, your choice must be relevant to the ‘taking business away’ aspect of the conclusion and must be relevant to the small businesses ‘taking businesses away’ from the large businesses. Do not misread the conclusion as ‘small businesses will be more profitable than are the large businesses. IF the conclusion is that ‘if tolls are increased on the highways, the revenues will increase’, and if you are required to attack this conclusion, you should look for a reason why ‘increasing tolls on the highways will not result in increased revenues’. It is not enough for you to identify a reason why ‘it is unfair to raise the tolls on the highways and to penalize motorists’. LEARN TO DEAL WITH THE SPECIFICS OF AN ARGUMENT, AND TO ATTACK THE ORIGINAL CONCLUSION, NOT YOUR INTERPRETATION OF THE ORIGINAL CONCLUSION.

Page 33: CRITICAL REASONING 101 T - Amazon S3 · Critical Reasoning Section on the GMAT and GRE will require that you draw conclusions that are logically certain for a set of stated or implied

© Maple Leaf International Consulting, Inc., New York New York. For exclusive use by our registered participants only. Not to be copied or reproduced without our consent.

Page 33

The SCOPE of the argument is the set of activities that the SUBJECT is described as pursuing or engaging in, and/or all the FACTORS relevant to those specific activities. In the argument: “Tom received a GMAT score of 400 out of 800 on the GMAT. Therefore, Tom will not be admitted to Harvard Business School.”, the SCOPE is limited to “the GMAT score of 400, and admission to Harvard on the basis of this score”. Also, any other factor or factors that may be relevant to admission to Harvard Business school in addition to or instead of the GMAT score will be part of the scope of the argument. What other factors may play out in the admission decision or in the admission process? Tom’s G.P.A., or his entrepreneurial background, or his motivation, or his excellent recommendations. What will be considered NOT within the SCOPE of the argument? ♦ Peter’s GMAT score and his admission to Stanford. ♦ Tom’s GRE score and what he could do with it. ♦ And so on… You should bear in mind that “OTHER SUBJECTS” are strictly off limits and must be dealt with as IRRELEVANT. “OTHER FACTORS” may be used to suggest that the activities described in the argument may be impacted by OTHER FACTORS relevant to the same SUBJECT and the defined SCOPE. When you come across the term “OTHER” in answer choices, be sure to check it to see whether “OTHER” modifies an extraneous SUBJECT or modifies ‘FACTORS’ relevant to the activity described. The latter may be acceptable but the former will not be.

The first step in the procedure you need to follow when you deal with a critical reasoning argument is to get the hang of the SUBJECT and the SCOPE of activities pursued by the Subject, or of the factors having an impact on the

activities pursued by the SUBJECT. We will also learn in the following pages how we can identify “out of scope” information, and how we can eliminate them out of hand. You should keep in mind that intelligent selection involves intelligent elimination of answer choices that use

information not relevant to the scope of the argument as described.

Page 34: CRITICAL REASONING 101 T - Amazon S3 · Critical Reasoning Section on the GMAT and GRE will require that you draw conclusions that are logically certain for a set of stated or implied

© Maple Leaf International Consulting, Inc., New York New York. For exclusive use by our registered participants only. Not to be copied or reproduced without our consent.

Page 34

STEP 2: Pay attention to the QUALIFIERS used to described the SUBJECT and the SCOPE of the argument. The QUALIFIER could be a simple adjective or a predicate adjective, but regardless of the type of qualifier used, you should keep in mind that the qualifiers define a very narrow subset of a larger set. Also, the qualifier may be subject specific or scope specific. You will be expected to pay attention to these various ‘subject-critical’ and ‘scope-critical’ adjectives, and to stay away from straying from these narrow definitions. Consider the following argument: “Very overweight people whose metabolism remains sluggish generally tend to lose weight through dieting, which involves carefully controlling the intake of carbohydrates and fats.” As you can see, the subject is “very overweight people”, not just ‘any people’, and the scope is limited to losing weight through dieting, not through working out or through physical activities. The answer choice you are tempted to pick must be relevant to this narrowly defined subset of people and the scope, namely, losing weight through dieting. Consider the following argument: “Businesses that make huge profits have a moral obligation to reinvest bulk of their profits in the community that helped businesses make those profits in the first place.” As you can see, the subject is not ‘any business’, but ‘businesses that make huge profits’. Also, the scope is about a specific category of obligations: It is not about a legal obligation or a business obligation. It is about a moral obligation. The answer choice you are tempted to pick must make a reference to these two qualifiers, not to any other variation of qualifiers. For example, you cannot pick a choice that talks about “businesses that lose money year in and year out” or about “legal obligations of businesses”. These qualifiers define a subject and a scope not addressed in the original argument, and must be construed as going beyond the limited scope of the argument. You might want to make a note of the qualifiers that define the subject and the scope on your scratch paper, and stay focused on this aspect of the argument. You can bet your bottom dollar that the answer choices will have different variations of the qualifiers to define the same subject. Also, if the original argument does not use any qualifiers to define the subject or the

The only logic that will be used throughout Critical Reasoning is the following: “If all of X is Y, all of Y is not necessarily X. But at least some of Y is X”. A real world translation of this logic is as follows: “If LSAT is a standardized test, not all standardized tests are necessarily LSAT. But at least some standardized tests taken by the test takers could be LSAT.” Another example: “If all married people are people, not all people are necessarily married. But at least some people will be married folks.” In view of this logic, if the argument is about ‘male bower birds’, any answer choice that talks about ‘birds’ must be considered illogical.

Page 35: CRITICAL REASONING 101 T - Amazon S3 · Critical Reasoning Section on the GMAT and GRE will require that you draw conclusions that are logically certain for a set of stated or implied

© Maple Leaf International Consulting, Inc., New York New York. For exclusive use by our registered participants only. Not to be copied or reproduced without our consent.

Page 35

scope, you cannot pick a choice that talks about a qualified subject or scope. If, for example, the argument talks about “airliners operating in the United States”, you cannot pick a choice that talks about “cargo airliners operating in North America” or about “executive jets flying between major cities in the United States”. The original argument does not make such distinctions, and the chances are good that you are looking at a bad answer choice. “PEOPLE” could define a larger universe of population. “Very overweight people” will define a very narrow subset of the larger set called PEOPLE. ♦ “BIRDS” define a larger universe of flying creatures, but ‘BOWER

BIRDS’ define a narrower set, and ‘MALE BOWER BIRDS’ define an even narrower set.

♦ “Collision warning systems” define a type of ‘warning systems’ but “NOT FULLY TESTED COLLISION WARNING SYSTEMS” define a very narrow subset of the “collision warning systems”.

♦ “Employees” define a larger universe of workers in a company, but “newly hired employees” define a very narrow subset of the larger universe.

♦ “Married people” defines a larger universe of people in a marital relationship, but “Married people adopting different sleeping waking cycles” defines a narrow subset of the category of married people. Notice the predicate adjective used as a qualifier to define “married people”.

♦ “People” defines a broad category of humans. “Overweight people” narrows down the type of humans we are dealing with. “Overweight people who lose weight through dieting” defines a really narrow subset of the category of “overweight people”. Notice again the predicate adjective “who lose weight through dieting” used to define the subject called “overweight people”.

GMAT or LSAT or GRE is going to test your ability to pay attention to the qualifiers used to define the SUBJECT and the SCOPE of the argument and to pick a choice that uses the same qualifiers, not any other. The test, for the most part, will require that you PAY ATTENTION and not go to sleep at the switch. You should bear in mind that if the subject of the argument is “married people adopting different sleep-wake cycles”, then an answer choice that talks about “people adopting different sleeping waking cycles” or about “married people adopting the same sleep-wake cycles” will be considered extraneous, and not within the scope of the argument.

Try to be a QUALIFIER POLICE. Make sure that the ‘qualifiers’ used in the evidence and in the conclusion are reasserted in the answer choice or that the answer choice is relevant to the specific qualified information used in the argument. Also, be concerned about any MISSING QUALIFIERS in the answer choices or about any NEW or Additional Qualifiers appearing in the answer options. If the argument is about ‘married people’, any answer choice that talks about ‘people’ will be irrelevant to the argument. For the same reason, any option that talks about ‘NEWLY married people’ is questionable too.

Be aware of the possibility that the adjectives used in the original argument could be stated in the answer options in a qualifier form. For example, ‘wealthy individuals’ could appear in the correct answer choice as ‘individuals who are wealthy’. Similarly, the qualifiers may sometimes be paraphrased. If the argument talks about ‘would be aggressor nation’, the answer choice might talk about ‘nations that are potential aggressors’. Notice that ‘would be’ is paraphrased to ‘potential’.

Page 36: CRITICAL REASONING 101 T - Amazon S3 · Critical Reasoning Section on the GMAT and GRE will require that you draw conclusions that are logically certain for a set of stated or implied

© Maple Leaf International Consulting, Inc., New York New York. For exclusive use by our registered participants only. Not to be copied or reproduced without our consent.

Page 36

ANSWER SELECTION STRATEGY for LOGICAL/CRITICAL REASONING Critical Reasoning (Logical Reasoning in LSAT) is about your ability to make the distinction between what is just right and what might be ‘too specific’ or ‘too vague/ too unspecific/ too broad’, conditional, etc. Be sure to examine the answer choices and to spot statements of the following types so that you can avoid them like you would avoid plague. As a rule, watch out for answer choices that are VAGUE, UNSURE, UNSPECIFIC, in passive voice using EXPLETIVE form (Expletive

construction uses the dummy noun IT or THERE – “it is long believed that….”, “there was a study indicating….”)

TOO SURE, TOO SPECIFIC, EXTREME POLITICALLY INCORRECT (As in ‘products made in China are of low quality’ or ‘women are

unfit to be in combat role’). SPECULATIVE AND UNVERIFIABLE (as in ‘predict’, ‘project’, ‘expect’, ‘CLAIM’) NOT COMMONSENSICAL – any answer option that is clearly not commonsensical cannot

be a good answer. For example, an answer stating that the ‘human boxes are likely to be more effective if they are LESS visible to SOME wood-ducks than they currently are’ is not commonsensical – how does anyone make the boxes less visible to some ducks and more visible to others?

COMMONSENSICAL but EXPECTATIONAL --- these answer options are deadly ones because they distract you and tempt you to pick them using your real-world expectations. Watch out for such answer choices. Usually, these answer options will be consistent with a fact or belief about the subject of the argument but not relevant to the scope of the argument. Test-takers are easily distracted when they come across passages about children, drugs, wars, business, insurance, cars, etc., and pick an answer that agrees with their real world expectations. Stay alert and do not be distracted.

Restatement of evidence / embellishment of evidence / contradiction of evidence. (If the evidence is ‘John received a GMAT score of 550’, and if an answer choice for a weakening or strengthening or assumption question reads “the maximum score on the GMAT is 800” or “John received a GMAT score of 700 two years ago”, you are dealing with answer choices that essentially restate evidence or contradict evidence or embellish evidence). The evidence cannot be questioned. Only the conclusion or the assumption leading to the conclusion can be questioned.

CONDITONAL OR HYPOTHETICAL (SUCH AS ‘ONLY IF’ OR ‘EVEN IF’ OR ‘IF JOHN HAS A GOOD GPA’) – ESPECIALLY FOR WEAKENING OR STRENGTHENING QUESTIONS.

Adverbial Emphasis provided by the ‘LY’ qualifiers. Most of the time, such an emphasis is not required and inappropriate. Any answer choice using LY qualifier must be critically examined. Also, remember that if the passage uses an LY qualifier, it is important for you to process this qualifier as it expresses an emphasis that the passage requires you to process.

Additional adjectives or missing adjectives -- If the argument is about ‘private black colleges in the south’, your answer must be relevant to ‘private black colleges in the south’. If there are missing qualifiers (“private black colleges” or ‘’Black colleges” or ‘private colleges’ or ‘colleges in the south’), you need to know that you are looking at a bad answer choice. On the other hand, any additional qualifier is problematic too: if the answer is about SMALL private black colleges in the south, the added qualifier is troublesome. PAY ATTENTION. That is what the test requires. Do not just go through the motions of picking answers as a zombie would do. Stay up, pay attention, and engage in critical analysis.

POLITICIAN’S ANSWERS -- usually in the context of ‘strengthening’ or ‘weakening’ questions. Answers of this type will take the form ‘X is good because Y is bad’ or vice versa. “Alternative medicine is effective because orthodox medicine is sometimes ineffective’ is not a good answer for anything. Politicians use this form of argument a lot: “If you do not support my tax cut plan, you are for tax increases’ is not a good argument; maybe, you are for status quo and no change in tax rates. Recognize the politician’s answer when you see one.

Page 37: CRITICAL REASONING 101 T - Amazon S3 · Critical Reasoning Section on the GMAT and GRE will require that you draw conclusions that are logically certain for a set of stated or implied

© Maple Leaf International Consulting, Inc., New York New York. For exclusive use by our registered participants only. Not to be copied or reproduced without our consent.

Page 37

A NOTE OF CAUTION ABOUT ADVERBS (‘LY’ DETECTION) The ADVERBS bring on an EMPHASIS to the adjectives or verbs used in a statement, and any emphasis in the context of an answer option is usually suspect. As a rule, you must watch out for ‘LY’ adverbial forms used in answer choices, and examine the answer choices for possible incorrect use of adverbial qualifiers. Adverbs such as successfully, precisely, indefinitely, exactly, significantly, steadily, and roughly generally render an answer option incorrect. As a rule, any answer option using any adverb or adjective needs to be closely scrutinized. If you have 100 answer choices using an ‘LY’ adverb, it is a fair bet that 95 of them will be incorrect and will inappropriately use the ‘ly’ adverb. Examples of statements using the ‘LY’ adverbial forms, and that will raise a red flag: Company X has more HIGHLY PROFITABLE products than does

company Y. Married couples whose sleeping and waking cycles SIGNIFICANTLY

DIFFER from those of their spouses. Company’s sales will grow INDEFINITELY The number of people using XM radios will STEADILY INCREASE. People submitting EXTREMELY LOW GMAT Scores are

OCCASIONALLY admitted to leading Business Schools. Note that a correct answer choice may contain an adverb that is appropriately used but any answer choice containing an adverb must be scrutinized critically and checked out for any improper use of adverbial qualifiers. STAY FOCUSED ON THE QUALIFIERS provided in the argument and be sure to make your answer choice relevant to the qualifiers used. As a rule, any missing qualifier or additional qualifier in the answer option is problematic. If the argument is about ‘deterring would-be aggressor nations’, your answer cannot be about ‘deterring any nation on earth’. If the argument is about ‘the effect of warm weather conditions on agricultural production in northern Asia’, your answer cannot be about ‘effect of weather on agriculture in Asia’ or ‘the effect of UNUSUALLY warm weather on CORN production in SOUTHERN Asia’. Be a QUALIFIER POLICE and make sure that your answer is relevant to the specifics used in the argument.

Page 38: CRITICAL REASONING 101 T - Amazon S3 · Critical Reasoning Section on the GMAT and GRE will require that you draw conclusions that are logically certain for a set of stated or implied

© Maple Leaf International Consulting, Inc., New York New York. For exclusive use by our registered participants only. Not to be copied or reproduced without our consent.

Page 38

The following table shows what is just right and what descriptions will be either too vague or too specific.

INFORMATION THAT IS TOO SPECIFIC

INFORMATION THAT IS JUST RIGHT AND AS DESCRIBED IN THE ARGUMENT

INFORMATION THAT IS TOO UNSPECIFIC OR TOO BROAD OR TOO VAGUE

Married persons having children

Married persons persons parents

GMAT GRE LSAT

Standardized tests tests exams contests

bald eagles trapped in fishing nets

Birds that get trapped in fishing nets

birds creatures of nature wild things

collision warning systems used on cargo planes

Collision warning systems

mechanical devices

electronic devices As a rule, answer options using the following words and phrases will make the answer choice ‘suspect’ in respect of any WEAKENING or STRENGTHENING question, and must be examined further before you confirm them. If you are in doubt, it is a good idea to ‘stay away’ from answer options using any of the following words and phrases”. (Put them on a flash card and memorize them; also, consciously look for them during the test, and critically examine options using them.) AVOID / CHECK OUT CLOSELY GOOD PHRASES occasionally / sometimes / rarely (VAGUE/UNSURE) most always/ whenever (TOO SURE) at least some all only a few some if or even if or only if (CONDITIONAL) other (OTHER COUNTRIES, OTHER BIRDS, ETC) better/ newer/ larger/ or any DEGREE COMPARISONS

(If the argument is that ‘handwriting analysis is an ineffective way of predicting personality types’, it does not imply that there are other techniques that are MORE EFFECTIVE. Watch out for DEGREE COMPARISONS that turn out to be ILLOGICAL interpretations of Statements in the arguments). only (incorrectly used 98% of the time) primary reason or purpose; only reason (speculative) majority (hackneyed; abused word) whenever (too sure) despite ‘ACCORDING TO A STUDY’ / ‘ACCORDING TO LAW’ ‘SOMETHING WILL REMAIN CONSTANT/STEADY/UNCHANGED’ total (as an adjective) predict / project/ expect (speculative) DATA (NOUN) (hackneyed; abused word) Statistics / probability / average (hackneyed; abused words) Price / competition ‘success’ or successfully

Also, be skeptical of any ‘added’ qualifiers and of ‘any missing qualifiers’ in the answer choices. If the argument is about ‘married people’, any choice referencing ‘newly married people’ will be considered too narrow in scope. Likewise, any answer choice that describes ‘people’ will also be questionable. Also, watch out for the adjective ‘other’. If the argument is about ‘Germany’, any answer choice referencing ‘other European countries’ will be irrelevant to the scope of the argument. Remember: This is a test of your ability to choose a statement containing an acceptable choice of words put together.

Watch out for ‘passive voice statements’ such as ‘Obesity is classified as a disability’. The problem with this statement is that it is vague inasmuch as it does not state ‘by who’? Also, watch out forpolitically incorrect statements in answer choices

Page 39: CRITICAL REASONING 101 T - Amazon S3 · Critical Reasoning Section on the GMAT and GRE will require that you draw conclusions that are logically certain for a set of stated or implied

© Maple Leaf International Consulting, Inc., New York New York. For exclusive use by our registered participants only. Not to be copied or reproduced without our consent.

Page 39

PROCESS THE QUESTION WELL AND REMAIN FOCUSED ON THE QUESTION AT ALL TIMES Most test-takers do not process the question well. As a result, they end

up picking an answer that may be relevant to the scope of information provided but not related to the question. Any answer that BEGS the question is not a good answer.

Some test-takers like to read the question first before they read the argument. There is nothing wrong with this strategy except that if you are one of those mortals, be sure to read the question again and process it well. Most test-takers forget the question while they are in the middle of processing the argument, and end up selecting an answer that is not relevant to the question.

Understand the Question you are required to answer. In some instances, REPHRASE the question. We will discuss in the next few pages how GMAT or LSAT will try to make the question more ‘challenging’ by using some convoluted phrasing of the question. Expect that some answers will do the weakening or strengthening of the argument better than the others. Pick the COMPLETE answer, not the partial one. Also, expect STRENGTHENING information for WEAKENING question, and vice versa, in the answer choices. KEEP YOUR EYES ON THE BALL, AND STAY FOCUSED ON THE SPECIFICS OF THE ARGUMENT AND ON THE SPECIFICS OF THE QUESTION. You may disregard this advice at your own peril.

PROCESS information in context by asking two questions: What is being said? And Why is it being said? Remember that a statement such as conclusion needs to be processed in context and in terms of the setting in which the statement is made. If the conclusion is that the Universities should control ‘grade inflation’, we need to know ‘why do you say that?’ Because, nearly 90% of graduates at leading Universities graduate with ‘honors’ degrees. The argument implies that those who graduate with ‘honors’ qualifications receive INFLATED GRADES and do not naturally EARN them. This implied information becomes the assumption on which the argument rests.

Page 40: CRITICAL REASONING 101 T - Amazon S3 · Critical Reasoning Section on the GMAT and GRE will require that you draw conclusions that are logically certain for a set of stated or implied

© Maple Leaf International Consulting, Inc., New York New York. For exclusive use by our registered participants only. Not to be copied or reproduced without our consent.

Page 40

WORDING OF THE VARIOUS TYPES OF QUESTIONS WEAKENING questions can be phrased in any of the following ways: (they all use the qualifier ‘if true’) -- Which of the following UNDERMINES the argument?

♦ Which of the following ARGUES AGAINST the conclusion? ♦ Which of the following COUNTERS the conclusion? ♦ Which of the following PRESENTS A CONSIDERATION

ARGUING AGAINST the conclusion? ♦ Which of the following suggests that the CONCLUSION

SHOULD NOT BE IMPLEMENTED? The difficulty level of the questions can be enhanced by framing the question differently. If the question, ‘which of the following would weaken the argument?’ were rephrased, ‘which of the following would argue against the commission’s proposal that tolls be not raised on bridges to augment the city’s revenues’. Notice that the use of ‘double negative’ in the question stem makes for difficult reading and processing of information. Paraphrase the question to a simpler form such as “Why should the tolls be raised on bridges to augment the revenues even though the commission has suggested otherwise?” ♦ STRENGTHENING QUESTIONS can be phrased in any of the following

ways, using the qualifier ‘if true: ♦ Which of the following STRENGTHENS the argument? ♦ Which of the following PROVIDES SUPPORT to the

argument? (if the qualifier ‘if true’ is missing, then it is a logical conclusion question. The strengthening question will read: Which of the following, IF TRUE, will support the argument? Whereas the LOGICAL CONCLUSION or INFERENCE question will read: “The passage supports which of the following?” or ‘which of the following is SUPPORTED by the passage?”). Answer supporting the argument is a strengthening response; if the passage supports the answer, then the answer is logical conclusion. Remember this distinction and do not be distracted if you notice the word SUPPORT in the question stem.)

♦ Which of the following offers a consideration in SUPPORT of the conclusion?

♦ Which of the following is an ASSUMPTION underlying the argument?

Page 41: CRITICAL REASONING 101 T - Amazon S3 · Critical Reasoning Section on the GMAT and GRE will require that you draw conclusions that are logically certain for a set of stated or implied

© Maple Leaf International Consulting, Inc., New York New York. For exclusive use by our registered participants only. Not to be copied or reproduced without our consent.

Page 41

♦ ASSUMPTION question will be phrased in any of the following ways:

♦ Which of the following is ASSUMED in the argument? ♦ Which of the following is the ASSUMPTION in the

argument? ♦ Which of the following is IMPLIED in the argument? ♦ Which of the following is an unstated PREMISE of the

argument? (Remember that the stated PREMISE is the evidence. The assumption is implied and unstated.)

(Remember that Assumption is a QUESTIONABLE INFERENCE supported by a questionable statement (conclusion) or by two statements at least one of which is questionable) ♦ LOGICAL CONCLUSION or INFERENCE questions can be phrased in

any of the following ways: ♦ Which of the following can be LOGICALLY CONCLUDED? ♦ Which of the following PREDICTION is supported by the argument? ♦ Which of the following INFERENCES can be made on the basis of the

information above? ♦ Which of the following EXPLAINS what is described in the argument? ♦ Which of the following IS SUPPORTED BY the information in the

argument? Do not be misled by the use of the verb ‘SUPPORT’. Strengthening questions that use the verb ‘SUPPORT’ typically use the qualifier ‘if true’. If the qualifier is missing, it is an inference question.

♦ The argument SUPPORTS WHICH OF THE FOLLOWING? (see the caveat above)

Questions asking you to identify a PARALLEL STRUCTURE will ask you: “The argument parallels which of the following?” or “Which of the following has the same structure as the given argument above?”

Page 42: CRITICAL REASONING 101 T - Amazon S3 · Critical Reasoning Section on the GMAT and GRE will require that you draw conclusions that are logically certain for a set of stated or implied

© Maple Leaf International Consulting, Inc., New York New York. For exclusive use by our registered participants only. Not to be copied or reproduced without our consent.

Page 42

COMPREHENSION OR THE PURPOSE OF A (set of) STATEMENT(s) QUESTIONS IN CRITICAL / LOGICAL REASONING We have seen earlier that processing of any information must be done by asking two questions: What is being said? Why is it being said? Questions under this category require that you do the following:

• Understand the relationship between the statements within a passage. • Identify the PURPOSE of a statement or a set of statements. The

purpose of a statement could be to describe something, or to discuss something, or to ‘disprove’ something, counter something, critique something, criticize something, or provide ‘context’ for another statement.

Careful parsing of the words used in an answer choice is critical to your ability to come up with a winner answer. The questions under this category could be based on any of the following settings:

1. Passage describing an argument and a counter argument, and question requiring that you identify the main point of agreement between the two, or the main point of disagreement.

2. Passage in which two sections are highlighted or underlined, and the question requires that you identify the function of the identified sections within the passage.

3. Passage describing a conversation between two people and the question requiring that you identify the manner in which one person responds to the other’s argument.

4. An argument is made and you are required to identify the PRINCIPLE that will support the crux of the argument. Note that the principle supporting the entire argument is different from the Strengthening question that basically tends to support the conclusion. The principle must be consistent with the entire argument.

5. An argument is described and you are required to IDENTIFY THE FLAW in the reasoning of the argument.

6. An argument is described and you are required to IDENTIFY an answer choice having the same PARALLEL structure or flaw or assumption.

Page 43: CRITICAL REASONING 101 T - Amazon S3 · Critical Reasoning Section on the GMAT and GRE will require that you draw conclusions that are logically certain for a set of stated or implied

© Maple Leaf International Consulting, Inc., New York New York. For exclusive use by our registered participants only. Not to be copied or reproduced without our consent.

Page 43

QUESTIONS ASKING YOU TO RESOLVE A PARADOX OR TO EXPLAIN THE DISCREPANCY Questions under this category will require that you understand the paradox or the discrepancy described in the passage and to identify an answer that will RESOLVE the paradox or LOGICALLY EXPLAIN the discrepancy presented. A paradox is a statement that is seemingly contradictory or opposed to common sense and yet is perhaps true. Consider the following paradox or discrepancy: “Cars built since 1990 all have airbags and mechanical features that are meant to prevent fatalities during head-on collisions. Yet, the number of fatalities per 1000 drivers arising from head-on collisions involving cars having the safety features has remained constant or gone up since 1991.”. Our logical explanation is that the cars may be safe but the drivers may be lulled into a false sense of security making them drive recklessly (thereby leading to fatalities as described). Consider another paradox presented in a passage: ‘Products sold under brand name neither offer higher quality not command premium pricing than those sold under generic names. Yet, brand names are still viewed by companies as offering a huge marketing advantage” How do we explain this paradox that brand names offer a huge marketing advantage while not commanding premium prices or offering better quality? Our literal interpretation of the term ‘marketing advantage’ allows to conclude that products having a brand name are easy to sell to consumers. A logical explanation for why consumers will still reach for a branded product is that, all things being equal—price, quality, etc.,- a branded product is seen as a known devil and a safe bet. You will look for an answer along these lines. PROCESS THE PARADOX or the DISCREPANCY well and understand what it is that you are trying to EXPLAIN. Questions asking you to explain the paradox or the discrepancy at once test your comprehension and your ability to logically explain an apparent discrepancy. In the final analysis, the test is about how you process information and about whether you understand the question in order to be able to identify an answer that is relevant to the specifics mentioned in the passage and to the question. Stay alert, and avoid distraction. The expectation by the testing company is that a person who is easily distracted and who takes his or her eyes off the ball is not likely to make a sound decision. We at MLICETS concur.

Page 44: CRITICAL REASONING 101 T - Amazon S3 · Critical Reasoning Section on the GMAT and GRE will require that you draw conclusions that are logically certain for a set of stated or implied

© Maple Leaf International Consulting, Inc., New York New York. For exclusive use by our registered participants only. Not to be copied or reproduced without our consent.

Page 44

• IDENTIFICATION OF FLAW IN THE ARGUMENT: Questions of this type will read, “which of the following identifies the FLAW in the argument or in the line of reasoning?” The flaw is a defect that detracts from the strength or the gravity of the argument. The flaw could be an interchangeable use of different qualifiers or the use of a word that could mean two different things in two different contexts. For example, if the statement in the argument was, ‘we should pass legislation to punish EXPLOITATION of workers in the same manner as we have passed legislation to punish EXPLOITATION of pristine natural resources’, you should notice that the word ‘exploitation’ when used in the context of workers suggests potential ‘abuse’ or ‘taking advantage’ whereas ‘exploitation’ as used in the context of natural resources implies ‘making full use of’. There is a connotational difference in the use of the same word in two different settings. The flaw could be that the argument interchangeably uses two different contexts alike. In an earlier discussion, we highlighted the need to pay attention to and process deliberate ambiguities that are woven into the statements provided to you. Actively look for and process such subtleties. PRINCIPLES QUESTIONS (TYPICALLY TESTED ON THE LSAT)

Practice of law is about identifying core principles, rulings, and precedents that will lend support to a cause of action that is a prerequisite for making a case out of anything. In the United States, the elements of the constitution – both explicit wording and implied but necessarily valid interpretation of it – provide the legal basis or constitute the principles backing up a line of reasoning or arguments. LSAT will ask you to understand a line of reasoning set forth in an argument and to identify a principle that would necessarily support the line of reasoning. You may want to look at questions under this category as similar to Strengthening questions. The questions under this category will typically go as follows: “Which of the following principles will necessarily support the rebuttal offered by Samantha?”

Page 45: CRITICAL REASONING 101 T - Amazon S3 · Critical Reasoning Section on the GMAT and GRE will require that you draw conclusions that are logically certain for a set of stated or implied

© Maple Leaf International Consulting, Inc., New York New York. For exclusive use by our registered participants only. Not to be copied or reproduced without our consent.

Page 45

DIFFICULTY LEVEL ASSOCIATED WITH AN ARGUMENT IS A FUNCTION OF DIFFICULTY CREATED IN THE WORDING OF THE QUESTION Now that we know that comprehension is tested in one form or another, you should expect the test to challenge your comprehension by phrasing the question in somewhat ‘incomprehensible’ manner. The difficulty level of a question is also predicated on the ‘difficulty’ associated with the ‘phrasing’ of the question. Consider the following two questions:

• WHICH OF THE FOLLOWING WOULD STRONGLY UNDERMINE THE ARGUMENT?

• WHICH OF THE FOLLOWING PRESENTS A CONSIDERATION ARGUING AGAINST WHY THE COMMISSION’S PROPOSALS SHOULD NOT BE IMPLEMENTED?

Can you see that the second question is unarguably more challenging than the first one? Notice the double negative: “argue against’ and ‘not’. If you encounter questions of this nature, be sure to PARAPHRASE and SIMPLIFY the question to a more manageable form such as: “WHICH OF THE FOLLOWING IS A REASON WHY THE COMMISSION’S PROPOSAL SHOULD BE IMPLEMENTED?” Notice that the paraphrased question is more manageable than the original one. The anatomy of a challenging or even ‘terrorizing’ question typically involves ‘double negatives’, or ‘not, but’ form, or ‘conclusion bundled into the question’. Consider the following annoying question in which the conclusion is bundled into the question:

• ARGUMENT: The publishing company wants to cut in half the number of issues of its magazine that it publishes each year. The quality and the number of articles will remain the same. The company expects that neither subscribers nor advertisers will be lost as a result of this move.

• QUESTION: Which of the following, if true, would strongly argue against the company’s expectation that profits will be maximized if the plan is put into effect?

You have to process the question in such a way that you separate the BUNDLED CONCLUSION from the question itself. You will read the conclusion as part of the argument as follows and then process the question as shown.

• ARGUMENT: The publishing company wants to cut in half the number of issues of its magazine that it publishes each year. The quality and the number of articles will remain the same. The company expects that neither subscribers nor advertisers will be lost as a result of this move. The company also expects that, if this plan is implemented, PROFITS WILL BE MAXIMIZED.

• QUESTION: Which of the following, if true, would suggest that the profits will NOT be maximized if the magazine cuts in half the number of issues of the magazine that it publishers?

Notice that the ‘cut and paste’ effort makes it a lot easier to process.

Page 46: CRITICAL REASONING 101 T - Amazon S3 · Critical Reasoning Section on the GMAT and GRE will require that you draw conclusions that are logically certain for a set of stated or implied

© Maple Leaf International Consulting, Inc., New York New York. For exclusive use by our registered participants only. Not to be copied or reproduced without our consent.

Page 46

UNDERSTAND THE VARIOUS ARGUMENT TYPES Understanding the different types of illogical arguments that will be tested on the GMAT, GRE, and LSAT will help you identify the correct response more effectively. Broadly, GMAT, GRE, and LSAT will test arguments that mostly fall into one of the following major categories. In fact, your ARGUMENT essays will also use a line of reasoning that will have characteristics of one or more of the following types of arguments. When you are asked to IDENTIFY the Assumption of an argument, it will be useful for you to identify the type of argument that is being dealt with. IF the argument relies on a sample, then the argument assumes that the sample is representative. If the conclusion is drawn on the basis of consideration of a single factor, the argument implies or assumes that the advantages or disadvantages of the single factor may not be offset by the corresponding disadvantages or advantages associated with other relevant factors. Each argument type will require that you use a ‘different’ strategy of attack in respect of identifying the assumption or of weakening it. It is possible for an argument to have more than one ‘type’ connotation. Consider the following argument: “Compared to a decade ago, people now are not as concerned about regulating their daily intake of red meat and fatty cheeses. This is evident from the fact that an Organic Food Store that opened a decade ago has started selling high-butter fat content cheese last year. The owner of the HOUSE OF BEEF is a millionaire whereas the owner of VEGGIE HOUSE, a vegetarian restaurant in the same block, is making modest profits.” Notice that the conclusion is stated in the first sentence of the argument, and you must identify the conclusion and the evidence used in the argument.. (Remember: The conclusion is not always stated at the end of the argument. You could start with the conclusion and then present the basis for that conclusion, Evidence) If we take a look at the evidence that ‘the owner of House of Beef is a millionaire whereas that of a veggie place is making a modest living’, the implication of this statement is obvious: The owner of House of Beef became a millionaire as a result of the restaurant business referred to. This is a classic ‘chicken and egg argument’ identified below. Maybe, the owner was always a millionaire who opened a restaurant. This evidence also could qualify for consideration under the category “one explanation for the evidence versus another”. The implied explanation for the ‘millions’ is that he is running a restaurant; an alternative explanation for the same evidence could be that he is making his millions through other business ventures that he could be simultaneously managing. Understanding the type of argument is critical to your ability to come up with an ‘attack’ strategy. Let us now review the commonly tested types of arguments.

Page 47: CRITICAL REASONING 101 T - Amazon S3 · Critical Reasoning Section on the GMAT and GRE will require that you draw conclusions that are logically certain for a set of stated or implied

© Maple Leaf International Consulting, Inc., New York New York. For exclusive use by our registered participants only. Not to be copied or reproduced without our consent.

Page 47

Cause and Effect arguments (Chicken and Egg arguments): These arguments will be of the form ‘X causes Y’. You need to consider the possibility that --Y may have caused X, or --a third factor may have created both X and Y, or --a third factor may have cause Y, with X being coincidental. IF the evidence is that ‘most smokers snore’ and the conclusion is ‘therefore, smoking causes snoring’, you need to consider the possibility that ‘snoring caused smoking’ (does not make sense) or that another factor such as ‘stress causes both smoking and snoring’. If the evidence is that ‘employees in corner offices are more productive’ and the conclusion is that ‘corner offices make employees more productive’, you need to question this ‘interpretation of the evidence’ by suggesting that ‘one has to be productive to get the corner offices’ (which one came first: the chicken or the egg? Or, were they both created simultaneously by the ‘creator’?) If the argument is that a study of married people shows that marriages in which both spouses work outside the home are ended in divorce after three years of marriage leading to the conclusion that two-income situation leads to a break-up of marriages, consider the possibility that the couples in a marriage recognize that the marriage is going nowhere in a hurry and take up jobs in order to be financial secure and independent.

Page 48: CRITICAL REASONING 101 T - Amazon S3 · Critical Reasoning Section on the GMAT and GRE will require that you draw conclusions that are logically certain for a set of stated or implied

© Maple Leaf International Consulting, Inc., New York New York. For exclusive use by our registered participants only. Not to be copied or reproduced without our consent.

Page 48

One explanation for the evidence versus another explanation : These arguments will ‘interpret’ the EVIDENCE in one fashion. You may want to question this ‘lone’ interpretation by suggesting ‘other logical interpretations’. If the evidence is that ‘women have the same chance of winning elections as men and yet women do not run for elective offices’, and the conclusion is that ‘women do not run for elective offices because they do not WANT to run’, you need to consider another explanation for why ‘women are not running for elective offices even though they have the same chance of winning elections’. Another explanation not addressed in the conclusion could be that ‘women do not receive adequate funding for running a successful campaign (but they do want to run)’. SINGLE FACTOR ARGUMENTS that reach a conclusion on the basis of examination of a SINGLE FACTOR: GOOD FACTOR / BAD FACTOR : An argument is considered a ‘good factor’ argument if on the basis of a SINGLE FACTOR, it reaches a GOOD CONCLUSION. If the evidence identifies ‘one good factor’ and the argument concludes that the ‘good factor makes it worthwhile to pursue the activity’, you need to attack this line of reasoning by coming up with a ‘bad factor’. If the evidence is that ‘Tom received a GMAT score of 750’ and the conclusion is that ‘Tom will be admitted to Harvard Business School’, you need to question the conclusion by suggesting that Tom may have a bad factor such as ‘applying late’ or ‘have a poor recommendation’. On the other hand, if the argument reaches a BAD conclusion on the basis of examination of a single factor, then the argument will be considered a BAD FACTOR argument. Example: “John received an LSAT Score of 140, and will NOT be admitted to Harvard Law School”. Notice that the conclusion is BAD outcome for John and the factor leading to this outcome is a bad factor. The assumption is that ‘John cannot use his other positive factors to offset the negative effect of the LSAT score of 140’. There could be other factors such as a good GPA, legacy, good recommendations, or lack of emphasis on test scores, and the cumulative effective of these other factors could outweigh the impact of an LSAT score of 140. OLD PROBLEM / NEW IMPERFECT OR PARTIAL SOLUTION: If the evidence is about a new solution to an old problem, and the conclusion is that the old problem will not fester, you need to attack this conclusion by suggesting that the problem may not be aggravated but the old problem will persist. If the evidence is that ‘all countries agreed not to produce chemical weapons from here on’ and the conclusion is that ‘the threat posed by chemical weapons is ELIMINATED’, we should question this conclusion about the ‘ELIMINATION OF THE THREAT’ by asking ‘what about the existing stockpile of chemical weapons? We may not produce more but may have enough to kill the entire world ten times over’.

Page 49: CRITICAL REASONING 101 T - Amazon S3 · Critical Reasoning Section on the GMAT and GRE will require that you draw conclusions that are logically certain for a set of stated or implied

© Maple Leaf International Consulting, Inc., New York New York. For exclusive use by our registered participants only. Not to be copied or reproduced without our consent.

Page 49

OLD VERSUS NEW: If the argument is that “X is better because it is new”, you need to counter this by suggesting that the ‘new (technology)” may have bugs or may not be sufficiently tested in terms of reliability, or may be easily hacked into (Windows 2000 is more hacker-friendly than is Windows 98, as a counter example). BEFORE VERSUS AFTER ARGUMENTS: IF the argument recommends that a certain action be taken preemptively, you may want to argue that, maybe, doing it after the fact may be more prudent or cost effective. If vaccinating all of America for bird flu is going to cost a few billion dollars, and if the probability that a person can get bird flu is infinitesimal, then it may be more cost-effective to deal with any situation after the fact. Or, if the argument recommends that we wait until after a situation develops, you may want to argue why it may be useful to do it BEFORE the fact. SIZE ARGUMENTS – BIG VERSUS SMALL: IF the argument suggests that big is better or mightier, you may want to suggest that small may actually be more agile and more lethal. In a legal situation, if the prosecution argues that the defendant did perpetrate the crime against a smaller person because the defendant is BIGGER than the victim, the defense attorney may want to argue that the defendant merely responded to an act of provocation by the so-called small person, etc. QUANTITY VERSUS QUALITY ARGUMENTS The arguments of this type are also called NUMBERS ARGUMENTS. If the argument relies on evidence that is stated in numerical terms or in proportional terms, usually in the context of a comparison, be sure to do the following: Look at numbers in terms of PROPORTIONS or vice versa. For example, if the argument is that “the number of fatal accidents is the same in Montana and in Connecticut during a certain period and that Montana does not have speed limits in place whereas CT does; therefore, Connecticut should get rid of speed limits on its freeways”, you may want to argue that the same NUMBER, if looked at in terms of proportions, may present a different story. For example, 1000 fatal accidents in CT could represent 1% of the population of drivers on its highways whereas the same number could represent 7% of drivers on Montana highways. When the numbers are viewed as proportions, the comparison appears to be no longer ‘apples to apples’. Likewise, if the argument uses PROPORTIONS or comparison of proportions, look at the proportions in terms of numbers and see if it holds up. If the argument is that “one-third of all females in Canada smoke compared to 1/5th of all females in the U.S.A; therefore, there are more female smokers in Canada than in the USA”, you may want to argue that 1/3rd of a smaller value could still be far less than 1/5th of a larger value. (PROPORTION EXPRESSES A VALUE IN TERMS OF THE TOTAL POPULATION). If the comparison of numbers does not permit evaluation in terms of proportions, consider the possibility that the comparison could be pitting QUALITY against QUANTITIY.

Page 50: CRITICAL REASONING 101 T - Amazon S3 · Critical Reasoning Section on the GMAT and GRE will require that you draw conclusions that are logically certain for a set of stated or implied

© Maple Leaf International Consulting, Inc., New York New York. For exclusive use by our registered participants only. Not to be copied or reproduced without our consent.

Page 50

NUMBERS ARGUMENT --- QUANTITY VERSUS QUALITY If the argument compares the total population, then view the argument in terms of QUANTITY VERSUS QUALITY argument. Quantity versus Quality argument is also referred to as ‘vital few versus trivial many’ argument. If company X has 100 sales people and company Y has 70 sales people, can we argue that Company X will outsell company Y? We cannot because the comparison could turn on a comparison of quantity versus quality. In comparing quantity, you may also need to factor in quality of the items. VITAL FEW VERSUS TRIVIAL MANY (PARETO ARGUMENT): These arguments pit ‘many’ against ‘a few’, and argue that the ‘many’ will prevail over the ‘few’. We call it the Pareto argument because it has similarities to the Pareto principle that states that ‘80% of the work is done by 20% of the employees in a company’. Consider the following argument: “Company X and Company Y are selling widgets in the same market place and compete for the same customers. Company X has 100 sales people whereas Company Y has only 40. Therefore, Company X will outsell Company Y, and will be more profitable.” The problem with this line of reasoning is that Company X’s sales force may be poorly trained, poorly compensated, and poorly motivated sales team selling an inferior widget whereas Company Y’s sales force could be highly trained, handsomely compensated, and highly motivated group selling an excellent widget. The argument assumes that the ‘many’ is not trivial and that the ‘few’ cannot be vital. Your attack strategy must counter this assumption by suggesting that the Vital Few may prevail over the Trivial many. If the argument is that ‘the building timber discovered at the archeological site shows that the settlement was older than previously estimated’, your weakening response cannot be that ‘the archeologists found one piece of timber versus several pieces of pottery”. Remember that ‘more’ is not always ‘better’.

Page 51: CRITICAL REASONING 101 T - Amazon S3 · Critical Reasoning Section on the GMAT and GRE will require that you draw conclusions that are logically certain for a set of stated or implied

© Maple Leaf International Consulting, Inc., New York New York. For exclusive use by our registered participants only. Not to be copied or reproduced without our consent.

Page 51

SAMPLE / TRIAL / COMPARISON ARGUMENTS : Any argument that uses a sample as evidence to draw a conclusion about the larger population must be attacked by suggesting that the sample may not be typical of the larger population, or supported by confirming that the sample indeed speaks for the larger population. Such arguments usually use the phrase ‘in general’ or ‘generally’. If the evidence is that ‘a select group of employees who took part in a ‘work at home’ project showed the same or better productivity as if they stayed back in the office and worked’, and the conclusion is that ‘the company should make every employee work from home’, you need to attack the conclusion by stating that the sample group that is identified in the evidence may not be representative of the larger group. You can support the conclusion by asserting that the sample group identified in the conclusion does speak for the larger group. TRIAL ARGUMENTS: if the evidence is about a successful trial run, and the conclusion is about the viability of the project on a larger scale, you need to question whether or not the trial is ‘repeatable’. The GMAT and GRE test-takers should know that the algorithm used on the test will measure consistency of response by repeating questions at the same difficulty level in order to check whether a correct or incorrect response to a question at the same difficulty level can be repeated with another question at the same difficulty level. Pharmaceutical companies always treat a trial result as something that needs to be checked in a different scenario or setting or time period. A product that works in a sample of black Americans may not work in the same fashion with white Americans or vice versa. POLITICIAN’S ARGUMENTS: Arguments of this type are frequently resorted to by politicians everywhere. Arguments of this type use the form: “X IS GOOD BECAUSE Y IS BAD”. If the conclusion of an argument is that ‘alternative medicine is ineffective’, your weakening response cannot be that ‘even orthodox medicine is sometimes ineffective”. Your weakening response must be about why alternative medicine is effective. “Tories are good because Liberals are bad or Republicans are good because Democrats are bad” is an argument that is typically advanced by politicians and partisan political hacks. DO NOT ATTACK THE MESSENGER: Some answer options will fall into the category of ‘politician’s argument’. Do not be tempted to pick them. For example, if the evidence that the ‘tests on the newly found piece of lumber show that the civilization is older’, your weakening response cannot be that ‘the tests used to determine the age of lumber are unreliable or less accurate than those used to determine the age of other artifacts’. You cannot weaken an argument by weakening or attacking the evidence.

Page 52: CRITICAL REASONING 101 T - Amazon S3 · Critical Reasoning Section on the GMAT and GRE will require that you draw conclusions that are logically certain for a set of stated or implied

© Maple Leaf International Consulting, Inc., New York New York. For exclusive use by our registered participants only. Not to be copied or reproduced without our consent.

Page 52

COMPARISON ARGUMENTS: Such arguments compare two entities and a reach a conclusion on the basis of such comparison. The argument assumes that the entities compared are comparable in respect of factors that have a bearing on the stated conclusion. Take a look at the following comparison argument: “Country X and Country Y experience the same weather patterns throughout the year. Whereas the agricultural production in Country X has been increasing, that in country Y has been on the decline. Therefore, the economic policies of the regime in Country Y is to blame.” The argument assumes that the two countries mentioned are comparable with respect to factors that affect the agricultural production, factors such as types of crops grown, soil conditions, overuse of the farmland, lack of crop rotation, and so on. If it turns out that Country X grows tobacco whereas country Y grows opium and because the rest of the world frowns upon opium production accounting for the decline in country y, the comparison attempted in the argument is not well taken. SPECULATIVE OR CONSEQUENCE ARGUMENTS: Such arguments tend to SPECULATE about the consequence of information presented in the evidence. An example of such argument is: “John received a GRE score of 1000 in the two adaptive sections. Therefore, John will not be admitted to the graduate program in engineering at M.I.T.” Notice that the argument’s conclusion SPECULATES about the consequence of the evidence used in the argument. Notice also that the argument has qualities of ‘single factor (bad factor)’ mentioned previously. Your attack strategy must question such speculation (‘may be, M.I.T. does not require a score of more than 1000 on the GRE, or may be other factors in John’s favor could offset the negative effect, if any, of the GRE score of 1,000). Any argument that uses ‘cost’ as evidence and ‘profits’ as the conclusion must be dealt with in terms of an answer choice talking about ‘revenue’. (Revenue minus Cost = Profits). Any argument that uses ‘price’ as its evidence must be dealt with in terms of an answer choice that talks about ‘supply’ or ‘demand’. However, ‘higher demand’ may not be explained in terms of lower prices. In fact, the higher the demand, the higher the price is likely to be. If the evidence is that ‘the demand for imported cars is high’, you cannot explain this evidence by stating that ‘the price of imports is low’. It is possible that the demand may be high regardless of price. If Toyota quality is exceptional, you may want to buy the product even if it costs more.

Page 53: CRITICAL REASONING 101 T - Amazon S3 · Critical Reasoning Section on the GMAT and GRE will require that you draw conclusions that are logically certain for a set of stated or implied

© Maple Leaf International Consulting, Inc., New York New York. For exclusive use by our registered participants only. Not to be copied or reproduced without our consent.

Page 53

NON-SEQUITURS THAT DO NOT FALL INTO ANY SPECIAL SUB-CATEGORY: All other arguments that do not fall into one or the other categories identified above must be dealt with as non sequitur arguments. The strategy for dealing with such arguments is that you pay close attention to the specifics of the conclusion and to the basis on which the conclusion is drawn. You can then give a reason why the specifics of the conclusion are not valid. For example, if the evidence is that ‘The U.S. dollar has depreciated in relation to Euro and to the British Pound’ and the conclusion is that ‘U.S. manufactures can be more competitive on the global marketplace’, you can attack this conclusion by looking for a reason why the value of dollar may not give the competitive edge to the U.S. manufacturers that the argument claims that it will. Try to identify the type of argument that is presented to you so that you can come up with a response that is appropriate. The discussion presented in the preceding pages will show that each of the different types of arguments discussed requires a specific type of response. If you learn to recognize the type of argument that you are dealing with, it is a lot easier for you to identify the appropriate response to the question presented to you. You will also be better equipped to deal with the Analysis of Argument essays tested on the GMAT, LSAT, and GRE. If you can type the arguments into one of the above categories, you will be able to come up with the seven counter scenarios that are required to write a decent essay. _______________________________________________________________ In the following pages, we will discuss the specific procedures for dealing with the different types of questions that will be presented to you on the GMAT, LSAT, and GRE.

Page 54: CRITICAL REASONING 101 T - Amazon S3 · Critical Reasoning Section on the GMAT and GRE will require that you draw conclusions that are logically certain for a set of stated or implied

© Maple Leaf International Consulting, Inc., New York New York. For exclusive use by our registered participants only. Not to be copied or reproduced without our consent.

Page 54

WEAKENING OF AN ILLOGICAL ARGUMENT IS DONE BY ONE OF THE FOLLOWING TWO MEANS:

1. PROVIDE AN ALTERNATE EXPLANATION FOR THE EVIDENCE. ASK YOUR SELF: WHAT OTHER FACTORS COULD HAVE LED TO THE SAME OUTCOME AS DESCRIBED (FACTOR OTHER THAN THE ONE IDENTIFIED IN THE ARGUMENT). If the evidence is that ‘more than 2/3rd of high school graduates from Einstein High School enroll in a university course now compared to no more than ½ a decade ago’ and the conclusion is ‘therefore, teaching effectiveness has significantly improved at Einstein High School’, you should be able to recognize that the argument ‘attributes’ university enrollment after high school graduation to ONE factor: teaching effectiveness. Think of other factors that could have led to the same outcome as described in the evidence: factors such as affordability of college education, jobs requiring college degrees versus high school diplomas, changes in demographics of the community (south Asian families requiring their children to go to colleges), recessionary conditions in the job market making it difficult for high school graduates to find jobs soon after graduation and forcing them to do something including attend college until they can find a job, etc.

2. Any alternative explanation for the evidence will weaken the argument by suggesting that the explanation proffered in the argument is not the only one that could account for the evidence. Therefore, the PROBABILITY of the original explanation is considerably diminished when you come up with alternative explanations or when you identify other factors that could led to the same outcome.

3. YOU CAN ALSO WEAKEN AN ILLOGICAL ARGUMENT by identifying a REASON WHY THE SPECIFICS OF THE CONCLUSION ARE NOT VALID. If the conclusion is that ‘import quotas on foreign manufactured steel will cause small domestic mills to take business away from large domestic mills, your weakening of the argument should be such that you identify a reason why small domestic mills CANNOT take business away from large domestic mills. This is the equivalent of providing counter-evidence against the conclusion, given the evidence. Given that Lisa made 550 on the GMAT, if counter-evidence exists that another person with 550 has been admitted previously to Harvard Business School, then the conclusion that Lisa will not be admitted to HBS is not well taken.

4. STEP 4: Understand what needs to be done for each question type. The following sections will explain the procedures you need to follow in order to deal with the different types of questions that will appear on your tests.

PROCESS THE CONCLUSION IN THE CONTEXT OF THE EVIDENCE. Remember that the test is about your ability to process information in the context of another statement that constitutes the setting for the information being processed. The evidence provides the setting or the context for the conclusion. Remember that EVIDENCE + CONCLUSION act as ‘terms’ of a binomial expression, and one may not be treated as a separate entity from the other. They need to be processed together. Most test-takers commit the fatal error of taking the conclusion as a stand-alone statement that has nothing to do with the evidence. Watch out for a tendency to do so. DO NOT ATTACK THE EVIDENCE. ATTACK THE INTERPRETATION OF THE EVIDENCE.

Page 55: CRITICAL REASONING 101 T - Amazon S3 · Critical Reasoning Section on the GMAT and GRE will require that you draw conclusions that are logically certain for a set of stated or implied

© Maple Leaf International Consulting, Inc., New York New York. For exclusive use by our registered participants only. Not to be copied or reproduced without our consent.

Page 55

WEAKENING QUESTIONS

A question asking you to WEAKEN an argument asks you: Why do you think that the SPECIFICS OF THE CONCLUSION are not valid, given the evidence. In the argument, “John received a GRE score of 1,000 and, therefore, will not be admitted to M.I.T,” the weakening question must be read: “Why do you figure that John MAY BE ADMITTED TO M.I.T, given than he received a GRE score of 1,000?” Because, M.I.T does not place an emphasis on one factor only, and any problem with the GRE score can be offset by other good factors in John’s favor. Weakening of the argument is done by doing any one of the following: Provide a different explanation for the evidence

used. Provide a different reason for the evidence used; Provide a reason why the conclusion is NOT valid.

In short, you need to identify an answer choice that answers the question: WHY do you figure that the SPECIFICS of the conclusion are NOT valid? (DO NOT WEAKEN AN ARGUMENT BY WEAKENING ITS EVIDENCE) The strategy that you need to follow is argument-specific. If the argument’s conclusion reads like ‘one explanation or one reason’ for the evidence, then you need to look for another explanation or another reason for the evidence. For example, if the argument reads, “Company X signed a new labor contract in 2002, and the company’s productivity increased 20% in the same year. Therefore, the new labor contract led to the improved productivity”, you can see that the conclusion reads like ‘one explanation’ for the evidence: the improved productivity in 2002. Our weakening strategy must be to come up with another explanation for what might have led to the productivity increase: new machinery installed in the same year could be one other explanation. On the other hand, if the conclusion reads like something that one might ‘subjectively’ expect to happen as a result of the stated evidence on the basis of one’s experience or expectation, then our weakening strategy must be to provide a counter evidence stating that the conclusion is not valid.

When you try to weaken an argument, try to deal with the specifics addressed in the conclusion and in the evidence. Let us say that the argument goes: “Heavy fines imposed against companies that cause environmentally damaging accidents will make companies clean up their acts and implement safeguards”. When you try to weaken this argument, look for a reason why the heavy fines will not make companies clean up. A choice that explains why the companies will not act without reference to fines is not as good as the one that explains why the FINES will not make companies clean up their acts and implement safeguards”

Another example of how the ‘specifics’ are critical to the scope of the argument and to your ‘answer picking strategy’ is as follows: Argument: “If import quotas are imposed on steel imports, domestic small mills will take more business away from domestic large mills.” We are required to weaken this argument, and must pay attention to the specifics: small mills taking business away from large ones. Your weakening choice must directly explain why the small mills cannot take business away from large ones. A choice that explains why the large mills will not be affected by the quotas is not good enough or direct enough. Your weakening must explain why the specifics as mentioned will not play out.

Page 56: CRITICAL REASONING 101 T - Amazon S3 · Critical Reasoning Section on the GMAT and GRE will require that you draw conclusions that are logically certain for a set of stated or implied

© Maple Leaf International Consulting, Inc., New York New York. For exclusive use by our registered participants only. Not to be copied or reproduced without our consent.

Page 56

Take a look at the following argument in which the conclusion is what one might expect will happen as a result of the stated evidence: “Lucy received a GMAT score of 500. Therefore, she is not smart.” Notice that the conclusion does not provide one explanation or reason for the evidence. Instead, the conclusion subjectively states what one might expect on the basis of the stated evidence of a GMAT score of 500. Our weakening strategy must be to come up with a counter evidence stating that the conclusion is not valid, and cannot be arrived at on the basis of the stated evidence. Our counter evidence weakening the conclusion is: GMAT is not a smartness test but an aptitude test. If the test does not measure smartness, the conclusion about smartness is not well taken. Take a look at the following arguments in which one or the other of the two strategies has been followed: Example: “Children get attacked by malaria several times before they become immune to malaria. Therefore, a single attack does not confer complete immunity”. If we are required to weaken this argument, we will look for a statement suggesting that a ‘single attack DOES confer complete immunity but there is a good explanation for the evidence, namely the repeated attacks: There are several different strains of malaria and the children need to be exposed to all strains before they become fully immune to malaria as a disease but each attack provides immunity against that specific strain.’ Another example: “Inflation was 1.2% last year but it is 4.0% this year. Therefore, inflation is on an upward swing.” If we are required to weaken this argument, we need to look for a statement stating that “inflation is NOT on an upward swing but there is a good explanation for why it was 1.2% last year but 4.0% this year.” The actual choice might read something like: “Last year a temporary dip in the oil prices brought the inflation below its RECENT STABLE LEVEL OF 4.0%.”

Page 57: CRITICAL REASONING 101 T - Amazon S3 · Critical Reasoning Section on the GMAT and GRE will require that you draw conclusions that are logically certain for a set of stated or implied

© Maple Leaf International Consulting, Inc., New York New York. For exclusive use by our registered participants only. Not to be copied or reproduced without our consent.

Page 57

Consider the argument: “Women have the same chance of winning elections but women represent only 10% of all candidates running for elective offices. Therefore, women do not want to run for elective offices.” If we are required to weaken it, we will look for a statement affirming that WOMEN WANT to run but there is a good explanation for why they are not running: they cannot get the same funding as men can. Notice how we weakened the arguments by providing counterevidence attacking the conclusion or by providing an alternate explanation/reason for the evidence. Also, note that the strategy is argument specific. Let us consider another argument in which we will weaken the argument by explaining the skewed statistics pertaining to “annual rate of violent crime” in states with gun-control laws. Consider the following argument: “The annual rate of violent crime in the states with strict gun control laws is much higher than that in the states without any gun control laws. Therefore, the states with strict gun control laws should get rid of them”. If we are required to weaken this, we should look for counter evidence affirming that it makes sense for the states with strict gun control laws to keep them in place rather than get rid of them. The choice might read: “The annual rate of violent crime has decreased in the states with strict gun control laws after the passage of the strict gun control laws. (Therefore, it does not make sense for them to get rid of them.). Notice that we explained the evidence by stating that the situation might be a lot worse if it were not for the gun-control laws in those states. Arguments about Cause and Effect (Chicken and Egg arguments) These arguments are also referred to as the ‘chicken and egg’ arguments. Which comes first? The chicken or the egg? When you deal with arguments that state a conclusion about cause and effect connection between two events described, the weakening is done by stating that one may be the effect of the other, not the cause as stated in the argument. Or, there may be a third factor causing both events stated in the argument to happen at the same time and create an appearance of connection.

Page 58: CRITICAL REASONING 101 T - Amazon S3 · Critical Reasoning Section on the GMAT and GRE will require that you draw conclusions that are logically certain for a set of stated or implied

© Maple Leaf International Consulting, Inc., New York New York. For exclusive use by our registered participants only. Not to be copied or reproduced without our consent.

Page 58

Consider the following argument: “Married people adopting different sleep-wake cycles have often violent arguments and stressful situations. Therefore, different sleep-wake cycles jeopardize a marriage.” (This argument is of the type: “X happens. Y happens at the same time. Therefore, X causes Y.”) We notice that the “different sleep-wake cycles” are stated as the cause of the “jeopardized marriage” in which violent arguments are the norm. If we are required to weaken it, we need to look for one of the following two statements: ♦ “different sleep-wake cycles are NOT the CAUSE of a bad

marriage but the RESULT of it. ♦ Financial Stress causes married couples to adopt different

sleep-wake cycles and have violent arguments in their relationships (thereby causing both to happen at the same time.)

The first statement says that X is not the cause of Y but X is caused by Y. The second statement says that a third factor Z causes both X and Y to happen at the same time while creating an apparent but misleading connection between them. Take a look at another ‘chicken and egg’ type argument: “A recent study of companies showed that employees who occupy corner offices, which provide a good view and are larger, are more productive than are those who work from cubicles. Therefore, a good office environment improves productivity” Do we know whether a good office environment leads to better productivity or whether one needs to show higher productivity in order to get the better office? If the argument’s conclusion states one story, our weakening will suggest the other possible scenario. ‘Chicken and egg’ arguments are routinely tested on the GMAT. You should be able to recognize the form of the argument before deciding on the appropriate strategy for dealing with the argument. PRICE IS A FUNCTION OF SUPPLY AND DEMAND If you come across an argument in which “price of something” is the subject and the conclusion is about the demand or supply, find a weakening statement that states “higher demand” as the reason for the “higher price” is the conclusion attributes the “higher price” to “reduced supply”, or vice versa. Consider the following example: “The price of cocaine in the open market has been falling steadily this past few months. Therefore, the Government’s enforcement measures are not working.” Notice that the conclusion alludes to an “increased supply” due to

Page 59: CRITICAL REASONING 101 T - Amazon S3 · Critical Reasoning Section on the GMAT and GRE will require that you draw conclusions that are logically certain for a set of stated or implied

© Maple Leaf International Consulting, Inc., New York New York. For exclusive use by our registered participants only. Not to be copied or reproduced without our consent.

Page 59

faltering enforcement efforts by the government. If you are required to weaken the argument, you will look for a statement suggesting that the demand for cocaine has dropped significantly in view of a competing alternative product available at a cheaper price. Notice that this statement provides an alternative explanation for why the price of cocaine has dropped significantly. The argument attributes the price drop to increased supply, but we weaken it by stating that there may be another explanation for the price drop. Remember: Any argument that talks about price has some explanation in the form of answer choice about supply or demand. BROAD GENERALIZATIONS: If the argument uses an example to reach a conclusion about a broad category of which the example is a part, then look for a weakening statement suggesting that the example used is not typical of the whole. The same approach will be followed if a decision at a macro level is reached on the basis of a micro level trial. Consider the following: “People do not get much play out of the sports equipment they purchase. Consider the example of jogging shoes: only 1/3rd of all those who purchase jogging shoes say that they jog regularly.” Notice that the argument states the conclusion right at the outset followed by the evidence used to reach the conclusion. Also notice that the information about jogging shoes is used to reach a broad-based generalization about all sporting equipment. If we are required to weaken the argument, we need to look for a statement suggesting that the “jogging shoes” are not a good example to reach the stated conclusion about all “sports equipment”. The choice might read: “Jogging shoes are used by the owners in activities other than jogging”. This statement tells us that even though only 33% of the owners of jogging shoes may jog regularly, more people are using them in activities other than jogging. (Therefore, the conclusion about the sports equipment use is not valid.)” Consider another example: “Company X asked volunteer employees to work from home for a few months. During this period, the productivity of these volunteer employees was the same as or higher than before. Therefore, company X should implement the “work at home” plan on a company wide basis”. If we are required to weaken this conclusion, we should look for something suggesting that the volunteer employees are not typical of the rest of the employees. (Therefore, the decision to implement the plan on a company wide basis is not valid.)

Page 60: CRITICAL REASONING 101 T - Amazon S3 · Critical Reasoning Section on the GMAT and GRE will require that you draw conclusions that are logically certain for a set of stated or implied

© Maple Leaf International Consulting, Inc., New York New York. For exclusive use by our registered participants only. Not to be copied or reproduced without our consent.

Page 60

“EXCEPT QUESTIONS” in the context of ‘WEAKENING’ of the argument. Occasionally, you will see questions along the lines of: “Each of the following, if true, will weaken the argument, EXCEPT” The question stem tells us that there are FOUR choices that will HAVE THE POTENTIAL to WEAKEN the conclusion stated in the argument, and there is ONE choice that will either STRENGTHEN the conclusion or BE TOTALLY IRRELEVANT TO THE ARGUMENT. Consider the following argument: “A greater number of newspapers is sold in Town S than in town T. Therefore, the citizens of Town S are better informed about major world events than are the citizens of town T.” Each of the following, if true, weakens the conclusion above EXCEPT:

We will read the question to mean that there are FOUR answer choices that HAVE THE POTENTIAL TO SUGGEST THAT THE RESIDENTS OF TOWN S MAY NOT BE BETTER INFORMED ABOUT THE WORLD AFFAIRS, and ONE choice that will either support the stated conclusion or be totally irrelevant to the argument. (A) Town S has a larger population than Town T. (B) Most citizens of Town T work in Town S and buy their newspapers there. (C) The average citizen of Town S spends less time reading newspapers than does the average citizen of town T. (D) A weekly newspaper restricted to the coverage of local events is published in Town S. (E) The average newsstand price of newspapers sold in Town S is lower than the average price of newspapers sold in Town T. We notice that the argument compares sales of newspapers in two towns in order to reach the conclusion it does. Choice A provides an alternative reason for why the sales may be higher in Town S than in Town T. Any alternative reason for the evidence will weaken the argument. ♦ Choice B also provides an alternative reason for why the sales of

newspapers may be more in Town S than in Town T. If the residents of Town T buy the newspapers in Town S, then obviously higher newspaper sales cannot be the basis for a logical conclusion about awareness of world events.

♦ IF the town S residents spend less time reading the newspaper, then

obviously, they are not LIKELY to know more about the world events. Therefore, Choice C weakens the argument by stating a reason for why the conclusion is not valid.

♦ If the newspaper sold in Town S does not cover international stories,

then the residents who buy the newspaper are not going to know more about world events. Choice D provides a reason for why the stated conclusion in the argument is not valid.

Page 61: CRITICAL REASONING 101 T - Amazon S3 · Critical Reasoning Section on the GMAT and GRE will require that you draw conclusions that are logically certain for a set of stated or implied

© Maple Leaf International Consulting, Inc., New York New York. For exclusive use by our registered participants only. Not to be copied or reproduced without our consent.

Page 61

♦ If the first Four have the potential to weaken the conclusion, then the

last one cannot do the same, because we are required to pick a choice on the test. E does not do anything to the argument because price of newspaper is irrelevant to the scheme of things described. IF the choice E were to read that the Citizens in Town S bought the newspaper because it was the cheapest thing to do, then the residents of Town S bought the newspaper for the wrong reasons and may not be expected to know what is going on in the world. Because this choice does not state that Price was the driving factor in the buy decision, this choice is irrelevant to the argument, and must be picked.

Remember: When you deal with EXCEPT questions, read the question to mean that there are FOUR choices that will have the POTENTIAL to weaken the question, and there is ONE choice that will do the opposite or be totally irrelevant. Let us take a look at another example “Advocates of a large-scale space-defense research project conclude that it will present a net benefit to civilian business. They say that since government-sponsored research will have civilian applications, civilian

businesses will reap the rewards of government-developed technology.”

Each of the following, if true, raises a consideration arguing against the conclusion above, EXCEPT:

We will understand the question to mean that there are FOUR choices that will suggest that the Civilian Businesses will NOT benefit from the space-defense research project, and there is ONE choice that will do the opposite or be totally irrelevant.

The development of cost-efficient manufacturing techniques is of the highest priority for civilian business and would be neglected if resources go to military projects, which do not emphasize cost efficiency.

Choice A states that the interests of Civilian businesses will be neglected. Not a benefit to the Civilian businesses. This choice actually weakens the conclusion, not just have the potential to do so. Scientific and engineering talent needed by civilian business will be

absorbed by the large-scale project. This choice says that civilian businesses will hurt. Weakens the conclusion

big time. Many civilian businesses will receive subcontracts to provide materials

and products needed by the research project. This one says that civilian businesses will actually benefit from the space

defense research project. This information does not weaken the conclusion but actually strengthens it. This is the odd-person-out choice we should pick. If government research money is devoted to the space project, it will not be

available for specifically targeted needs of civilian business, where it could be more efficiently used. Says that civilian businesses will hurt, thereby weakening the conclusion.

The increase in taxes or government debt needed to finance the project will severely reduce the vitality of the civilian economy. Says that civilian businesses will hurt, thereby weakening the conclusion.

C is the answer that does NOT weaken the conclusion, and must be picked as the answer choice.

Weakening Questions

We saw at the beginning of this discussion that a strategy for weakening an argument could involve providing a reason for why the conclusion is not valid. Consider the following Example: “Legislature should not decide public policies on the basis of public referenda. This is because special interest groups opposed to the issue can influence public opinion through a series of television ads, and cause the outcome to be biased.” If we are required to weaken the argument, we will do so by stating the opposite of the conclusion and by offering a reason for why the conclusion is not valid. The reason why the conclusion is not valid could be: The outcome of public referenda will not be biased because the group opposed to the policies of the special interest group can run similar television ads and counter the bias.

Page 62: CRITICAL REASONING 101 T - Amazon S3 · Critical Reasoning Section on the GMAT and GRE will require that you draw conclusions that are logically certain for a set of stated or implied

© Maple Leaf International Consulting, Inc., New York New York. For exclusive use by our registered participants only. Not to be copied or reproduced without our consent.

Page 62

NOT WEAKEN questions Sometimes, you will come across questions asking you to identify an answer that will NOT WEAKEN the argument. Any answer that will NOT WEAKEN the argument may either strengthen the argument or BE IRRELEVANT TO THE ARGUMENT. Consider the following argument and the question:

The function of government is to satisfy the genuine wants of the masses, and government cannot satisfy those wants unless it is informed about what those wants are. Freedom of speech ensures that such information will reach the ears of government officials. Therefore, freedom of speech is indispensable for a healthy state. Which one of the following, if true, would NOT undermine the conclusion of the argument? (A) People most often do not know what they genuinely want. (B) Freedom of speech tends ultimately to undermine social

order, and social order is a prerequisite for satisfying the wants of the masses.

(C) The proper function of government is not to satisfy wants, but to provide equality of opportunity.

(D) Freedom of speech is not sufficient for satisfying the wants of the masses: social order is necessary as well.

(E) Rulers already know what the people want. The conclusion is that freedom of speech is INDISPENSABLE for a healthy society. (You must process strong qualifiers, verbs, etc. Indispensable means that which we cannot do without.) Why do they say that freedom of speech is indispensable? (You need to process the ‘why’ for the statement as well). Because freedom of speech will ENSURE (make sure) that the information about what people want will reach the ears of the government. We are required to identify an answer option that will NOT undermine that freedom of speech is indispensable for a healthy society. You can see that Choices A, B, C, and E all UNDERMINE the conclusion that freedom of speech is indispensable so that the government can hear what people want and then satisfy them. Choice A weakens the argument by stating that if people do not know what they want, how can the government proceed to satisfy the genuine wants of masses? Choice B weakens the argument by stating the freedom of speech can actually undermine the social order and cause harm (so that it is not indispensable). Choice C weakens the premise of the argument by stating that the role of the government is not to satisfy the wants but to provide equality of opportunity. Choice E weakens the argument by stating that if the rulers already know what masses want, why would they want to hear them again? Choice D does not weaken the argument. It simply states that freedom of speech is NOT enough. It does not mean that it is not required. WE must go with option D.

Page 63: CRITICAL REASONING 101 T - Amazon S3 · Critical Reasoning Section on the GMAT and GRE will require that you draw conclusions that are logically certain for a set of stated or implied

© Maple Leaf International Consulting, Inc., New York New York. For exclusive use by our registered participants only. Not to be copied or reproduced without our consent.

Page 63

Let us recap our understanding of what needs to be done when we are confronted with a WEAKENING QUESTION: READ THE WEAKENING QUESTION AS: WHY DO YOU THINK

THAT THE SPECIFICS OF THE CONCLUSION ARE NOT VALID, GIVEN THE EVIDENCE? Your answer must be a BECAUSE statement.

♦ Pick a choice that will directly suggest that the conclusion cannot be drawn as specified for the same evidence. If the evidence is that John Scored 1100 on the GRE and the conclusion that he will not be admitted to UPENN Graduate School, and if it turns out that UPENN has admitted people with the same GRE scores in the past and on the basis of a consideration of other factors, then the conclusion that is suggested in the argument becomes untenable.

♦ Weakening of an argument can also be done by providing an alternate explanation for the evidence or identifying an alternate factor that could have contributed to the evidence. Remember that the conclusion of an illogical argument is a PROBABLE statement. In the absence of any other explanation, the only explanation that is offered in the argument for the evidence has 100% chance of being true. If you can identify 4 other explanations for the evidence, then the probability of the original explanation drops to 1/5. Even if you can identify 1 more explanation for the evidence, then the probability of the original explanation drops from 1 to ½. Remember that Weakening of an argument is, for the most part, decreasing the probability of the original explanation for the evidence by offering or identifying other explanations that are equally probable, if not more probable so that we do not know which is the true explanation for the evidence.

♦ When you deal with “EXCEPT” questions, know that there are FOUR choices having the POTENTIAL to weaken the conclusion, and there is ONE that has the POTENTIAL either to strengthen the conclusion or to be totally irrelevant to the argument.

♦ EXCEPT questions can also be tested as ‘NOT UNDERMINE’ questions.

♦ PROCESS THE QUESTION WELL AND REMAIN FOCUSED ON THE QUESTION. Also, be sure to process the conclusion in the context of the evidence.

♦ REMEMBER TO PROCESS ALL INFORMATION IN TERMS OF ANSWERS TO TWO QUESTIONS: WHAT IS BEING SAID, AND WHY IS IT BEING SAID?

Page 64: CRITICAL REASONING 101 T - Amazon S3 · Critical Reasoning Section on the GMAT and GRE will require that you draw conclusions that are logically certain for a set of stated or implied

© Maple Leaf International Consulting, Inc., New York New York. For exclusive use by our registered participants only. Not to be copied or reproduced without our consent.

Page 64

STRENGTHENING QUESTIONS

PROCEDURE FOR STRENGTHENING AN ARGUMENT: Strengthening of an argument is done by providing additional anecdotal evidence that would make the conclusion of an inductive argument MORE PROBABLE for a stated set of premises. A ‘STRENGTHENING’ QUESTION MUST BE READ: “How do you know that the SPECIFICS of the conclusion ARE VALID, given the evidence?” If the conclusion is that ‘the bower building styles of bower birds are culturally acquired and not genetically transmitted’, and if the question asks you to ‘strengthen it’, you should read the question as: “How do you know that the bower building styles of bower birds are culturally acquired and not genetically transmitted?” The answer must appear in the form of anecdotal reason supporting that the bower building style is indeed culturally acquired. The answer could read: “Young bowerbirds are clumsy bower builders and can be seen observing the elders as they build bowers”. Notice that this answer is the answer to ‘how do you know that the bower building styles are culturally acquired?” PROVIDE ADDITIONAL ANECDOTAL INFORMATION or

EVIDENCE LENDING SUPPORT TO THE CONCLUSION AND CONFIRMING IT. Strengthening of an argument si also done by providing additional information in support of information IMPLIED in the conclusion. If the conclusion is that the ‘bower building is a learned skill’, the statement implies that the bower-building is not genetically transmitted. Your strengthening information could be such that the IMPLIED information is supported rather than the explicit information contained in the conclusion.

You should bear in mind that the STRENGTHENING question is very similar to the ASSUMPTION question in that both strengthening information and the assumption support the conclusion stated in the argument. The difference is, the strengthening question answers HOW do you know that the specifics of conclusion are valid whereas the ASSUMPTION question answers WHY do you think that the conclusion is valid?

Page 65: CRITICAL REASONING 101 T - Amazon S3 · Critical Reasoning Section on the GMAT and GRE will require that you draw conclusions that are logically certain for a set of stated or implied

© Maple Leaf International Consulting, Inc., New York New York. For exclusive use by our registered participants only. Not to be copied or reproduced without our consent.

Page 65

While an ASSUMPTION provides a reason why the conclusion is valid given the evidence, the SUPPORTING information will be in the form of an additional information/evidence confirming the conclusion. A strengthening information can also confirm that which is implied in the conclusion. A strengthening response typically reads like an ‘anecdotal’ information, and will answer the question: How do we know that the assumption is valid? Example: “Male bower-birds build elaborately decorated nests in styles that vary from one location to the other. Therefore, the bower building skill is culturally acquired rather than genetically transmitted.” If we are required to STRENGTHEN this argument, we should look for an additional piece of information confirming that the bower building skills on the part of ‘male bower birds’ is indeed learned from the environment. The choice might read something like: “Male bowerbirds, when born, are clumsy bower builders and become proficient by watching the elders build bowers over time.” Notice that this statement confirms the conclusion by stating that the bower building skills are acquired from the cultural interactions and not genetically. Also, the assumption in this argument is that ‘male bowerbirds are unable to build the bowers of the same style and type as soon as they are born. (this is what is implied in the statement ‘not genetically transmitted’). The question is: How do we know that this assumption is valid? Our strengthening response will explain why. If the argument states that the “selection-of- mate behavior is a learned behavior, not genetically programmed”, we need to look for anecdotal information confirming that indeed the selection of mate is a learned behavior, not genetically transmitted. If the conclusion is that ‘human consumption of bacterially infected meat leads to resistant bacteria, given that routine use of antibiotics produce resistant bacteria’, our strengthening of this argument is going to be done by providing evidence that the human consumption of bacterially-infected meat does indeed produce resistant bacteria (because animals are ROUTINELY fed food containing antibiotics to protect them from illnesses).

Page 66: CRITICAL REASONING 101 T - Amazon S3 · Critical Reasoning Section on the GMAT and GRE will require that you draw conclusions that are logically certain for a set of stated or implied

© Maple Leaf International Consulting, Inc., New York New York. For exclusive use by our registered participants only. Not to be copied or reproduced without our consent.

Page 66

If the conclusion that an LSAT Score of 140 will not be good enough to go to Mercer Law School, our strengthening information must make this conclusion more probable by asserting that Mercer does require LSAT scores of better than 140 for admission purposes. THOUGHT REVERSER QUESTIONS USING ‘EXCEPT’ OR ‘NOT’ Strengthening Questions can also be asked with the “thought reverser” EXCEPT. OR ‘WHICH OF THE FOLLOWING WILL NOT STRENGTHEN THE ARGUMENT’? Let us consider the following argument. “State researchers have found that since the oil price increases of the 1970’s, there has been a decline in home energy consumption. They concluded that almost all of the decline has been achieved through reduced standards of living and changes in the way people spend their time.”

Each of the following, if true, would support the conclusion above EXCEPT:

There will be FOUR answer choices lending support to the conclusion that the decline in energy consumption was accompanied by reduced standard of living, and ONE answer choice that will either suggest that the standard of living was not affected at all or provide a response that is totally irrelevant.

Sales of portable heaters rose as families concentrated their winter activities in a limited number of rooms.

this choice confirms that there was indeed a reduced standard of living. This strengthens the conclusion. During the winter months, more people frequented public places such as

libraries and community centers, and on the average, spent considerably longer periods in them than they had done previously.

IF people spend more time in pubic places so that they can stay warm, there is a definite lowering of standard of living. More than 39 percent of households were able to decrease energy costs

substantially by having relatively inexpensive work done to improve the efficiency of their existing heating systems.

This answer choice provides another explanation for why energy consumption declined. This explanation does not suggest that the standard of living was compromised. Therefore, this choice does not strengthen the conclusion and is the one we should pick. At least 59% of the households maintained a lower indoor temperature

than they had been accustomed to maintaining on very cold days. More households maintained colder indoor temperatures. Points to a

reduced standard of living supporting the conclusion. Members of at least 60% of households showered for shorter periods of

time than they had done previously. If people are not bathing properly, there must definitely be a reduced

quality of life. This choice strengthens the conclusion by providing additional supporting information.

As we can see, choice C is the one that does not strengthen the

Page 67: CRITICAL REASONING 101 T - Amazon S3 · Critical Reasoning Section on the GMAT and GRE will require that you draw conclusions that are logically certain for a set of stated or implied

© Maple Leaf International Consulting, Inc., New York New York. For exclusive use by our registered participants only. Not to be copied or reproduced without our consent.

Page 67

argument’s conclusion by providing additional information in support of the conclusion. In fact, it does the opposite: It provides an alternative explanation for why the energy costs went down, thereby weakening the conclusion. Let us recap the rule for strengthening an argument: ANY QUESTION ASKING YOU TO IDENTIFY

STRENGTHENING INFORMATION MUST BE READ: “HOW DO YOU KNOW THAT THE SPECIFICS OF THE CONCLUSION ARE VALID, GIVEN THE EVIDENCE?”

Look for an additional piece of ANECDOTAL information

lending support to the conclusion of the argument consistent with the scope of the argument.

Remember: A strengthening response explains ‘how do

you know that that assumption is valid?’ and will provide anecdotal information confirming the assumption and validating the conclusion.

ASSUMPTION REDUX Every illogical argument involves an assumption in support of the conclusion. Technically, an assumption is a QUESTIONABLE INFERENCE that can be drawn between the evidence and the conclusion or from the conclusion itself. You can think of an assumption as a REASON why the conclusion is valid, given the evidence. Consider the following argument: “Prosperity, hitherto a matter of geographical location, will be henceforth a matter of a country’s ability to adopt emerging new information technologies. But the poor countries of the world lack the resources to acquire these new information technologies, and will, therefore, remain poor. As a result of this, the new information technologies will widen the economic gap between the rich and the poor countries.” If the poor countries lack the resources to acquire the new information technologies and will remain poor, in order for the economic gap between the rich and the poor countries to widen, the rich countries must get richer. Given that the prosperity is a matter of adopting the new information technologies, the

Page 68: CRITICAL REASONING 101 T - Amazon S3 · Critical Reasoning Section on the GMAT and GRE will require that you draw conclusions that are logically certain for a set of stated or implied

© Maple Leaf International Consulting, Inc., New York New York. For exclusive use by our registered participants only. Not to be copied or reproduced without our consent.

Page 68

argument assumes that at least some of the rich countries will spend their resources to acquire the new information technologies and get richer. This is what is implied between the evidence and the conclusion. The conclusion alone supports the inference that the rich countries will get richer or the rich countries will remain where they are while the poor countries get poorer. Viewed in the context of the evidence, the conclusion allows us to draw the inference as stated above. Any inference that is supported by a single statement such as the conclusion is a necessarily true condition that must exist in order for the conclusion to be valid. If the conclusion is that ‘Mary will not be admitted to Harvard Business School’, and if it is true that one must apply in order to be admitted, it necessarily follows that the conclusion implies that Mary will apply or has applied for admission to HBS. Consider another argument: “Cars made by Yujitsu motor company are sold with available antilock brakes that prevent skidding on slippery surfaces. Yuichi bought the car made by Yujitsu. Therefore, Yuichi need not worry about skidding on slippery roads.” In order for the car to not skid on slippery roads, it must have the antilock brakes that prevent skidding on slippery surfaces.. Antilock brakes are AVAILABLE on the Yujitsu cars. But then, Yuichi could have bought a car without this feature included in the car. Available implies ‘available to be purchased’ as an option. In order for the conclusion to be true, it is necessary that Yuichi bought the car with antilock brakes included. Therefore, the assumption is that Yuichi bought a Yujitsu automobile with antilock brakes. Notice that the assumption was identified as implied information between the evidence and the conclusion and through the interpretation of qualifiers provided in the argument. We will discuss in the following pages the procedure for determining the assumption underlying the argument in greater detail. For now, remember that the assumption is information that is

• Implied between the evidence and the conclusion, and required to support the specifics of the conclusion; or

• Implied in the conclusion alone and is required as a precondition in order for the conclusion to be true. We have seen that one must apply to a Graduate School in order to be considered for admission. Therefore, in order to support the conclusion that Mary will not be admitted to Harvard Business School, it must be true that she intends to apply to or has already applied for admission to HBS.

Page 69: CRITICAL REASONING 101 T - Amazon S3 · Critical Reasoning Section on the GMAT and GRE will require that you draw conclusions that are logically certain for a set of stated or implied

© Maple Leaf International Consulting, Inc., New York New York. For exclusive use by our registered participants only. Not to be copied or reproduced without our consent.

Page 69

ASSUMPTION QUESTIONS We saw how an illogical argument is NOT necessarily valid UNLESS the assumption or the assumptions is or are valid. The argument that ‘Feds raised interest rates, and , therefore, the stock markets will take a hit’ is a non sequitur because the conclusion ‘the stock markets will take a hit’ is not necessarily true UNLESS it is true that raising of interest rates by the Feds necessarily leads to decline in stock market valuations. The ‘unless statement’ is the assumption.

The ASSUMPTION will be an answer to the question: “WHY DO you figure that the VERBATIM conclusion stated in the argument is valid, given the evidence?” Do the BECAUSE TEST to confirm that the assumption that you want to identify is relevant to the specifics of the conclusion. ” The ‘verbatim conclusion is valid BECAUSE the (assumption).’ Your answer choice must replace (assumption) in the above test. The role of an assumption is to provide a strong reason for the stated conclusion in keeping with the evidentiary information, and to provide a logical quality to the whole argument by acting as a link between the elements of the Evidence and those of the Conclusion. If the argument is read with the assumption, then there will be a logical progression to the conclusion that is stated in the argument. Also, the assumption identifies an “implied and Critical information or activity” vital to supporting the conclusion. The test you will apply is as follows: “Conclusion because assumption” must make sense, and the assumption must provide a compelling reason for the conclusion in line with the basis for the argument. Consider the argument: “Tom received a GMAT score of 400 out of 800. Therefore, Tom will not be admitted to Harvard Business School.” The conclusion is that Tom will not be admitted to HBS, and the basis for the conclusion is his GMAT score. Let us consider two options for the assumption: A GMAT score of 400 is not sufficient for admission to HBS. Tom is dumb.

Let us read the conclusion along with each of the two assumptions and see how the test reads: Tom will not be admitted to HBS because his GMAT score of

400 is not sufficient for admission to the HBS. Tom will not be admitted to HBS because Tom is dumb.

Notice that both statements provide a reason for why Tom will not be admitted to HBS, but we must bear in mind that ‘dumbness’ is not the basis for the conclusion; GMAT score of 400 is, and for this reason, we must pick the first choice.

REMEMBER to pay attention to the SPECIFICS of the conclusion, and to take the conclusion verbatim. You must do this for any weakening or strengthening or assumption question. Do NOT interpret the conclusion and make it your own. Remember: you are required to deal with someone else’s conclusion, not interpret someone else’s conclusion, make it your own, and then attack YOUR conclusion. Do not make this fatal mistake that an average test taker commits. Your mission is to score well on the test by following the proper procedures, not go on a martyrdom mission.

Page 70: CRITICAL REASONING 101 T - Amazon S3 · Critical Reasoning Section on the GMAT and GRE will require that you draw conclusions that are logically certain for a set of stated or implied

© Maple Leaf International Consulting, Inc., New York New York. For exclusive use by our registered participants only. Not to be copied or reproduced without our consent.

Page 70

Remember that assumption is a reason that is consistent with the evidence and required to support the conclusion. You must make sure that the reason for why the conclusion is valid must be consistent with the basis and the scope of the argument. Consider another argument: “Dallas downtown has 40% of the office space vacant right now. But these vacant spaces are not readily suited to the requirements of the Federal government. Therefore, the Feds are not wasting money by building new buildings for their own use in downtown Dallas.” Let us take a look at two options and see which one we should select. Adaptation of existing vacant spaces to make them suitable

for Federal government use is not cost effective. The Feds have an obligation to provide stimulus to the

economy by engaging in building contracts. Let us read the conclusion of the argument along with each of the two choices we have shortlisted: The Feds are not wasting money by building new buildings

for their own use in downtown Dallas because adaptation of existing vacant spaces to make them suitable for Fed use is not cost effective.

The Feds are not wasting money by building new buildings for their own use in downtown Dallas because the Feds have an obligation to provide stimulus to the economy by engaging in building activities.

Notice that the first choice is the one we should pick because it encompasses the essence or the basis of the argument, namely, existing high vacancy rates. The second option, even though it provides a reason for why the conclusion may be valid, is not relevant to the scope of the argument. “Providing economic stimuli’ is not relevant to the scope of the argument.

Page 71: CRITICAL REASONING 101 T - Amazon S3 · Critical Reasoning Section on the GMAT and GRE will require that you draw conclusions that are logically certain for a set of stated or implied

© Maple Leaf International Consulting, Inc., New York New York. For exclusive use by our registered participants only. Not to be copied or reproduced without our consent.

Page 71

AN ASSUMPTION PROVIDES A REASON FOR WHY THE CONCLUSION IS VALID AND THE REASON must be IN LINE WITH THE BASIS FOR THE CONCLUSION AS STATED IN THE EVIDENCE. Remember to do the following test before confirming your answer choice: “The VERBATIM conclusion is valid BECAUSE (your assumption answer consistent with the stated basis of the argument).” This statement must make sense and must be consistent with the elements addressed in the explicit basis (evidence) of the argument. Consider another argument: “Top managers used intuition more significantly than did the middle level managers. Therefore, intuition is a more effective decision-making process.” Notice that the basis for the conclusion – “Intuition is a more effective decision-making process” – is the ‘more significant use of intuition’ by ‘top managers’. Our assumption must provide a strong reason for why the conclusion is valid but be in line with the basis for the conclusion. Consider the following two short-listed options: Intuition is the natural decision-making process. Top managers who significantly use intuition make effective

decisions. Let read the conclusion along with each of the short listed options. Intuition is a more effective decision making process

because intuition is the natural decision-making process. Intuition is a more effective decision making process

because top managers who significantly use intuition make effective decisions.

Notice that the first choice is too vague and unrelated to the basis for the evidence. Whether intuition is a natural decision making process or is unnatural process is not the issue. The second option is the best one because the reason provided in support of the conclusion is consistent with the basis for such conclusion. Let us take a look at another argument and see how we can identify the reason in support of the conclusion – also called the assumption – but without straying from the basis for the conclusion or the scope of the argument.

Page 72: CRITICAL REASONING 101 T - Amazon S3 · Critical Reasoning Section on the GMAT and GRE will require that you draw conclusions that are logically certain for a set of stated or implied

© Maple Leaf International Consulting, Inc., New York New York. For exclusive use by our registered participants only. Not to be copied or reproduced without our consent.

Page 72

Argument: “Interviewers, during an interview, can eliminate candidates who are not suited to the requirements of the job. Therefore, interview is an essential step in the hiring process.” The conclusion is that the ‘interview is an essential step in the hiring process’, and the basis for the conclusion is the expectation that the interviewers can eliminate candidates who are not suitable. Our assumption must provide a strong reason in support of the conclusion – the interview is an essential step in the hiring process – and be consistent with the basis for such conclusion. Let us take a look at the following three short-listed options: Job applicants tend to lie on their resumes. Interviewers can tell during the interview which applicant is

suited to the requirements of the job and which ones are not. Interviewers have the ability to eliminate candidates who are

unsuitable. We can safely eliminate the first choice because it does not address the basis for the conclusion – interview and the interviewers eliminating unsuited applicants. Let us read the conclusion along with the second and the third options using ‘because’. Interview is an essential step in the hiring process because

interviewers can determine during the interview which applicant is suited to the requirements of the job and which ones are not.

Interview is an essential step in the hiring process because interviewers have the ability to eliminate candidates who are unsuitable.

Obviously, the first is the best option because the second one talks about the interviewers having the ability to eliminate unsuitable candidates. The first one implies this ability and goes further by stating that they will use this ability to determine who is not suitable. Remember to pick a more COMPLETE answer by pausing and critically examining the wording of answer options.

Page 73: CRITICAL REASONING 101 T - Amazon S3 · Critical Reasoning Section on the GMAT and GRE will require that you draw conclusions that are logically certain for a set of stated or implied

© Maple Leaf International Consulting, Inc., New York New York. For exclusive use by our registered participants only. Not to be copied or reproduced without our consent.

Page 73

Let us take a look at another argument and try to identify the assumption. ARGUMENT: “A proposed change to federal income tax laws would eliminate deductions from taxable income for donations a taxpayer has made to charitable and educational institutions. If this change were adopted, wealthy individuals would no longer be permitted to make such deductions. Therefore, many charitable and educational institutions would have to reduce services, and some would have to close their doors.” The argument above assumes which of the following? ♦ many charitable and educational institutions would have to reduce

services, and some would have to close their doors because Without the incentives offered by federal tax laws, at least some wealthy individuals would not donate as much money to charitable and educational institutions as they otherwise would have.

♦ many charitable and educational institutions would have to reduce services, and some would have to close their doors because Money contributed by individuals who make their donations because of provision in the federal tax laws is the only source of funding for many charitable and educational institutions. (Notice that this choice does not talk about wealthy individuals)

♦ many charitable and educational institutions would have to reduce services, and some would have to close their doors because The primary reason for not adopting the proposed change in the federal income tax laws is to protect wealthy individuals from having to pay higher taxes.

♦ many charitable and educational institutions would have to reduce services, and some would have to close their doors because Wealthy individuals who donate money to charitable and educational institutions are the only individuals who donate money to such institutions.

♦ many charitable and educational institutions would have to reduce services, and some would have to close their doors because Income tax laws should be changed to make donations to charitable and educational institutions the only permissible deductions from taxable income.

The conclusion is that many charitable and educational institutions would have to reduce services, and some would have to close their doors, and the basis for the conclusion is the changes to the income tax laws disallowing deductions for charitable donations and the inability of wealthy individuals to make such a claim. You can see that if you read the conclusion along with each of the choices using ‘because’, choice A is the best answer consistent with the scope of the argument and the basis for the conclusion. Choice B provides an extreme reason by using the qualifier ‘only’ and does not use the adjective ‘wealthy’ to describe the ‘individuals’. Choice C is not good because it talks about ‘primary reason’ – a phrase on our HIT LIST summarized on page 6. Also, ‘protecting wealthy individuals’ is not relevant to the scope of the argument. Choices D and E both use the extreme qualifier ‘only’ and contain other wrinkles – Choice D does not talk about the changes to the tax laws and choice E talks about permissible deductions.

Page 74: CRITICAL REASONING 101 T - Amazon S3 · Critical Reasoning Section on the GMAT and GRE will require that you draw conclusions that are logically certain for a set of stated or implied

© Maple Leaf International Consulting, Inc., New York New York. For exclusive use by our registered participants only. Not to be copied or reproduced without our consent.

Page 74

STANDARD ASSUMPTIONS Our earlier discussion of the various types of arguments highlighted the importance of getting a grip on the type of argument so that you can determine the relevant assumption. For example, if you are dealing with a ‘cause and effect’ argument that reads “X causes Y”, the assumption is that “X could not have been caused by Y” or “a third factor such as Z could not have caused both X and Y”. If the argument reads that the ‘mismatched sleeping waking cycles between the two spouses in a marital relationship jeopardizes a marriage”, the argument assumes that ‘the mismatched sleeping waking cycle is not the direct result of an already jeopardized marriage’ or that ‘financial stress did not cause couples to adopt mismatched sleeping waking cycles and also jeopardize the marriage.” Here is a brief discussion of how you can spot the assumptions in the different types of arguments. ♦ If the argument’s conclusion appears to be just one

explanation for the evidence, then the standard assumption is that there are no other explanations for the evidence.

Example: “There is a tremendous geographical variation in the frequency of many surgical procedures across the two counties: County A and County B. In County A, the number of surgical procedures carried out is three times the number carried out in County B. Therefore, many surgical procedures are carried out unnecessarily in County A.” Notice that the conclusion identifies ONE of several possible explanations for the variation. The critical factor supporting the conclusion must rule out the possibility of variation arising due to other factors or explanations. Our assumption must read: “There are no other factors or explanations for the geographical variation.” Notice that the above statement reads too broad. The GMAT assumption statement might identify one other factor that may have contributed to the variation and rule that out as a possible explanation. For example, the assumption statement might read:

Page 75: CRITICAL REASONING 101 T - Amazon S3 · Critical Reasoning Section on the GMAT and GRE will require that you draw conclusions that are logically certain for a set of stated or implied

© Maple Leaf International Consulting, Inc., New York New York. For exclusive use by our registered participants only. Not to be copied or reproduced without our consent.

Page 75

“The variation is not because of greater population in County A.” Notice that “greater population in County A” may be another explanation for why the variation described happens. ♦ If the argument recommends as its conclusion a course of

action because of a single benefit identified, then the assumption is always that the “single benefit identified will more than offset the potential disadvantages of following the course of action.”

Consider the following example: “In any serious policy discussion about acceptable levels of risk in association with EXPLOSION, the term “explosion” elicits the desired level of attention whereas the substitute phrase “energetic disassembly” does not. Therefore, of the two terms, “explosion” should be the one used in the context of such policy discussions.” Notice that the conclusion advocates using the term “explosion” on the basis of a single identified benefit, namely, “the desired level of attention”. What is critical to the conclusion is a statement that the solitary benefit identified will more than compensate for the drawbacks of using the term. Our assumption will read: “The benefit of ‘desired level of attention’ will more than compensate for the drawbacks, if any, associated with using the term explosion.” ARGUMENTS PRESENTING COMPETING OPTIONS: ♦ If the argument justifies a course of action in its conclusion

on the basis of an evidence presented, the assumption is that alternative courses of action are not worth pursuing.

Consider the following argument: “The vacancy rates in the existing office towers in downtown Dallas are approaching 50%. However, these vacant spaces do not readily meet the security requirements of the Federal Government. Therefore, the Federal government is not fiscally irresponsible in deciding to go ahead with new construction of Federal buildings in downtown Dallas.”

Page 76: CRITICAL REASONING 101 T - Amazon S3 · Critical Reasoning Section on the GMAT and GRE will require that you draw conclusions that are logically certain for a set of stated or implied

© Maple Leaf International Consulting, Inc., New York New York. For exclusive use by our registered participants only. Not to be copied or reproduced without our consent.

Page 76

What is critical to the conclusion about “fiscal responsibility” is a statement that the alternative course of action – adaptation of existing vacant spaces – will not be more cost effective than building new buildings meeting the security standards of the Federal buildings. Our assumption will identify this critical factor: “Adaptation of existing vacant spaces in Dallas downtown to meet the security requirements of the Federal buildings will not be more cost effective than building new Federal buildings meeting the security standards.” PROFILING ARGUMENTS: ♦ IF the argument identifies a specific subset of a larger group

and if the subset is a significant part of the larger group, then any characteristic attributed to the subset must be ASSUMED to characterize the larger group as well. This is the stuff “profiling” is made of. If 50% of those who drive a Porsche turn out to be drug smugglers, then the assumption made by the law enforcement is that all Porsche drivers are worth stopping and checking for drug possession.

Consider the following argument: “Although only 3% of all drivers on Maryland highways use radar detectors, 33% of those caught speeding had radar detectors. Therefore, drivers using radar detectors are more likely to exceed speed limits REGULARLY than are those not using radar detectors.” Notice that while the owners of radar detectors represent a tiny proportion of all drivers, they represent a significant proportion of all drivers caught speeding. Therefore, any characteristic attributed to those caught speeding with radar detectors must be ASSUMED to belong to the larger group of “drivers getting caught exceeding speed limits.” Our assumption must read: “Drivers caught speeding on Maryland Highways are more likely to exceed speed limits REGULARLY than those not caught speeding.”

Page 77: CRITICAL REASONING 101 T - Amazon S3 · Critical Reasoning Section on the GMAT and GRE will require that you draw conclusions that are logically certain for a set of stated or implied

© Maple Leaf International Consulting, Inc., New York New York. For exclusive use by our registered participants only. Not to be copied or reproduced without our consent.

Page 77

ASSUMPTION IS A QUESTIONABLE INFERENCE SUPPORTED BY A QUESTIONABLE STATEMENT SUCH AS CONCLUSION OR BETWEEN TWO STATEMENTS ONE OF WHICH IS QUESTIONABLE (SUCH AS BETWEEN EVIDENCE, THE FACT, AND CONCLUSION, THE QUESTIONABLE STATEMENT). INFERENCE RULE: If X happens and in order for Z to happen, Y must happen, then Y is necessarily implied in the statement “X causes Y’. Or, if Z cannot happen unless Y happens, then Y is necessarily implied in the statement ‘Z happened’. Consider the following examples that explain the above inference rule: STATEMENT: John will not be admitted to Columbia Business School. INFERENCE: John will apply or has applied for admission to Columbia (because, unless he applies, he cannot be considered for admission). STATEMENT: The city X is implementing new parking rules that will robustly target illegally parked vehicles. As a result of these rules, the city’s revenue from parking fines will increase. INFERENCE: The City X will ticket more illegally parked vehicles or more of those ticketed will be required to pay the fines and not get away without paying. STATEMENT: 80% of Harvard University’s undergraduate students graduate with Honors degrees. Therefore, to restore faith in Honors qualifications, Harvard must take steps to control grade inflation. INFERENCE: Harvard University’s undergraduate students do not EARN those higher grades (because they receive inflated grades). The argument implies that higher grades are not feasible unless grades are inflated. Another eventuality that could have led to higher grades is that the students are naturally smarter or brighter. The argument discounts the latter scenario in stating that there is grade inflation. THE TEST IS ABOUT YOUR ABILITY TO IDENTIFY THAT WHICH IS IMPLIED IN EXPLICIT STATEMENTS.

Page 78: CRITICAL REASONING 101 T - Amazon S3 · Critical Reasoning Section on the GMAT and GRE will require that you draw conclusions that are logically certain for a set of stated or implied

© Maple Leaf International Consulting, Inc., New York New York. For exclusive use by our registered participants only. Not to be copied or reproduced without our consent.

Page 78

Let us recap what ASSUMPTION underlying an illogical argument is all about. Remember that Assumption is an INFERENCE that is questionable because one of the two statements (conclusion) involved in identifying this inference is questionable. If an inference is supported by two statements that are both factually correct, then the inference is classified as a logical conclusion. If the argument goes: ‘Feds raised interest rates by 50 basis points today. Therefore, the DJIA will sharply decline during the trading tomorrow’, the argument IMPLIES between the two statements that an increase of 50 basis points in the interest rate will negatively affect the stock market index. Notice that the conclusion is not valid UNLESS the identified inference is valid. The argument also assumes that the stock index had not factored in yesterday’s trading a possible 50 basis point increase in the Fed rate so that nothing may happen tomorrow in respect of the DJIA valuation. Because the conclusion in the above argument is a non-sequitur and questionable, any inference that is drawn between the evidence (not questionable) and the conclusion (questionable) itself is questionable inference and becomes the assumption. Assumption is also implied information in a questionable statement such as the conclusion. Consider the conclusion that ‘if internet access is widely available to the citizens of a dictatorial regime, repression cannot continue’, we can draw the inference that the citizens living under the repressive regime will be ALLOWED TO use the AVAILABLE internet access. The conclusion is not true UNLESS this implication of the conclusion is true. Reading comprehension will test your ability to draw inferences on the basis of a single statement, and we will discuss this procedure again in Module V.

Page 79: CRITICAL REASONING 101 T - Amazon S3 · Critical Reasoning Section on the GMAT and GRE will require that you draw conclusions that are logically certain for a set of stated or implied

© Maple Leaf International Consulting, Inc., New York New York. For exclusive use by our registered participants only. Not to be copied or reproduced without our consent.

Page 79

SUMMARY READ THE QUESTION ASKING YOU TO IDENTIFY THE ASSUMPTION AS: WHY DO YOU FIGURE THAT THE SPECIFICS OF THE CONCLUSION ARE VALID, GIVEN THE EVIDENCE? . Think of ASSUMPTION as a DENIAL of a MAYBE scenario. Think of ASSUMPTION as a REASON why the specifics of the conclusion are valid. Think of Assumption is INFORMATION IMPLIED IN A QUESTIONABLE STATEMENT SUCH AS THE CONCLUSION or between the Evidence and the Conclusion. The SPECIFICS of a statement are expressed by elements such as verbs, nouns, qualifiers, etc. PAY ATTENTION TO THE SPECIFICS and do not broadly interpret the conclusion or evidence. Take the statements given to you verbatim. Remember that you are dealing with someone else’s illogical argument, not your own.

LOGICAL CONCLUSION OR INFERENCE LOGICAL CONCLUSION

ANY INFORMATION IMPLIED IN A STATEMENT THAT IS NOT QUESTIONABLE OR BETWEEN STATEMENTS WHICH ARE ALL BEYOND REPROACH CONSTITES INFERENCE. We saw in our earlier discussion that Assumption is also an inference but is supported by a questionable statement. You need to keep this distinction in mind. If the question on the test requires you to identify the LOGICAL CONCLUSION, it is asking you to identify that which is implied in the evidence or in a factual set of statements, not in the conclusion. Conversely, if the question is about assumption, do not select an answer that identifies information implied in the evidence. An INFERENCE simply identifies implied but not openly stated information in the evidence. We can draw an inference between two statements that both unquestionable or within a factual statement . We have also seen that any inference that is supported by a literal interpretation of evidence is logical conclusion whereas any inference that is supported by the conclusion alone or between the evidence and conclusion of an illogical argument is a questionable inference or assumption. Any information implied in the evidence of an illogical argument

Page 80: CRITICAL REASONING 101 T - Amazon S3 · Critical Reasoning Section on the GMAT and GRE will require that you draw conclusions that are logically certain for a set of stated or implied

© Maple Leaf International Consulting, Inc., New York New York. For exclusive use by our registered participants only. Not to be copied or reproduced without our consent.

Page 80

constitutes a logical conclusion. A logical inference essentially Recycles evidence, and makes a paraphrased interpretation

of the evidence presented; Makes a literal interpretation of the qualifiers, verbs, and

nouns used in the evidence. Makes a literal interpretation of information stated in a

mutually exclusive or opposite context. A logical conclusion is about WHAT IS GOING ON, and not about WHY or HOW? Remember this distinction. We also stated that the Reading Comprehension section of all Graduate School admission tests requires you to make LOGICAL CONCLUSION by literally interpreting information stated in a specific context identified in the question stem. Our Module V will further explain to you how to identify the context in which information is stated in the passage and how to make logical conclusion from the evidence presented in the relevant context. For now, let us focus on how this skill may be tested in the GMAT, GRE, and LSAT critical/logical reasoning sections. If the evidence is: “30% of the entering class is women”, the inference is that 70% are men. If the evidence is that “most smokers snore”, then the inference is that “some smokers do not snore.” If it ‘usually’ rains in London, it sometimes does not. ♦ IF the evidence is that the operating costs went up at the

same time as the bottomline profits went up, then the inference is that the third element – Revenues – must have gone up too. You will be required to use the equation: REVENUES – COST = PROFITS.

♦ If the evidence is that the price of a commodity went down, then the inference is that either the supply went up or the demand fell for the commodity. You have to use the relationship that Price is a function of supply and demand.

Remember the following when you deal with Conclusion or Inference questions: Recycle or restate evidence or state that which is implied but not openly expressed. Do not attempt to explain the evidence if the requirement is that you identify an inference. Never pick a choice that forces you to speculate and matches your expectation not supported by the evidentiary information.

Page 81: CRITICAL REASONING 101 T - Amazon S3 · Critical Reasoning Section on the GMAT and GRE will require that you draw conclusions that are logically certain for a set of stated or implied

© Maple Leaf International Consulting, Inc., New York New York. For exclusive use by our registered participants only. Not to be copied or reproduced without our consent.

Page 81

What you see is what you know and recycle the evidence. No more. No less. If a piece of information you are tempted to pick is not supported by evidence openly stated or implied, you cannot pick that choice. If the evidence is that ‘people must take prep classes in order to do well in the GMAT, and Tom did not take a prep class”, our inference cannot explain why Tom might do well on the test even though he did not take a prep class. Our inference must be that Tom will not do well on the test. If the evidence is that ‘30% of the entering class is made up of women”, our inference must be that 70% of the entering class is made up of men, or that there are more men than women in the entering class. We cannot attempt to explain why only 30% of the class is made up of women. For example, ‘women do not prefer business or science courses’ is an attempt at explaining why the percentage is only 30%, and an inference cannot be an explanation of the evidence. HOW TO MAKE THE DISTINCTION BETWEEN THE ASSUMPTION AND THE LOGICAL CONCLUSION? Consider the following argument: “All of the world’s best comedians have had miserably unhappy childhood. Elaine had an extremely unhappy childhood. Therefore, Elaine must be among the best of comedians”. Logical Conclusion (supported by the evidence); Anyone who has had a happy childhood cannot be among the best comedians. This is necessarily implied in the evidence. Questionable implication or Assumption supported by the conclusion or by the LINE OF REASONING: Anyone who had an extremely unhappy childhood cannot be anything but among the best of comedians. (Remember the rule: IF all of X is Y, all of Y is not necessarily X. But Not Y is necessarily not X. Logical conclusion is of the form ‘not Y is not X’ whereas the assumption is of the form “if X is Y, then Y is X”. Notice that the implication in the conclusion is not necessarily true and is questionable. Therefore, it becomes the assumption. Remember: Any inference drawn from a questionable statement is assumption. Any inference drawn from a statement that is not questionable is logical conclusion). Remember that any inference supported by the evidence is not the Assumption of an argument. If the question asks you to identify the assumption of an argument, do not pick an answer implied in the evidence.

Page 82: CRITICAL REASONING 101 T - Amazon S3 · Critical Reasoning Section on the GMAT and GRE will require that you draw conclusions that are logically certain for a set of stated or implied

© Maple Leaf International Consulting, Inc., New York New York. For exclusive use by our registered participants only. Not to be copied or reproduced without our consent.

Page 82

Let us take a look at some passages and hone our ability to

identify the IMPLIED INFORMATION contained in information that is beyond reproach. Consider the following example: “The Rienzi, a passenger ship, sank as a result of a hole in its hull, possibly caused by sabotage. Normally, when a holed ship sinks as rapidly as the Rienzi did, water does not enter the ship quickly enough for the ship to be fully flooded when it reaches the ocean floor. Full flooding can be achieved, however, by sabotage. Any ship that sinks deep into the ocean when not fully flooded will implode. Deep-sea photographs, taken of the sunken Rienzi where it rests on the ocean floor, reveal that the Rienzi did not implode. Which one of the following must be true on the basis of the information above? (Hint: You must read this as a conclusion question.) (A) The Rienzi was so constructed as to reduce the risk of sinking by impact. (B) If the Rienzi became fully flooded, it did so only after it reached the ocean floor. (C) If the Rienzi was not sunk by sabotage, water flooded into it unusually fast. (D) If the Rienzi had sunk more slowly, it would have imploded. (E) The Rienzi was so strongly constructed as to resist imploding under deep-sea pressure. The phrase “possibly caused by sabotage” implies that the ship was not necessarily sunk by sabotage. Also, the fact that a “not fully flooded” ship will implode upon reaching the ocean floor and that Rienzi did not implode tell us that Rienzi was fully flooded when it hit the floor. But because Rienzi sank rapidly and water usually does not enter the ship fast enough to flood it, it follows that Rienzi was an exceptional case. Choice C is the logical inference permissible by the set of circumstances described in the evidence. Choice A is beyond the scope. “sinking by impact” is not relevant to the scope of the argument. Choice B is contrary to the argument’s evidence. If the ship was not fully flooded when it hit the ocean floor, it must implode. Rienzi did not. Therefore, it must have been flooded fully when it hit the ocean floor, not after the fact. Choice D is also contrary to the evidence presented. If the ship had sunk more slowly, it would have been fully flooded and implosion is not possible. Choice E talks about the “construction” of the ship to resist “deep sea pressure”. You can easily tell that the information is beyond the scope of the evidentiary information presented.

Page 83: CRITICAL REASONING 101 T - Amazon S3 · Critical Reasoning Section on the GMAT and GRE will require that you draw conclusions that are logically certain for a set of stated or implied

© Maple Leaf International Consulting, Inc., New York New York. For exclusive use by our registered participants only. Not to be copied or reproduced without our consent.

Page 83

Let us consider another argument asking us to make a logical conclusion, and hone the skill to stay within the scope and to recycle the evidence presented. “Nearly all mail that is correctly addressed arrives at its destination within two business days of being sent. In fact, correctly addressed mail takes longer than this only when it is damaged in transit. Overall, however, most mail arrives three business days or more after being sent.” If the statements above are true, which one of the following must be true? Hint: Read the question as asking you to make a logical conclusion. (A) A large proportion of the mail that is correctly addressed is damaged in transit. (B) No incorrectly addressed mail arrives within two business days of being sent. (C) Most mail that arrives within two business days of being sent is correctly addressed. (D) A large proportion of mail is incorrectly addressed. (E) More mail arrives within two business days of being sent than arrives between two and three business days after being sent. The evidence is that “nearly 100%” of the mail that is correctly addressed should be delivered within TWO business days of mailing. We also know that a correctly addressed mail takes longer than TWO business days only when it is damaged in transit. However, MOST mail is delivered after THREE business days of mailing. Because Most mail is delivered after THREE business days and because if they were correctly addressed they should have been delivered within TWO business days, the logical inference is that MOST mail is incorrectly addressed. Choice D is the best inference supported by the evidence presented to us. Why is Choice A not as good? It sure is tempting. If the evidence were to say that “most mail” is delivered after TWO days of mailing, then A might be a good inference because the argument would have us believe that a correctly addressed but damaged in transit mail is delayed beyond TWO days. The evidence does not tell us that the “correctly addressed but damaged in transit” mail is delayed beyond THREE business days. Also, “nearly all correctly addressed mail” is delivered within TWO business days implies that a small percentage may be damaged in transit and not delivered within two business days. Choice A talks about “a large proportion” of correctly addressed mail being damaged in transit. You can see that this inference is not supported by the evidentiary information. You should pay attention to the qualifiers used in the evidence in order to determine whether a stated inference is well supported or not. Notice the highlighted qualifiers in the argument. Nearly all arguments asking you to make an inference will ask you to pay attention to any qualifiers used. Choice B is not good because we do not know what happens to the incorrectly

Page 84: CRITICAL REASONING 101 T - Amazon S3 · Critical Reasoning Section on the GMAT and GRE will require that you draw conclusions that are logically certain for a set of stated or implied

© Maple Leaf International Consulting, Inc., New York New York. For exclusive use by our registered participants only. Not to be copied or reproduced without our consent.

Page 84

addressed mail. B says that none of them is delivered within 2 business days. This is not supported by the evidence used in the argument. Choice C is tempting too but we must be on guard. It is conceivable that some incorrectly addressed mail could be delivered within two business days too. Choice E talks about “within two days” versus “between Two and Three days”. The argument does not support this statement. Consider another argument asking us to make a logical conclusion. Orthodox medicine is ineffective at both ends of the spectrum of ailments. At the more trivial end, orthodox medicine is largely ineffective in treating aches, pains, and allergies, and, at the other extreme, it has yet to produce a cure for serious, life-threatening diseases such as advanced cancer and lupus. People turn to alternative medicine when orthodox medicine fails to help them and when it produces side effects that are unacceptable to them. One of the reasons that alternative medicine is free of such side effects is that it does not have any effects at all.

If the above statements are true, which of the following can be properly concluded from the above? ♦ Practitioners of alternative medicine are acting in bad faith. ♦ There are some medical conditions for which no orthodox or alternative

treatment is effective. ♦ There are some trivial illnesses that can be treated effectively by the

methods of alternative medicine. ♦ There are no effective medical treatments that are free of unacceptable side

effects. ♦ Orthodox medicine will soon produce a solution for the diseases that are

currently incurable by alternative medicine. The evidence is that “orthodox” medicine does not work at trivial end and at the serious end of the disease spectrum. People turn to “alternative medicine” when orthodox medicine does not help and when it produces serious side effects. But the alternative medicine does not have any effect, leave alone side effects. The only logical inference permissible is that for the trivial and for the serious illnesses, neither orthodox medicine nor alternative medicine offers effective help. Choice B states this “recycled information” best. Choice A talks about the “practitioners of alternative medicine” acting in bad faith. This is what you would expect but is not supported by the evidence used. Choice C is contrary to the information presented. The evidence is that alternative medicine does not work. Choice D is tempting but we will not select it because, obviously, the orthodox medicine appears to work in the mid range of the disease spectrum. Choice E is asking you to gaze at a crystal ball and make a “wild” prediction that is not supported by anything in the argument. Although a “conclusion” is often termed “prediction”, it is not called a “wild prediction” unsupported by the evidence used.

Page 85: CRITICAL REASONING 101 T - Amazon S3 · Critical Reasoning Section on the GMAT and GRE will require that you draw conclusions that are logically certain for a set of stated or implied

© Maple Leaf International Consulting, Inc., New York New York. For exclusive use by our registered participants only. Not to be copied or reproduced without our consent.

Page 85

Let us get up to speed with our “recycling skill” by examining another argument asking us to make a logical conclusion. “Each year, an official estimate of the stock of cod in the Grand Banks is announced. This estimate is obtained by averaging two separate estimates of how many cod are available, one based on the number of cod caught by research vessels during a once-yearly sampling of the area and the other on the average number of tons of cod caught by various commercial vessels per unit of fishing effort expended there in the past year – a unit of fishing effort being one kilometer of net set out in the water for one hour. In previous decades, the two estimates usually agreed closely. However, for the last decade, the estimate based on commercial tonnage has been increasing markedly, by about the same amount as the sampling-based estimate has been decreasing.”

If the statements in the passage are true, which of the following can be concluded on the basis of the information provided above?

♦ Last year’s official estimate was probably not much different from that ten years ago.

♦ The number of commercial vessels fishing for cod in the Grand Banks has increased substantially over the past decade.

♦ The sampling-based estimate is more accurate than the estimate based on commercial tonnage in that the data on which it relies is less likely to be inaccurate.

♦ The once-yearly sampling by research vessels should be used as the sole basis for arriving at the official estimate of the stock of the cod.

♦ Twenty years ago, the overall stock of cod in the Grand Banks was officially estimated to be much larger than it is estimated to be today.

The subject of the argument is “OFFICIAL ESTIMATE”, which is obtained by averaging two estimates: the average commercial tonnage and the research tonnage. The evidence is that in the last decade, the average commercial tonnage has gone up by about the same amount as the research tonnage has gone down. Of the two components that make up the official estimate, one component increases by about the same amount as the other decreases. Our conclusion is that the OFFICIAL ESTIMATE will remain unaffected by these changes. Choice A is the best recycling of this information. Choice B is tempting but we must not pick it because the number of commercial vessels has already been factored into the AVERAGE COMMERCIAL TONNAGE. Notice the highlighted part of the argument. The test is designed to check whether you are paying attention or not. Choice C talks about which is more accurate, etc. There is nothing in the argument to support this conclusion. Choice D makes a recommendation that is not required. We are asked to make a logical conclusion, not make an unsolicited recommendation. Choice E talks about “twenty years ago”. There is nothing in the argument to support this conclusion because the evidence does not go as far back as 20 years. Choice A is the best recycled evidentiary information we call conclusion.

*Be suspicious of answer choices using the word “majority”. The fact that “more people suffer from sinusitis than from arthritis” does not imply that a “majority” of people suffer from sinusitis. The fact that insiders buy more stocks now than they did previously does not mean that a majority of stocks will be owned by the insiders.

Page 86: CRITICAL REASONING 101 T - Amazon S3 · Critical Reasoning Section on the GMAT and GRE will require that you draw conclusions that are logically certain for a set of stated or implied

© Maple Leaf International Consulting, Inc., New York New York. For exclusive use by our registered participants only. Not to be copied or reproduced without our consent.

Page 86

Let us take a look at another argument asking us to process quantitative information and make a logical conclusion. ♦ Corporate officers and directors commonly buy and sell, for their own portfolios,

stock in their own corporations. Generally, when the ratio of such inside sales to inside purchases falls below 2 to 1 for a given stock, a rise in stock prices is imminent. In recent days, while the price of MEGA corporation stock has been falling, the corporation’s officers and directors have bought up to nine times as much of it as they have sold.

The facts above support which of the following predictions? (A) The imbalance between inside purchases and inside sales of MEGA stock will grow even further. (B) Inside purchases of MEGA stock are about to cease abruptly. (C) The price of MEGA stock will soon begin to go up. (D) The price of MEGA stock will continue to drop, but less rapidly. (E) The majority of MEGA stock will soon be owned by MEGA’s own officers and directors. The subject is the price of a stock and the impact the ratio of inside sales to inside purchases has on the stock price. The evidence is that if the ratio of inside sales to inside purchases is less than 2/1, then the price should go up. For MEGA corporation, the ratio is 1/9, which is significantly less than 2/1. The only logical prediction that can be made is that price of MEGA stock should go up immediately because the evidence is that price increase will happen “imminently” if the ratio is less than 2 to 1. C states this conclusion best. Choice A talks about imbalance between sales and purchases. There is nothing in the argument to support this wild prediction. Choice B makes another wild prediction not supported by the evidentiary information. Choice D is contrary to what we will expect in keeping with the information presented to us. Choice E talks about “majority of ownership”. The fact that the inside purchases exceed inside sales does not mean that a majority* of the stocks will be owned by the officers of the corporation.

Page 87: CRITICAL REASONING 101 T - Amazon S3 · Critical Reasoning Section on the GMAT and GRE will require that you draw conclusions that are logically certain for a set of stated or implied

© Maple Leaf International Consulting, Inc., New York New York. For exclusive use by our registered participants only. Not to be copied or reproduced without our consent.

Page 87

Let us take a look at another argument presenting ratio information and asking us to make a logical (not wild) conclusion. Last year the worldwide paper industry used over twice as much fresh pulp (pulp made directly from raw plant fibers) as recycled pulp (pulp made from wastepaper). A paper industry analyst has projected that by 2010 the industry will use at least as much recycled pulp annually as it does fresh pulp, while using a greater quantity of fresh pulp that it did last year. If the information above is correct and the analyst’s projections prove to be accurate, which of the following projections must also be accurate? (A) In 2010, the paper industry will use at least twice as much recycled pulp as it did last year. (B) In 2010, the paper industry will use at least twice as much total pulp as it did last year. (C) In 2010, the paper industry will produce more paper from a given amount of pulp than it did last year. (D) As compared with the last year, in 2010 the paper industry will make more paper that contains only recycled pulp. (E) As compared with last year, in 2010 the paper industry will make less paper that contains only fresh pulp. The subject of the argument is “consumption of two kinds of pulp”. Bear in mind that the “subject” is what the argument is about. Sometimes you need to process the information presented in the argument by setting up a matrix such as the one shown below:

CONSUMPTION OF PULP BY PAPER INDUSTRY PULP TYPE LAST YEAR YEAR 2010 ♦ FRESH 2+ 2++

♦ RECYCLED 1 2++ Last year for every 1 ton of Recycled pulp, the industry used over 2 tons of Fresh pulp. In 2010, the consumption of Fresh pulp will increase but the evidence does not quantify it. We will call the increase 2++ from 2+. But the evidence is that the consumption of Recycled pulp will be at least at the same level as the Fresh pulp during 2010. As we can tell from the matrix above, we cannot quantify the increase in Fresh pulp consumption but we know that the consumption of Recycled pulp will more than double in year 2010. Choice A is the best inference supported by the above analysis.

Page 88: CRITICAL REASONING 101 T - Amazon S3 · Critical Reasoning Section on the GMAT and GRE will require that you draw conclusions that are logically certain for a set of stated or implied

© Maple Leaf International Consulting, Inc., New York New York. For exclusive use by our registered participants only. Not to be copied or reproduced without our consent.

Page 88

Choice B is tempting but we must bear in mind that TOTAL PULP is not relevant to the scope of the argument. May be, there is a third kind of pulp (synthetic pulp?) that is not addressed in the evidence Choice C talks about “productivity” of the paper industry. Productivity is not supported by the evidence used in the argument. All that we know is that the industry will USE a certain kind of pulp. Choices D and E use the restrictive qualifier ONLY that says that the industry will produce paper that contains ONLY one kind of pulp. There is nothing in the argument to suggest in what fashion the industry uses pulp. Choice A is the best recycling of the argument’s evidence. You should also know that a logical conclusion question could be asked in the form of a sentence completion. We will examine such arguments in detail a bit later in this module. For now, take a look at the following: ♦ In a mature tourist market such as Bellaria, there are only two ways in

which Hotel owners can increase profits: by building more rooms or by improving what is already there. Rigid land-use laws in Bellaria rule out construction of new hotels, or, indeed, any expansion of hotel capacity. It follows that hotel owners cannot increase their profits in Bellaria since Bellarian hotels ________________________________________________________

Which of the following logically completes the argument?

♦ Are already operating at an occupancy rate approaching 100 percent year-

round. ♦ Could not have been sited any more attractively than they are even in the

absence of land-use laws. ♦ Have to contend with upward pressure on the cost of labor, which stem from

an incipient shortage of trained personnel. ♦ Already provide a level of luxury that is at the limits of what even wealthy

patrons are willing to pay for. ♦ Have shifted from serving mainly Bellarian tourists to serving foreign

tourists traveling in organized tour groups. The evidence is that there are ONLY two ways in which to increase profits: build more rooms or improve existing levels of luxury. Building more rooms is not an option because of land use laws. The conclusion is that the profits will not increase. The other option of “improving luxury” has been bypassed and the logical inference is that “improving luxury” is not a viable option either. Choice D provides this logical conclusion. Choice A is tempting but we must bear in mind that “occupancy rate” is beyond the scope of the argument. Hotels may want to build more rooms even if they are operating at 90% of the capacity.

Page 89: CRITICAL REASONING 101 T - Amazon S3 · Critical Reasoning Section on the GMAT and GRE will require that you draw conclusions that are logically certain for a set of stated or implied

© Maple Leaf International Consulting, Inc., New York New York. For exclusive use by our registered participants only. Not to be copied or reproduced without our consent.

Page 89

Choice B talks about “location”, which is beyond the scope of the argument. Choice C talks about “cost of labor” which is beyond the scope of the argument. Choice E talks about shift in focus from “foreign tourists” to “tour groups”, a major wrinkle. You should expect about 5 to 6 questions on the GMAT asking you to make a logical inference, and you need to hone your skill in recycling the evidentiary information. Also, bear in mind that the prediction you are allowed to make must be consistent with the evidence used, and not be a wild, way-out one.

EXPLANATION OF A DISCREPANCY OR PARADOX

A PARADOX is a statement that is seemingly contradictory or inconsistent with commonsense and yet might be true. Rich folks are unhappy is a paradoxical statement because you would think that a rich person should be able to buy happiness. Poor people are happy is also paradoxical because if you are poor and unable to make ends meet, you should not be happy. Yet, both of these statements, albeit paradoxical, are true. How do we rationally or logically explain the paradox that rich people are unhappy? They are unhappy because they run into tax complications or they are too stressed by their lifestyles, or because they are exploited by men and women who love them for their money, etc. The logical reasoning section on the GMAT, GRE, and LSAT will test this ability to identify a logical explanation for the apparent paradox that will be described in the passage. Before you begin to identify the explanation for the discrepancy (which is a variation from the norm or expected) or the paradox (contradictory to commonsense information), you need to PROCESS THE PARADOX OR THE DISCREPANCY. Far too many test-takers get messed up because they do NOT process the discrepancy or the paradox presented in the passage and end up picking answers that do not explain anything,, leave alone the discrepancy. PROCESS THE INFORMATION PROPERLY. The test is a measure of your ability to process information in context and in terms of the specifics. Having processed the discrepancy or the paradox presented to you, look for the best explanation for the paradox or the discrepancy. Keep in mind that more than one answer choice might seem to do the job but you must pick the BEST one that explains the paradox as completely as possible. Take a look at the following passage in which a paradox or discrepancy is presented: “During a recent sales conference, the national sales manager of McDonald’s chain of restaurants announced that the overall sales of the restaurant chain increased last year by 10% over the sales in the preceding year. The accounting manager, who followed the sales manager, said that each and every restaurant doing business for at least one year showed a marginal decline in sales last year compared to the previous year.” If both the sales manager and the accounting manager are accurate with their statements, how do we resolve the apparent conflict we see in the information presented? We begin to attack the problem by noticing that the accounting manager introduces a category of restaurants called “more than one year old restaurants”. This means that there is another implied category called “doing business for less than one year” restaurants. If the first category showed marginal decline in sales and if the overall sales of all restaurants increased year over year, then the inference is that the second category of restaurants made up for the shortfall.

Page 90: CRITICAL REASONING 101 T - Amazon S3 · Critical Reasoning Section on the GMAT and GRE will require that you draw conclusions that are logically certain for a set of stated or implied

© Maple Leaf International Consulting, Inc., New York New York. For exclusive use by our registered participants only. Not to be copied or reproduced without our consent.

Page 90

Your ability to read the arguments in a discerning fashion will be put to the test in this segment of the test. ♦ Let us take a look at some more passages presenting a paradox or discrepancy, and learn how to

resolve the paradox or the discrepancy. EXAMPLE 1. Adult female rats, which have never before encountered rat pups will start to show maternal behaviors after being confined with a pup for about seven days. This period can be considerably shortened by disabling the female’s sense of smell or by removing the scent-producing glands of the pup. Which of the following hypotheses best explains the contrast described above? (A) The sense of smell in adult female rats is more acute than that in rat pups. (B) The amount of scent produced by rat pups increases when they are in the presence of a female rat that did not bear them. (C) Female rats that have given birth are more affected by olfactory cues than are female rats that have never given birth. (D) A female rat that has given birth shows maternal behavior toward rat pups that she did not bear more quickly than does a female rat that has never given birth. (E) The development of a female rat’s maternal interest in a rat pup that she did not bear is inhibited by the odor of the pup. The inference we can make is that the onset of maternal instinct is delayed by the factor of smell. Choice E explains this phenomenon best and is the correct answer. EXAMPLE 2: “Mud from a lake on an uninhabited wooded island in Northern Lake Superior contains toxic chemicals, including toxaphene, a banned pesticide for cotton that previously was manufactured and used, not in the nearby regions of Canada or the northern United States, but in the southern United States. No dumping has occurred on the island. The Island Lake is sufficiently elevated so that water from Lake Superior does not reach it.” The statements above, if true, most strongly support which of the following hypotheses? (Read this to mean: How do you explain the fact that we have toxaphene in the island lake?) (A) The waters of the island lake are more severely polluted than those of Lake Superior. (B) The toxaphene was carried to the island in the atmosphere by winds. (C) Banning Chemicals such as toxaphene does not aid the natural environment. (D) Toxaphene has adverse effects on human beings but not on other organisms. (E) Concentrations of toxaphene in the soil of cotton-growing regions are not sufficient to be measurable. If there is no physical contact or dumping, how did the chemical get to north of Lake Superior from Georgia? May be, migrating birds stop by at the island lake and contaminate the waters with traces of chemicals they picked up in Georgia, where they spent the Canadian winter months. May be, a plane carrying the chemical to Alaska from Georgia crashed near the island lake a while ago and spilled the chemical into the waters. May be, the winds carried the chemical from Georgia to the lake waters. The possibilities are endless and choice B provides a reasonable explanation for what might have happened. As you can see, choices A, C,D, and E do not provide an explanation for how the chemical got to the island lake waters from the far away Georgia. Only choice B does and is the best answer. Let us do one more and understand how we can deal with questions of this genre.

Page 91: CRITICAL REASONING 101 T - Amazon S3 · Critical Reasoning Section on the GMAT and GRE will require that you draw conclusions that are logically certain for a set of stated or implied

© Maple Leaf International Consulting, Inc., New York New York. For exclusive use by our registered participants only. Not to be copied or reproduced without our consent.

Page 91

Extinction is a process that can depend on a variety of ecological, geographical and physiological variables. These variables affect different species of organisms in different ways, and should, therefore, yield a random pattern of extinction. However, the fossil record shows that extinction occurs in a surprisingly definite pattern, with many species vanishing at the same time. Which of the following, if true, forms the best basis for at least a partial explanation of the patterned extinction revealed by the fossil record? (A) Major episodes of extinction can result from widespread environmental disturbances that affect numerous different species. (B) Certain extinction episodes selectively affect organisms with particular sets of characteristics unique to their species. (C) Some species become extinct because of accumulated gradual changes in their local environments. (D) In geologically recent times, for which there is no fossil record, human intervention has changed the pattern of extinction. (E) Species that are widely dispersed are the least likely to become extinct. How do we explain the fact that many species vanished at the same time? May be, a nuclear war wiped out all species at the same time. May be, ice age resulted in the death of all species at the same time. Choice A suggests such a widespread calamity that may have affected numerous species at the same time. As you can see, choices B through E do not explain how all or many species vanished at the same time when we would expect different climatic and environmental factors to have different effects on different species.

Page 92: CRITICAL REASONING 101 T - Amazon S3 · Critical Reasoning Section on the GMAT and GRE will require that you draw conclusions that are logically certain for a set of stated or implied

© Maple Leaf International Consulting, Inc., New York New York. For exclusive use by our registered participants only. Not to be copied or reproduced without our consent.

Page 92

INFERENCE QUESTIONS INVOLVING SENTENCE COMPLETION - Revisited

Sentence Completion questions usually require that you understand the flow of information in the argument and pick a choice that best completes the argument. Sometimes, you need to identify an inference and pick a choice that is logically consistent with the rest of the information. The strategy appropriate for dealing with such questions is that you must try to get a “big picture” idea as to how the rest of the sentence must be completed before you begin to examine choices. You can then go and look for a choice that best expresses the picture you have in your mind. It may be the single most outrageous time-waster to try to marry each choice to the argument and determine whether any choice makes sense. Let us see how we can predetermine close and identify the close in the choices. Consider the following flow of information: “John likes Mary more than he does Lisa because he took Mary on a Caribbean cruise. John also bought Mary a diamond ring on her birthday. However, it is clear that John does not dislike Lisa because, if he did, _________________________________________________________________________________________” We can see the double negative- “does not dislike Lisa” and notice that the sentence must be completed in a manner that states a “like for Lisa” as a “negative” “John does not dislike Lisa because, if he did, he would not have done (some positive thing for or with Lisa).” “done positive thing for or with Lisa” could be anything: “sent her a dozen roses on Valentine’s day.” Or “gone to her birthday bash”, or “gone to see a movie with Lisa”. We can also complete the sentence with a negative as follows: “John does not dislike Lisa because, if he did, he would have done some negative things to or with her” For example, the choice might read: ““John does not dislike Lisa because, if he did, he would have dumped her sooner”, or “John does not dislike Lisa because, if he did, he would have no contact with her whatsoever.” The chances are, the choice you are about to select will be in the “negative form” identified above.

Page 93: CRITICAL REASONING 101 T - Amazon S3 · Critical Reasoning Section on the GMAT and GRE will require that you draw conclusions that are logically certain for a set of stated or implied

© Maple Leaf International Consulting, Inc., New York New York. For exclusive use by our registered participants only. Not to be copied or reproduced without our consent.

Page 93

Consider the following SENTENCE COMPLETION argument. Which of the following, if true, is the most logical completion of the argument below? “The tax system of the Republic of Grootland encourages borrowing by granting its taxpayers tax relief for interest paid on loans. The system also discourages saving by taxing any interest earned on savings. Nevertheless, it is clear that Grootland’s tax system does not consistently favor borrowing over saving, for if it did, there would be no ___________________” (A) tax relief in Grootland for those portions of a taxpayer’s income, if any, that are set aside to increase that taxpayer’s total savings. (B) tax relief in Grootland for the processing fees that taxpayer pays to lending institutions when obtaining certain kinds of loans. (C) tax relief in Grootland for interest that taxpayers are charged on the unpaid balance in credit card accounts. (D) taxes due in Grootland on the cash value of gifts received by taxpayers from banks trying to encourage people to open savings accounts. (E) taxes due in Grootland on the amount that a taxpayer has invested in interest-bearing savings account. The nutshell essence of the argument is: “The tax system favors borrowing over savings because there are some tax incentives for borrowing. However, the tax system does not favor borrowing over saving because if it did there would be no tax benefits or incentives for savings.” The system does not always favor borrowing implies that the system does favor savings in some manner. This is the logical inference we can make. We are going to eliminate choices D and E because they begin with “taxes due” or “tax liability” instead of “tax incentive”. Choices A, B, and C begin with “tax incentives” but choices B and C talk about “tax incentives for borrowing”. The only choice that mentions “tax incentive for savings” is choice A and that is the one we must select. IF you predetermine the close in macro terms, you can easily identify the choice that matches your predetermined completion. Let us see how the Grootland can have real-life significance in the context of what is happening in the United States of America. “The US tax laws favor borrowing over saving because you are allowed to write off interest paid on home mortgages. The tax laws also require that you pay tax on interest earnings and on capital gains. However, it is clear that the tax laws do not consistently favor borrowing over saving because, if they did, there would be not any tax break for the money put way in the retirement savings accounts or IRA’s.”

Page 94: CRITICAL REASONING 101 T - Amazon S3 · Critical Reasoning Section on the GMAT and GRE will require that you draw conclusions that are logically certain for a set of stated or implied

© Maple Leaf International Consulting, Inc., New York New York. For exclusive use by our registered participants only. Not to be copied or reproduced without our consent.

Page 94

Another example of inference question asking you to complete a sentence is given below. “Companies defend their established practices and products, and resist change even if the market forces demand such change. When consumers insist on newer technologies and products, some companies respond to this demand by offering the same old-technology products with improved cosmetics. This rear-guard action is exemplified by ___________________________________________ “ (A) an automobile manufacturer offering a new line of hybrid cars running on electricity

and conventional fuel to meet the growing demand for fuel-efficient cars. (B) A manufacturer of analog mobile phones offering a set of designer face plates on their

analog phones in the face of a growing demand for digital mobile phones. (C) A computer manufacturer offering free office suite with every computer purchased. (D) A supermarket chain beginning to accept debit cards. (E) Consumers boycotting a line of genetically modified foods carried by a leader grocery

chain. The nutshell essence of the argument is that companies respond to demand for new technology products by ‘souping’ up their existing products. The best example of this rear-guard action is stated in choice B. In fact, choice A is a poor example of the scenario described in the argument. Choices C, D, and E do not address the meat of the argument. Now that we have examined the mainstream question types and the best approach to dealing with them, let us examine some other types of questions you may come across during the test. ♦ QUESTIONS ASKING YOU TO IDENTIFY A PARALLEL STRUCTURE. ♦ QUESTIONS ASKING YOU TO IDENTITY THE RELATIONSHIP BETWEEN

TWO PARTS OF THE ARGUMENT. ♦ Questions asking you to summarize the essence of the argument. Let us examine the parallel structure arguments first.

Page 95: CRITICAL REASONING 101 T - Amazon S3 · Critical Reasoning Section on the GMAT and GRE will require that you draw conclusions that are logically certain for a set of stated or implied

© Maple Leaf International Consulting, Inc., New York New York. For exclusive use by our registered participants only. Not to be copied or reproduced without our consent.

Page 95

Logic Structure Questions

Logic structure questions ask you to identify a form that is parallel to the one provided. Your strategy must be to understand the argument in terms of X, Y, and Z. If the argument goes: “Any person who is blind, cannot see. John cannot see. Therefore, John is blind”, we will understand the argument as follows: “If X, then Y. Y. Therefore, X”. We should try to change all the answer choices to this format and decide which one has the same structure. Let us see how we can hone this skill by examining the following arguments. Parallel Structure: “We all know that it is against international law to sell plutonium to countries that are on the verge of nuclear capability. But if the United States of America does not sell plutonium to Pakistan, then some other country certainly will.” What do we notice in this argument? We notice that the argument mentions some illegal act, namely selling plutonium to countries on the verge of nuclear capability. Let us call this illegal act X and read the argument in this form: “We all know that it is illegal to do X. If entity Y does not do it, then Z will.” The GMAT answer choice will change the X, Y, and Z to something else. What are the possible options here? Unlimited. For example, the choice could read: “It is against the city bye-laws to dump garbage in parks. But if I do not do it, then some Joe Six-pack is going to do it.” Or, “It is illegal to smoke in airline toilets. But if I did not do it, some other passenger would have done it.” You get the drift, don’t you? The answer choice you were required to pick in the test read: “We all know that it is against the law to break and enter peoples’ homes. But if my client did not break and enter the Scotts’ home, then somebody else would certainly have.” When you deal with parallel structure arguments, be sure to read the argument in terms of X, Y, and Z. Let us take a look at another parallel form argument and see how we can apply this “trick” to develop a good understanding of what the answer choice will look like.

Page 96: CRITICAL REASONING 101 T - Amazon S3 · Critical Reasoning Section on the GMAT and GRE will require that you draw conclusions that are logically certain for a set of stated or implied

© Maple Leaf International Consulting, Inc., New York New York. For exclusive use by our registered participants only. Not to be copied or reproduced without our consent.

Page 96

“IF Max was guilty, he would not ask the police to investigate. Therefore, his asking the police to investigate shows that he is not guilty.”

The logical structure of the argument above is most similar to which one of the following?

If Lucille were in the next room, I would not be able to see her. Therefore, the fact that I can see her shows that she is not in the next room.

IF Sam were rich, he would not spend his vacation in Alaska. Therefore, his spending his vacation in the Bahamas shows that he is rich.

IF Joe were over 40, he would not want to learn to ski. Therefore, the fact that Joe does not want to learn to ski shows that is over 40.

If Mark were a good cook, he would not put cinnamon in the chili. Therefore, the fact that he is not a good cook shows that he put cinnamon in the chili.

IF Sally were sociable, she would not avoid her friends. Therefore, the fact she is sociable shows that she does not avoid friends.

Let us read the argument in this form: “If X is Y, then Z will not happen. Because Z is happening, X is not Y.” The first choice is exactly in the above form and is the answer. The second choice is of the form: “If X is Y, Z will not happen. Because A is happening, X is Y.” And we can see that it is not in the required parallel form. The Third choice is of the form: “If X is Y, Z will not happen. Because Z is NOT happening, X is Y”. We can see that this choice is not in the required form either. The Fourth choice is of the form: “If X is Y, Z will not happen. Because X is NOT Y, Z happened.” Do we think that this choice is in the required form? Not by a long shot. The Fifth choice is of the form: “If X is Y, Z will not happen. Because X is, Z does not happen”. What do we do with this choice? Give it a cold shoulder and move on to confirm the first choice. Let us take a look at another parallel form argument.

Page 97: CRITICAL REASONING 101 T - Amazon S3 · Critical Reasoning Section on the GMAT and GRE will require that you draw conclusions that are logically certain for a set of stated or implied

© Maple Leaf International Consulting, Inc., New York New York. For exclusive use by our registered participants only. Not to be copied or reproduced without our consent.

Page 97

People who are red/green color-blind cannot distinguish between green and brown. Gerald cannot distinguish between green and brown. Therefore, Gerald is red/green color-blind.

Which one of the following most closely parallels the reasoning in the above argument?

People who are fair-skinned suffer from sunburn. William is fair-skinned. Therefore, William suffers from sunburn.

People who are suffering from sinusitis lose their sense of smell. Mary has lost her sense of smell. Therefore, Mary is suffering from sinusitis.

People who have suffered from jaundice cannot become blood donors. Jean is a blood donor. Therefore, Jean has not suffered from jaundice.

People who are color-blind cannot become airline pilots. Arthur is color-blind. Therefore, Arthur cannot become an airline pilot.

People who are diabetic cannot eat large amounts of sugar. Freda is diabetic. Therefore, Freda is on a special diet.

Let us read the given argument in the form: “If people are X, they cannot do Y. Z cannot do Y. Therefore Z is X.” We can tell that the first choice is not in the required form because it reads: “If people are X, they do Y. Z is X. Therefore Z does Y.” Not a good one. The second choice is in the required form because it reads: “If people are X, they cannot do Y. Z cannot do Y. Therefore, Z is X.” We must select the second choice. The Third choice is of the form: “If people are X, they cannot do Y. Z is Y. Therefore, Z is not X” Can you tell that this is not the form in which the original argument is stated? The Fourth choice is of the form: “People who are X cannot do Y. Z is X. Therefore Z is not Y”. Check to see whether this is in the same form as the original argument. It is not. The Fifth choice is of the form: “People who are X cannot do Y. Z is X. Therefore Z is A” Compare this to the form we have in the gray bar above and you will see that the form is not the same. We must stay with choice 2. Logic Structure questions are usually tested as high difficulty level questions. It will help you identify the correct answer if you can read the argument in terms of X, Y, and Z so that you will not be confused each time you read a choice. GMAT can also test ‘COMPREHENSION’ by asking you to understand the essence of what is described. The question might simply ask you to demonstrate your comprehension by picking the appropriate choice

from the list of 5 options. Let us consider such arguments in the following pages.

Page 98: CRITICAL REASONING 101 T - Amazon S3 · Critical Reasoning Section on the GMAT and GRE will require that you draw conclusions that are logically certain for a set of stated or implied

© Maple Leaf International Consulting, Inc., New York New York. For exclusive use by our registered participants only. Not to be copied or reproduced without our consent.

Page 98

COMPREHENSION TESTED IN THE CONTEXT OF CRITICAL REASONING

BOLD FACE ARGUMENTS You should expect to get at least one argument in which two sections are highlighted or provided in bold face. You will be expected to understand in what manner one bold face or highlighted section relates to the other, and pick a choice that expresses this connection best. In short, how do you understand the bold face segments in relation to the whole argument? Consider the following argument showing two highlighted sections: “Interest rates are manipulated by the Federal Reserve in order to influence economic growth. It is generally true that each time the Federal Reserve cut interest rates in the past, economic activity picked up and consumer spending increased. However, the recent maneuvers by the Federal Reserve to change the course of economic activity paint a different picture. The recent interest rate cuts by the Federal Reserve, far from accelerating economic growth and promoting consumer spending, will depress economic activity and encourage consumers to seek alternate avenues for investment of their savings.” The second boldface section of the argument is related to the first boldface section in which of the following ways? (A) The first is a general principle upon which the second boldface statement is based. (B) The first is a cause and effect that can be expected anecdotally and the second

acknowledges a circumstance in which the same will be repeated. (C) The first boldface provides the cause and effect connection between two activities and the

second boldface provides an exceptional circumstance in which the same cause and effect connection between the same activities will not hold.

(D) The first is a universal axiom that will be denied in the second boldface. (E) The first provides an indication of what can be expected if a course of action is undertaken;

the second provides an alternative outcome that is likely to happen. Our understanding of the boldface sections is that the first bold face provides a cause and effect connection between interest rate cuts and economic activity and the second bold face describes an exception to the cause and effect expectation. Choice C is the best statement of this understanding. Choices B and E are very close but contain major wrinkles: Choice B states that the second boldface is a confirmation of the expectation outlined in the first boldface. That is not true. Choice E states that the second boldface describes an “alternative outcome”. The outcome described in the second boldface is the “opposite” outcome, not the “alternative” outcome. You must examine answer choices carefully, take words literally for their meanings, and be careful not to select answers using unacceptable word choices. You should expect to one question on the test asking you to deal with “boldface” sections in the GMAT. Some boldface arguments are easier to deal with. For example, the first boldface will describe a principle and the second boldface may describe and example of that principle. Try to get a big picture understanding of the manner in which the two boldface sections are related and pick a choice that best matches this understanding. Make sure that your understanding is good. Also, be sure to parse words, and do not settle for choices using vague terms and descriptions.

Page 99: CRITICAL REASONING 101 T - Amazon S3 · Critical Reasoning Section on the GMAT and GRE will require that you draw conclusions that are logically certain for a set of stated or implied

© Maple Leaf International Consulting, Inc., New York New York. For exclusive use by our registered participants only. Not to be copied or reproduced without our consent.

Page 99

FLAW IN THE ARGUMENT REDUX (Also tested as ‘why is the argument vulnerable to criticism?’)

arlier in this file, we discussed what constitutes a FLAW or a defect that detracts from the wholeness or the logical validity of an illogical argument. The flaw could be an interchangeable use of terms when

the terms of are not synonymous, ignoring an alternate and equally plausible explanation for the evidence, concluding that something is bad on the basis of a single known defect or concluding that something is good on the basis of a single known positive attribute, vagueness in the use of a key term, and so on. The arguments advanced by politicians are fraught with ‘flaws’, especially of the kind that involves interchangeable use of terms that are not necessarily synonymous or the kind that involves deliberate vagueness. Consider the following argument involving interchangeable use of an adjective and a noun with the same force: “Before the war, we believed that Baddam Assassin had weapons of mass destruction, and took action to remove Baddam. Since the war started, we have discovered weapons of mass destruction programs in Riaq. Therefore, our decision to go to war is justified” Notice that the argument uses ‘weapons of mass destruction’ as a noun phrase in the first part but switches to the use of the same phrase as an adjective in the second part (weapons of mass destruction programs) modifying the noun, ‘programs’. According to one theory, if all citizens living in a democracy are training in logical/ critical reasoning and to identify flaws in arguments, then politicians will be out of business. The good news for those who perpetuate flawed arguments is that a significant proportion of voters in an election are not trained to identify flaws or do not bother to do so. That is your thought for the day! FLAW ARISING FROM IGNORING AN ALTERNATE EXPLANATION FOR THE EVIDENCE. Any argument of the type ‘one explanation for the evidence versus another’ falls into this category. Consider the argument: “While it might be expected that those neighborhoods most heavily patrolled by police have the least crime, the statistical evidence overwhelmingly supports the claim that such neighborhoods have the most crime. This shows that the presence of police does not deter crime in a neighborhood.”

E

Page 100: CRITICAL REASONING 101 T - Amazon S3 · Critical Reasoning Section on the GMAT and GRE will require that you draw conclusions that are logically certain for a set of stated or implied

© Maple Leaf International Consulting, Inc., New York New York. For exclusive use by our registered participants only. Not to be copied or reproduced without our consent.

Page 100

A weakening of this argument could be done by suggesting that it is a chicken and egg situation: Police presence is in response to the relatively high crime rate in the neighborhood, and in the absence of police presence, rate of crime would be much greater. The flaw in the argument is that it fails to consider the possibility that the police presence in a particular neighborhood is often a response to the relatively high crime rate in that area. Consider another FLAWED argument: “In New York State, where there is a strict gun control law in place, the rate of violent crime is much greater than that in Wyoming, where there is no gun control law in place. Therefore, we can conclude that the strict gun control law in New York State is not working.” Our counter could be that if New York State did not have strict gun control law in place, the rate of crime would be even greater than it is today. The flaw in the argument is that it fails to consider this possibility. FLAW IN THE ARGUMENT ARISING FROM INTERPRETATION OF DEGREE “People who are rich drive expensive automobiles. Sam is richer than Josh. Therefore, Sam drives a car that must be more expensive than Josh’s car.” What is the flaw in the above argument? The argument fails to consider the possibility that at a certain level of wealth, the utility of that wealth does not increase indefinitely. It also fails to consider the possibility that Josh is rich enough to afford and to drive the same car that Sam does. Consider the following argument having the same flaw and tested on the LSAT. “It is the characteristic of great artists generally, and of great writers in particular, to have discerning view of the basic social and political arrangements of the society in which they live. Therefore, the greater the writer one is, the more astute one will be in perceiving the basic social and political arrangements of one’s society” The flaw in the argument is that it assumes, without justification. That because people who have one quality tend to have a second quality, those who have more of the fist quality will have more of the second.

Page 101: CRITICAL REASONING 101 T - Amazon S3 · Critical Reasoning Section on the GMAT and GRE will require that you draw conclusions that are logically certain for a set of stated or implied

© Maple Leaf International Consulting, Inc., New York New York. For exclusive use by our registered participants only. Not to be copied or reproduced without our consent.

Page 101

Arguments having the ‘degree flaw’ can also be such that they ignore the possibility that a certain threshold level is required for the outcome as described. “A recent study shows that middle-class families are just as happy as families earning annual incomes in excess of $500,000. Therefore, the study confirms my view that income level is not a relevant consideration in determining happiness.’ The problem with this argument is that unless a family makes a certain minimum level of income, it will not be happy. Because the argument states that income level is not important, it implies that a person who makes no income ought to be just as happy as one making $30,000 per annum and as one making $600,000 per year. Consider another argument having the same flaw: “All sun-screen lotions display SPF ratings on the containers – ratings ranging from 5% to 50%. But the manufacturers and experts agree that in order for the sun-screen lotion to be effective, it must be frequently reapplied to the skin exposed to the sun. Therefore, we can conclude that the SPF rating is not important as long as one keeps splashing on the sun-screen lotion periodically.’ Again, the flaw is that the argument ignores the possibility that in order for the lotion to work, it must carry a minimum rating of whatever percentage. FLAW ARISING FROM CRITICISM OF THE PERSON RATHER THAN OF THE PERSON’S VIEWS Politicians engage in this type of flawed arguments. In America, such arguments are frequently referred to as ‘swift-boating’, made famous by the swift-boat veterans who successfully attacked Kerry’s character rather than his ideas, and helped George Bush win re-election. Consider the following argument engaging in swift-boating: “Roger Bacon,, the famous 13th century scientist, is said to have made important discoveries in optics. He was an early advocate of hands-on experimentation, and as a teacher warned his students against relying uncritically on the opinions of authorities. Nevertheless, this did not stop Bacon himself from appealing to authority when it was EXPEDIENT for his own argumentation. Therefore, Bacon’s work should be disregarded in view of the contradiction between his statements and his own behavior.”

Page 102: CRITICAL REASONING 101 T - Amazon S3 · Critical Reasoning Section on the GMAT and GRE will require that you draw conclusions that are logically certain for a set of stated or implied

© Maple Leaf International Consulting, Inc., New York New York. For exclusive use by our registered participants only. Not to be copied or reproduced without our consent.

Page 102

What are the possible explanations for why Bacon appealed to authority? He may have done so after CRITICALLY examining their works for relevance to his own argumentation. The argument ignores this possibility and engages in character assassination by suggesting that he acted in self-interest (‘expedient’) rather than in the interest of objectivity. Therefore, the flaw is that the argument criticizes Bacon’s conduct rather than his argumentation in order to question his scientific findings. Notice how we paid attention to the specifics of information presented in the passage? You must process the specifics and absorb the nuances introduced by the use of qualifiers. FLAW IN THE ARGUMENT ARISING FROM IGNORING THE QUALIFIERS Arguments having this flaw take the statements out of context and stripped of qualifiers. A recent report on USA Today is a case in point. USA Today ran a story in June stating that its earlier report about telephone companies providing data about calls to NSA needs to be clarified in view of the fact that the ‘paper could not confirm that the telephone companies provided data to the government under a contractual obligation’. The qualifier here is important. The fact that they may not have provided call data ‘under a contractual obligation’ does not mean that they did not provide call data under a non-contractual obligation. The conservative bloggers and talk-show hosts ran with the story that ‘the telephone companies did not provide call data to the government’ period. They forgot to factor in the qualifier that was part of the original story. Managers, Researchers, and Lawyers need to pay attention to qualifiers in order to be able to make the correct decisions and to process information in context. If a statement is processed without the attached qualifiers, then you are guilty of taking the information out of context. You are also guilty of the same offense if you take the qualifier used in a statement and turn that into something else. If the original statement is that ‘she is pretty mean’, we cannot process this statement out of context as ‘she is pretty’. In the original statement, ‘pretty’ is used adverbially whereas In our interpretation, it is used as an adjective. Pay attention to these nuances. NOTE: ‘FLAW’ in the argument can also be tested in many different ways: which of the following would weaken the argument by providing an alternate explanation? The argument is vulnerable to criticism on what grounds? What is the questionable assumption? Etc.

Page 103: CRITICAL REASONING 101 T - Amazon S3 · Critical Reasoning Section on the GMAT and GRE will require that you draw conclusions that are logically certain for a set of stated or implied

© Maple Leaf International Consulting, Inc., New York New York. For exclusive use by our registered participants only. Not to be copied or reproduced without our consent.

Page 103

SUBJECT-SCOPE-QUALIFIER

STRATEGY REVISITED We saw briefly at the outset that Critical Reasoning is about paying ‘critical’ attention to the qualifiers used in the argument. The qualifiers may be ‘subject specific’ and ‘scope specific’. If the argument says that “owners of ‘unique’ artworks have an ‘ethical obligation’ to preserve the artworks for posterity”, you should know that the subject is ‘unique artwork”, not just artwork, and the scope is about an ethical obligation, not any other kind of obligation such as legal obligation. If the answer choice does not mention “unique artwork” or makes a reference to ‘legal obligation’, you cannot pick that choice. Your task is to write down on your scratch paper the qualifiers defining the subject and pertaining to the scope, and to make sure that the answer choice does not talk about any other qualifiers. Consider the following example: Jorge: Owners of any work of art, simply by virtue of ownership, ethically have the right to destroy that artwork if they find it morally or aesthetically distasteful, or if caring for it becomes inconvenient. Shanna: Ownership of unique artworks, unlike ownership of other kinds of objects, carries the moral right to possess but not to destroy. A unique work of art with aesthetic or historical value belongs to posterity and so must be preserved, whatever the personal wishes of its legal owner. On the basis of their statements, Shanna and Jorge are committed to disagreeing about the truth of which of the following statements? Anyone who owns a portrait presenting his or her father in an unflattering light would for that

reason alone be ethically justified in destroying it. People who own aesthetically valuable works of art have no moral obligation to make them

available for public viewing. Valuable paintings by well-known artists are seldom intentionally damaged or destroyed by

their owners. If a piece of sculpture is not unique, its owner has no ethical obligation to preserve it if doing

so proves burdensome. It is legally permissible for a unique and historically valuable mural to be destroyed by its

owner if he or she tires of it. Notice that the argument uses a lot of qualifiers, and we must be on guard. The argument is about ‘legal ownership’ of ‘unique artworks’, and about a ‘moral or ethical obligation’ to preserve it for the future generations. Choice A is the only choice that uses all the elements correctly and answers the question. Choice B talks about ‘making it available to public viewing’, which is not the same as preserving it for posterity. Choice C talks about ‘intentional damage’ – not relevant to the scope of the argument. Choice D talks about ‘not unique’ artworks. The argument is about ‘unique artworks’. We cannot mess with choice D. Choice E talks about ‘legal permissibility’, which is not addressed anywhere in the argument as the scope of the argument. The argument is about an ethical obligation arising from a legal ownership of unique artworks. You will be expected to critically parse each answer choice and to stay away from the ones using the wrong adjectives or any information not relevant to the scope of the argument. Let us hone this skill by examining additional arguments in the following pages.

Page 104: CRITICAL REASONING 101 T - Amazon S3 · Critical Reasoning Section on the GMAT and GRE will require that you draw conclusions that are logically certain for a set of stated or implied

© Maple Leaf International Consulting, Inc., New York New York. For exclusive use by our registered participants only. Not to be copied or reproduced without our consent.

Page 104

If you want to understand why Qualifiers are a big deal in a GMAT argument, you must bear in mind that an argument usually makes a big deal about a specific, well-defined category of people or objects. Also, the scope may be limited to another specific, well-defined activity. Consider the following argument: “American Citizens living overseas but maintain family ties in the United States have a legal obligation to declare world-wide income and file tax returns.” Notice that we are not talking about a global population of people. Also, the blinking phrases tell us that the scope is limited to some specific activities pertaining to some well-defined circumstances. PEOPLE AMERICANS NON-AMERICANS LIVING NOT LIVING IN THE STATES IN THE STATES BUT OVERSEAS HAVE FAMILY DO NOT HAVE FAMILY TIES IN THE STATES TIES IN THE STATES OBLIGATIONS (of this group of people) FAMILIAL MORAL/ETHICAL PROFESSIONAL LEGAL As you can see from the tree above, the argument is limited to elements in the blinking background and in the ‘loop’, as it were. You are going to be asked to pay attention to these qualified items of information, and make a note of them. You will also be required to recognize the same qualifiers in your answer choice, and to stay away from answer choices using the same qualifiers to describe anything other than a subject or using different qualifiers. For example, you cannot pick a choice that talks about “professional obligation” on the part of Americans. Likewise, you cannot pick a choice that talks about ‘people’ or about ‘Americans living in the United States’ or about ‘Americans not living in the United States and do not have family ties in the States’. You will understand that any information outside the circles defined will be considered outside the scope of the argument. If you come across an argument using many qualifiers, be sure to write them down. Also, if the argument does not use any qualifier to define the subject or the scope, be careful not to pick an answer choice that uses some undefined qualifiers. If the argument talks about “Americans” without any qualifiers, and if the answer choice talks about “Obese Americans”, you cannot pick that choice. Likewise, if the original argument talks about “obligation” without any qualifiers, you cannot pick a choice that talks about “moral obligation” or “legal obligation”. PAY ATTENTION. That is what it takes to do well in Critical Reasoning, and in the GMAT.

Page 105: CRITICAL REASONING 101 T - Amazon S3 · Critical Reasoning Section on the GMAT and GRE will require that you draw conclusions that are logically certain for a set of stated or implied

© Maple Leaf International Consulting, Inc., New York New York. For exclusive use by our registered participants only. Not to be copied or reproduced without our consent.

Page 105

Let us see how we can apply the understanding in the context of some arguments.

SUBJECT SCOPE STRATEGY: Example 1: The airline president argued that the newly developed collision-avoidance system must be installed in the passenger planes immediately on the grounds that their mechanical warnings enable pilots to avoid crashes. However, the pilots said that they will not fly in planes with collision-avoidance systems that are not fully tested. Pilots argued that malfunctioning systems could mislead pilots, causing crashes. The pilots’ objection is most strengthened if which of the following is true? (A) It is always possible for mechanical devices to malfunction (B) Jet engines, although not fully tested when first put into use, have achieved exemplary performance and safety records. (C) Although collision-avoidance systems will enable pilots to avoid some crashes, the likely malfunctions of the not-fully-tested systems will cause even more crashes (D) Many airline collisions are caused in part by the exhaustion of overworked pilots. (E) Collision-avoidance systems, at this stage of development, appear to have worked better in passenger planes than in cargo planes during experimental flights made over a six-month period. The subject of the argument is “collision avoidance system” but the pilots object to a well-qualified subject – “not fully tested collision avoidance systems”. The answer choice we need to pick must make a reference to this qualified subject – ‘not fully tested collision avoidance system’. We are required to strengthen the pilot’s objection to the ‘not fully tested collision avoidance system’ on the grounds that such systems will mislead pilots causing crashes. As you can see, only Choice C uses the same qualifier, and is the logical choice strengthening the pilot’s objection. Choice A talks about “mechanical devices”. You must bear in mind that the argument is about “collision avoidance systems” which may or may not be mechanical devices. Therefore, “mechanical devices” must be considered ‘beyond the scope’. Also, notice the absence of the qualifier “not fully tested systems” in this answer choice. Choice B talks about “not fully tested jet engines”. Jet engines are not the subject of the argument, and must be considered beyond the scope. Choice D talks about “exhaustion of overworked pilots”, and does not mention anything about “not fully tested collision avoidance systems”. Choice E fails to use the qualifier ‘not fully tested’, and also brings in a wrinkle called “cargo planes”. Your ability to pay attention to the qualifier used in the definition of the subject of the argument is a critical ability assuring success in this segment of the test. Let us consider another argument and see how we can identify the subject, the scope, and the qualifiers.

Page 106: CRITICAL REASONING 101 T - Amazon S3 · Critical Reasoning Section on the GMAT and GRE will require that you draw conclusions that are logically certain for a set of stated or implied

© Maple Leaf International Consulting, Inc., New York New York. For exclusive use by our registered participants only. Not to be copied or reproduced without our consent.

Page 106

Example 2: The proposal to hire ten new police officers in Middletown is quite foolish. There is sufficient funding to pay the salaries of the new officers, but not the salaries of additional court and prison employees to process the increased caseload of arrests and convictions that new officers usually generate. Which of the following, if true, will most seriously weaken the conclusion drawn above? (A) Studies have shown that an increase in a city’s police force does not necessarily reduce crime. (B) When one major city increased its police force by 19 percent last year, there were 40 percent more arrests and 13 percent more convictions. (C) If funding for the new police officers’ salaries is approved, support for other city services will have to be reduced during the next fiscal year. (D) In most United States cities, not all arrests result in convictions, and not all convictions result in prison terms. (E) Middletown’s ratio of police officers to citizens has reached a level at which an increase in the number of officers will have a deterrent effect on crime. The subject of the argument is “hiring of police officers in Middletown”. Notice that there are no fancy qualifiers used except that we are dealing with a specific town called Middletown. If we come across any choice that talks about ‘hiring of anybody in uptown or in downtown’, we must consider those choices beyond the scope. We are required to weaken the conclusion that hiring of cops is foolish because Middletown cannot deal with the increased caseload of arrests and convictions. Our “weakening strategy” must state the exact opposite of the conclusion and assert that “hiring of cops in Middletown is not such a bad idea because more cops does not translate to more arrests and more convictions”. We notice that the only answer choice that talks about Middle town is choice E. Choice D talks about “most U.S. cities”. For all we know or care, Middletown may be a small town in the United Kingdom, not in the United States. Choice C talks about “other city services”, not relevant to the scope of the argument. Also, choice C does not specifically mention Middletown. Choice B talks about a “major city”, not relevant to the scope of the argument. Choice A does not specifically mention Middletown. Also, choice A has the potential to strengthen the argument, not weaken it. Pay attention to the qualified subject and you will be a winner all the way.

Page 107: CRITICAL REASONING 101 T - Amazon S3 · Critical Reasoning Section on the GMAT and GRE will require that you draw conclusions that are logically certain for a set of stated or implied

© Maple Leaf International Consulting, Inc., New York New York. For exclusive use by our registered participants only. Not to be copied or reproduced without our consent.

Page 107

Example 3: ♦ Mouth cancer is a danger for people who rarely brush their teeth. In order to achieve early detection

of mouth cancer in these individuals, a town’s public health officials sent a pamphlet t all town residents describing how to perform weekly self-examinations of the mouth for lumps.

Which of the following, if true, is the best criticism of the pamphlets as a method of achieving the public health official’s goals?

◊ Many dental diseases produce symptoms that cannot be detected in a weekly self-examination. ◊ Once mouth cancer has been detected, the effectiveness of treatment can vary from person to person. ◊ The pamphlet was sent to all town residents, including those who brush their teeth regularly. ◊ Mouth cancer is much more common in adults than in children. ◊ People who rarely brush their teeth are unlikely to perform weekly examination of their mouth. The qualified subject of the argument is “people who rarely brush their teeth”. You must bear in mind that the answer choice you will select must make a reference to this qualified subject. The scope of the argument is such people (who rarely brush their teeth) are prone to ‘mouth cancer’, not to any other dental disease such as cavities. The question is: Why is the pamphlet idea promoting self examination of mouth among the people who rarely brush their teeth not going to achieve the objective? Choice E brings out the problem with the pamphlet idea. Notice that this choice also makes a reference to the “qualified subject”. Choice A talks about “dental diseases”, which are not relevant to the scope of the argument. The argument is about “mouth cancer” among people who rarely brush their teeth. Also, choice A does not make a reference to the qualified subject. Choice B talks about mouth cancer but fails to mention the qualified subject. Instead, it talks about “person to person”, not relevant to the subject of the argument. Choice C talks about people who brush their teeth regularly. Notice that the subject of the argument is not those who brush their teeth regularly. Choice D talks about “adults versus children”, and fails to make a reference to the qualified subject – ‘people who rarely brush their teeth’. Only choice E talks about our ‘qualified subject’ and mentions a problem with the pamphlet idea. Pay attention to the qualified subject and to the scope of each argument. Then decide on the answer attack strategy. Pick a choice that restates or makes a reference to the same qualified subject and does the job of weakening or strengthening as required. Let us consider another example and hone this understanding.

Page 108: CRITICAL REASONING 101 T - Amazon S3 · Critical Reasoning Section on the GMAT and GRE will require that you draw conclusions that are logically certain for a set of stated or implied

© Maple Leaf International Consulting, Inc., New York New York. For exclusive use by our registered participants only. Not to be copied or reproduced without our consent.

Page 108

Example 4: ♦ Beverage company representative: The plastic rings that hold six-packs of beverage cans together

pose a threat to wild animals, which often become entangled in the discarded rings and suffocate as a result. Following our lead, all beverage companies will soon only those rings consisting of a new plastic that disintegrates after only three days’ exposure to sunlight. Once we all complete the switch over from the existing to the new plastic rings, therefore, the threat of suffocation that plastic rings pose to wild animals will be eliminated.

The argument depends on which of the following assumptions? ♦ None of the new plastic rings can disintegrate immediately after exposure to sunlight. ♦ The switch over to the new plastic rings can be completed without causing significant financial

hardship to the beverage companies. ♦ Wild animals will not become entangled in the new plastic rings before the rings have

had sufficient exposure to sunlight in order to completely disintegrate. ♦ Use of old plastic rings poses no substantial threat to wild animals other than that of suffocation. ♦ Any wild animal that becomes entangled in the plastic rings will suffocate as a result.

We notice that the subject of the argument is “the new plastic rings”, and the scope of the argument is the elimination of risk of suffocation posed to wild animals. The argument states that a critical characteristic of the new plastic rings is that they ‘disintegrate’ after 3 days of exposure to sunlight. The conclusion is that the new plastic rings will totally eliminate the risk of suffocation to the ‘wild animals’. In order to support the conclusion, we need to assume that the critical characteristic of the new plastic rings – namely, disintegration after 3-days of exposure to sunlight – is not a big deal and that the wild animals are not going to be entangled in the new plastic rings before they have had a chance to be exposed to sunlight and disintegrate. We notice that Choice C is the best expression of this assumption. Choice A uses the term “new plastic rings” but the assumption cannot counter the evidence stated in the argument. Also, this choice does not address the scope of the argument – risk of suffocation to wild animals. Choice B uses the term “new plastic rings” but adds a wrinkle called ‘financial hardship’. Also, this choice does not address the scope of the argument – ‘risk of suffocation to wild animals.’ Choices D and E do not talks about “new plastic rings” at all, and for that reason alone, they must be discarded. Also, “risk other than suffocation” is not relevant to the scope of the argument. The argument does not imply that there are no other risks.

Page 109: CRITICAL REASONING 101 T - Amazon S3 · Critical Reasoning Section on the GMAT and GRE will require that you draw conclusions that are logically certain for a set of stated or implied

© Maple Leaf International Consulting, Inc., New York New York. For exclusive use by our registered participants only. Not to be copied or reproduced without our consent.

Page 109

Let us consider another argument and see how we can pick a choice based on our understanding of the subject, the scope, and the question posed. EXAMPLE 5: ♦ Humans began to spread across North America around 12,000 years ago, as the climate became

warmer. During the same period, large mammals that were once abundant in North America, mammals such as mastodon, the wooly mammoth, and the saber-toothed tiger became extinct. Thus, contrary to the myth that humans formerly lived in harmony with the rest of nature, it is clear that even 12,000 years ago human activity was causing the extinction of animal species.

The above argument assumes that ♦ The human interests are somewhat at conflict with those of other creatures of nature. ♦ The large mammals did not become extinct for other reasons than poaching by humans. ♦ The extinction of mastodons, the wooly mammoth, and the saber-toothed tiger had the same

significance then as extinction of protected species does today. ♦ No other species besides the mastodon, the wooly mammoth, and the saber-toothed tiger became

extinct as a result of humans spreading across North America. ♦ The large mammals’ extinction was not a direct result of the same change in climate that allowed

humans to spread across North America. The qualified subject is “large mammals”, and the scope is about factors that may have led to their extinction. There are two major factors stated in the argument – Humans spreading across North America and the warming of the climate. The conclusion is that the ‘humans spreading across North America’ caused the extinction of large mammals. In order to support this conclusion, it is critical to exclude the other factor – warming of the climate – as responsible for the extinction of large mammals. Choice E does it best. Notice that the choice E talks about the same qualified subject – large mammals – and states what is critical to support the conclusion. Choice A talks about “other creatures of nature”, not about “large mammals”. Choice B, although tempting, is not good because it brings in a wrinkle called “poaching by humans”. Also, choice B fails to address the third factor – warming of the climate – as not responsible for the death of the large mammals. Choice C talks about “protected species”, which are totally beyond the scope of the argument. Choice D talks about “other species than the large mammals”, and they are not relevant to the scope of the argument.

Page 110: CRITICAL REASONING 101 T - Amazon S3 · Critical Reasoning Section on the GMAT and GRE will require that you draw conclusions that are logically certain for a set of stated or implied

© Maple Leaf International Consulting, Inc., New York New York. For exclusive use by our registered participants only. Not to be copied or reproduced without our consent.

Page 110

Consider another argument and hone your understanding of the Subject-Scope- Qualifier strategy. EXAMPLE 6: ♦ The book “To Save the Earth” is so persuasive that no one who reads it can fail to heed its

environmentalist messages and appeal. Members of the Earth Association have given away 2,000 copies of the book in the last month. Therefore, the Earth Association can justly claim credit for at least 2,000 people in one month converted to the environmentalist cause.

Which of the following is an assumption on which the argument depends? ♦ No other environmentalist groups gave away copies of To Save the Earth during the month in

which the Earth Association did. ♦ The people to whom the Earth Association gave copies of To Save the Earth would not have been

willing to pay to receive it from the Earth Association. ♦ The copies of To Save the Earth given away by members of the Earth Association were printed on

recycled paper. ♦ None of those who received To Save the Earth from a member of Earth Association

was already committed to the environmentalist cause when they received the book. ♦ Every recipient of the To Save the Earth will embrace the environmental program advocated by

the Earth Association. The subject is “the book ‘To Save the Earth’” and the scope is limited to “distribution of free copies of the book to 2,000 people by the members of the Earth Association”. The conclusion talks about “converting 2,000 people” by sending free copies of the book. What is critical to conversion is a ‘lack of belief in the environmental cause’ among those who received the books to begin with. Our assumption must state this critical piece of information. Choice D does it best. Choice E is tempting but we must bear in mind that the argument is NOT about the “program advocated by the Earth Association”. The argument simply states that “members of the Earth association” distributed some 2,000 free copies. Choice A talks about “other environmental groups”, and they are not relevant to the scope of the argument. The argument simply talks about the members of the Earth Association. Choice B talks about “payment or willingness to pay for the book”, not relevant to the scope of the argument. Choice C talks about “printing of the book on recycled paper”, not relevant to the scope of the argument. Bear in mind that the assumption is a statement of a critical piece of information supporting the conclusion.

Page 111: CRITICAL REASONING 101 T - Amazon S3 · Critical Reasoning Section on the GMAT and GRE will require that you draw conclusions that are logically certain for a set of stated or implied

© Maple Leaf International Consulting, Inc., New York New York. For exclusive use by our registered participants only. Not to be copied or reproduced without our consent.

Page 111

EXAMPLE 7: An ingredient in marijuana known as THC has been found to inactivate herpes viruses in experiments. In previous experiments researchers found that inactivated herpes viruses can convert healthy cells into cancer cells. It can be concluded that the use of marijuana can cause cancer. Which one of the following, if true, most seriously weakens the argument? (A) Several teams of scientists performed the various experiments and all of the teams had similar results. (B) The carcinogenic effect of THC could be neutralized by the other ingredients found in marijuana. (C) When THC kills herpes viruses it weakens the immune system, and it might thus diminish the body’s ability to fight other viruses, including viruses linked to cancers. (D) If chemists modify the structure of THC, THC can be safely incorporated into medications to prevent herpes. (E) To lessen the undesirable side effects of chemotherapy, the use of marijuana has been recommended for cancer patients who are free of the herpes virus. The subject is “an ingredient in marijuana known as THC”, and the scope is limited to its action in deactivating the herpes virus, which, in turn, turns healthy cells into cancer cells. The conclusion is that the use of marijuana causes cancer. We are required to weaken this conclusion and the strategy must involve stating the opposite of the conclusion but at the same time providing an explanation for the evidence used. Choice B does the weakening best by stating that the other ingredients of marijuana will counter the effects of THC, and therefore, the use of marijuana may not cause cancer. Choice A does not even talk about the THC in marijuana. We should stay away from this choice. Choice C has the potential to strengthen the argument, not weaken it. Choice D talks about “modifying the structure of THC”, not relevant to the scope of the argument. Choice E does not talk about THC at all. Also, “people who are free of herpes virus” is not relevant to the scope of the argument. The argument is about the action of THC in deactivating the herpes virus, and “absence of herpes virus” is not relevant to the details of the argument. Now that we are slowing but surely getting there, let us do one more.

Page 112: CRITICAL REASONING 101 T - Amazon S3 · Critical Reasoning Section on the GMAT and GRE will require that you draw conclusions that are logically certain for a set of stated or implied

© Maple Leaf International Consulting, Inc., New York New York. For exclusive use by our registered participants only. Not to be copied or reproduced without our consent.

Page 112

EXAMPLE 8: Besides laying eggs in her own nest, any female wood duck will lay an egg in the nest of another female wood duck if she sees the other duck leaving her nest. Under natural nesting conditions, this parasitic behavior is relatively rare because the ducks’ nests are well hidden. However, when people put up nesting boxes to help the ducks breed, they actually undercut the ducks’ reproductive efforts. These nesting boxes become so crowded with extra eggs that few, if any, of the eggs in those boxes hatch. The statements above, if true, most strongly support which one of the following? (This is a conclusion question. “Which of the following supports the argument?” is a strengthening question. “Which of the following is supported by the argument?” or “the argument supports which of the following?” is a conclusion question.) (A) Female wood ducks will establish nests in nest boxes only when natural nesting sites are not available. (B) Nesting female wood ducks that often see other female wood ducks are the most successful in their breeding efforts. (C) The nesting boxes for female wood ducks have less space for eggs than do natural nesting sites. (D) The nesting boxes would be more effective in helping wood ducks breed if they were less visible to other wood ducks than they currently are. (E) Nesting boxes are needed to supplement the natural nesting sites of wood ducks because of the destruction of much of the ducks’ habitat. The subject of the argument is “female wood ducks” and the scope is limited to their egg-laying behavior. The argument says that the female wood ducks free load on other nests, but in nature this is not a problem because of a lack of visibility of other females’ nests. But the wooden nesting boxes set out by humans are more visible and the evidence states that the these boxes become crowded, to the detriment of the eggs laid. “crowding” implies that the female wood ducks lay more eggs typically in these ‘human boxes’ than the size would allow. Choice C states this logical inference best. Choice A is pure speculation and a wild conclusion. There is nothing in the argument to suggest that the female wood ducks prefer natural nesting sites. Choice B talks about “seeing often other female wood ducks”. Also, there is nothing in the argument to support a conclusion that only those female ducks are successful in their breeding efforts. Choice D talks about “visibility to other wood ducks”. There is nothing in the argument to suggest that the boxes become crowded because too many female wood ducks lay eggs in them. Choice E talks about “destruction of the natural habitat”. Clearly, this is an irrelevant piece of information.

Page 113: CRITICAL REASONING 101 T - Amazon S3 · Critical Reasoning Section on the GMAT and GRE will require that you draw conclusions that are logically certain for a set of stated or implied

© Maple Leaf International Consulting, Inc., New York New York. For exclusive use by our registered participants only. Not to be copied or reproduced without our consent.

Page 113

Try to understand the Subject, the Scope, the qualifiers, and the question when you deal with critical reasoning arguments. You cannot go wrong. Let us wrap up the “subject-scope” strategy discussion by looking at two more arguments. EXAMPLE 9: The symptoms of mental disorders are behavioral, cognitive, or emotional problems. Some patients with mental disorders can be effectively treated with psychotherapy. But it is now known that in some patients, mental disorders result from chemical imbalances affecting the brain. Thus these patients can be effectively treated only with medication that will reduce or correct the imbalance. The argument depends on assuming which one of the following? (A) Treatment by psychotherapy can produce no effective reduction in or correction of chemical imbalances that cause mental disorders. (B) Treatment with medication always shows faster results for patients with mental disorders than does treatment with psychotherapy. (C) Most mental disorders are not the result of chemical imbalances affecting the brain. (D) Medication is always more effective in treating patients with mental disorders than is psychotherapy. (E) Treatment with psychotherapy has no effect on mental disorders other than a reduction of the symptoms. The subject is “mental disorders” and the scope is the treatment options. While some patients with mental disorders can be treated using psychotherapy, those with mental disorders caused by chemical imbalances can be treated effectively with medication only. That is what the conclusion says. Notice that the conclusion is about the qualified subject – mental disorders caused by chemical imbalances. Our answer choice must make a reference to this qualified subject. If it does not, there is a great chance that we are looking at the wrong option. Our assumption must state the critical information supporting the conclusion. It is critical to eliminate psychotherapy as a viable treatment option for mental disorders caused by chemical imbalances. Choice A states this assumption best. Notice that choice A talks about mental disorders caused by chemical imbalances.

Page 114: CRITICAL REASONING 101 T - Amazon S3 · Critical Reasoning Section on the GMAT and GRE will require that you draw conclusions that are logically certain for a set of stated or implied

© Maple Leaf International Consulting, Inc., New York New York. For exclusive use by our registered participants only. Not to be copied or reproduced without our consent.

Page 114

Choice B does not talk about “mental disorders caused by chemical imbalances”. The conclusion talks about treatment with medication for mental disorders caused by chemical imbalances only. Choice C is at best an inference, not an assumption. An inference is an implied piece of information whereas an assumption is a critical piece of information propping up the conclusion. Choices D and E do not mention the qualified subject – mental disorders caused by chemical imbalances – and, therefore, are not worthwhile options to consider. Your ability to understand the ‘qualified subject’ is a critical ability guaranteeing success in GMAT. EXAMPLE 10: ♦ The gray squirrel, introduced into the local woodlands ten years ago, threatens the indigenous

population of an endangered owl species because the squirrels’ habitual stripping of the tree bark destroys trees in which the owls nest. Some local officials have advocated setting out poison for the gray squirrels arguing that this measure, while eliminating squirrels, would pose no threat to the owl population because the poison would be placed in containers accessible to squirrels and other rodents only.

Which of the following, if true, most calls into question the official’s argument? ♦ One of the species whose members are likely to eat the poison is the red squirrel, a species on which

the owls do not prey. ♦ The owls whose nesting sites are currently threatened by the gray squirrels feed primarily on rodents. ♦ No indigenous population of any other bird species apart from the endangered owls is threatened by

the gray squirrels. ♦ The owls that are threatened build their nests in the tops of trees, but the gray squirrels strip away the

bark from the trunks. ♦ The officials’ plan entails adding the poison to food sources that are usually eaten by rodents but not

by other animals. The qualified subject is the “gray squirrel”, and the scope is limited to endangering of the indigenous owl species through the stripping of the tree bark and through the destruction of trees on which the owl nests. The conclusion is that if gray squirrel is poisoned, the owl species are not threatened. We are required to weaken this argument’s conclusion by stating that poisoning the gray squirrel will pose a threat to the indigenous species of owl in some fashion. Choice B provides one such weakening of the conclusion. If the owls feed on rodents poisoned, then the owls will be

Page 115: CRITICAL REASONING 101 T - Amazon S3 · Critical Reasoning Section on the GMAT and GRE will require that you draw conclusions that are logically certain for a set of stated or implied

© Maple Leaf International Consulting, Inc., New York New York. For exclusive use by our registered participants only. Not to be copied or reproduced without our consent.

Page 115

in danger too. Notice that the choice B makes a reference to the qualified subject and does an effective job of negating the conclusion. Choice A talks about “red squirrels” whereas the qualified subject of the argument is “gray squirrel”. Choice C talks about “other bird species than the owls”. The argument does not make a reference to other bird species. Choices D and E restate the evidence and do nothing to weaken or strengthen the argument. Let us recap the procedure for dealing with the critical reasoning questions. 1. Pay attention to the Subject of the argument, and to the qualifiers used

to describe the subject. A qualifier is a straight adjective or a predicate adjective. “Overweight people” is an example of a straight adjective but “people who are overweight” is an example of a predicate adjective. We will learn in Sentence Correction that the straight adjectival form is more concise than is the predicate adjective but the critical reasoning section does not care about “conciseness”. Be prepared to recognize qualifiers no matter in what form they appear. Also, be sure to recognize the same qualified subject in the answer choice. If the answer choice uses a different qualifier or fails to mention any qualifier used in the argument, then there is a good chance you are looking at the wrong answer choice.

2. Pay attention to the SCOPE of the argument. Scope is the set of activities described and all the factors relevant to those activities.

3. Strictly stay within the scope of the argument. Do not pick answer choices that go beyond the scope of the argument.

4. Understand the question posed, and be sure to paraphrase the question. If the question asks you to “weaken the argument”, then you will paraphrase the question to: “I am looking for a statement that will state that the conclusion is not logically derived by providing an explanation for the evidence used”. If the argument does not reach a conclusion on the basis of evidentiary information but on the basis of some sweeping statements, look for a statement stating the exact opposite of the conclusion. Also, bear in mind that if the question is about “weakening”, there will be an answer choice doing the exact opposite. When you deal with assumption questions, expect a choice to provide the inference. Bear in mind that an assumption is not the same as an inference.

5. Stay focused on the Scope, on the qualified subject, and on the question. It is easy to lose sight of any of the above or all of the above as you are engaged in the process of vetting the answer choices.

Page 116: CRITICAL REASONING 101 T - Amazon S3 · Critical Reasoning Section on the GMAT and GRE will require that you draw conclusions that are logically certain for a set of stated or implied

© Maple Leaf International Consulting, Inc., New York New York. For exclusive use by our registered participants only. Not to be copied or reproduced without our consent.

Page 116

YES NO YES NO YES NO YES NO YES NO YES NO YES

Understand the type of argument and its essence. What is the argument about?

Determine the assumption(s)

Weaken the argument?

Understand the question as ‘Why do you figure that the SPECIFICS of the conclusion are NOT

valid, given the evidence?”

Strengthen the argument?

Provide additional anecdotal evidence or information supporting the conclusion, and confirming the assumption..

What is the assumption?

An ASSUMPTION will answer the question: HOW DO WE KNOW that the VERBATIM conclusion is valid? Assumption provides a reason why the verbatim conclusion is valid. “Verbatim conclusion is valid BECAUSE 9assumption).”.

LOGICAL CONCLUSION or INFERENCE?

Recycle the argument’s evidence and/or state an implied information. Do not speculate or engage in wild predictions not supported by the information in the evidence.

LOGIC STRUCTURE?

Predetermine the completion in broad terms for sentence

completion; Understand the argument in terms of x, y, and z for parallel structure questions.

Rational Explanation?? (inference or hypothesis)

Another way of asking you to make a logical conclusion. Pick the best explanation for the evidence used.

COMPREHENSION QUESTION?

Understand the essence of the argument and pick a choice that matches this understanding.

Understand the subject of the argument, and the scope of

the argument. Pay attention to the QUALIFIERS that define and describe the subject and the scope. Stay focused on these QUALIFIERS. Control

your temptation to pick an answer choice that describes a different subject or a different

activity.

Page 117: CRITICAL REASONING 101 T - Amazon S3 · Critical Reasoning Section on the GMAT and GRE will require that you draw conclusions that are logically certain for a set of stated or implied

© Maple Leaf International Consulting, Inc., New York New York. For exclusive use by our registered participants only. Not to be copied or reproduced without our consent.

Page 117

CRITICAL REASONING SUMMARY

QUESTION TYPE WHAT IS REQUIRED TO BE DONE?

WEAKENING OF THE ARGUMENT

♦ Provide a different explanation for the evidence used. Any alternative explanation for the evidence used will weaken the conclusion of the argument.

♦ Provide a different reason for the evidence than the conclusion would have us believe.

♦ State the opposite of the conclusion and provide a reason for it. (If the conclusion is that “Tom cannot attend Harvard because his GMAT score is 450”, look for statement affirming that Tom can indeed go to Harvard because Harvard does not consider GMAT score seriously, or something to that effect).

STRENGTHENING OF THE ARGUMENT

♦ Provide additional information/evidence that lends support to the conclusion for the stated evidence.

♦ Provide Anecdotal information explaining that the assumption is valid. A strengthening question explains ‘how do you know that the assumption is valid?’

ASSUMPTION ♦ Identify a critical implied information required to make the conclusion logical and valid. Think of Assumption as a compelling reason supporting the Conclusion in keeping with the evidentiary information.

♦ Take the VERBATIM conclusion and do the following test: “Verbatim conclusion BECAUSE (assumption)’.

CONCLUSION / PREDICTION / INFERENCE / EXPLANATION

♦ Recycle the argument’s evidence and state that which is implied but not openly expressed. An inference is an implied information contained in the evidence used. Bear in mind that an inference is NOT the same as an assumption. An assumption is an IMPLIED critical information required to support the conclusion. An inference is an IMPLIED conclusion.

♦ Do not pick a choice that forces you to use your expectation or experience, and contains information not supported by the information in the evidence presented.

MISCELLANEOUS QUESTIONS ♦ Comprehension Questions ♦ LOGIC STRUCTURE

♦ Recycle the scope of the argument. The question tests

your ability to understand what is described. Treat this format as a miniature reading comprehension, providing a clearly identified reference for the question. Get the hang of the essence of the argument, and pick a choice that matches the understanding.

♦ Understand the argument in terms of X and Y, and pick a

choice that uses the same structure as the given argument does.

Read the question as: WHY DO YOU FIGURE THAT THE SPECIFICS OF THE CONCLUSION ARE VALID, GIVEN THE EVIDENCE?

HOW DO YOU FIGURE THAT THE SPECIFICS OF THE CONCLUSION ARE VALID, GIVEN THE EVIDENCE?

WHY DO YOU FIGURE THAT THE SPECIFICS OF THE CONCLUSION ARE VALID, GIVEN THE EVIDENCE?

PARAPHRASE EVIDENCE INTERPRET ADJECTIVES, VERBS,

NOUNS USED IN THE EVIDENCE. INTERPRET INFORMATION

STATED IN THE OPPOSITE CONTEXT.

Page 118: CRITICAL REASONING 101 T - Amazon S3 · Critical Reasoning Section on the GMAT and GRE will require that you draw conclusions that are logically certain for a set of stated or implied

© Maple Leaf International Consulting, Inc., New York New York. For exclusive use by our registered participants only. Not to be copied or reproduced without our consent.

Page 118

PROCEDURE FOR ANSWERING CRITICAL REASONING QUESTIONS 1. Pay attention to the QUALIFIERS that are used to define the SUBJECT

and the SCOPE. Make a note of them on your scratch paper. Be sure to recognize the same qualifiers in the answer choice you are tempted to pick. Also, stay away from answer choices using qualifiers not used in the argument. Remember: Qualifiers can be subject-critical and scope-critical, and your answer choice must be consistent with these aspects of the argument. Also, if the argument does not use any qualifiers, be sure not to pick answer choices using qualifiers. For example, if the argument is about “people”, be suspicious of answer choices referencing “obese people” or “rich people”.

2. Pay attention to the scope of the argument. Stay within the scope. Do not pick a choice that goes beyond the scope. IF the scope is limited to “adoption of technologies”, do not pick answer choices talking about “development of new technologies”.

3. Understand the question posed. Sometimes, the conclusion of the argument may be stated as part of the question itself. Formulate the strategy for each question type. For example, a weakening question asks you to identify a different explanation or significance for the evidence used in the argument. A strengthening question asks you to identify additional evidence supporting the conclusion. Assumption question asks you to identify a compelling reason for the conclusion in keeping with the evidentiary information. And so on.

4. Rephrase the question. Do not just say that you are looking for a ‘weakening information’. Rephrase the question, for example, as: “I am looking for a different explanation for why women are not running for elective offices even though they have the same chance as winning elections as men do.”

5. Stay focused on the question. It is possible that you lose focus of the question in the process of critically examining answer choices. Expect the “contradictory” options: For weakening questions, expect a choice to do the exact opposite – strengthen the conclusion, or vice versa. For Assumption questions, expect a choice to be an inference. You should know that an inference is not the same as an assumption. An inference is an implied information contained in the evidence; an assumption is an implied information supporting the conclusion.

PAY ATTENTION TO THE SUBJECT, THE SCOPE, AND THE QUALIFIERS. The test is also about your ability to make a distinction between “many procedures” and “all procedures”, and between “legal obligations” and “ethical obligations”. In the final analysis, the whole argument turns on the qualifiers used (or, lack of qualifiers, in some cases). REMEMBER: THE TEST DOES NOT LIKE VAGUE, UNSURE, TOO SURE, EXTREME, POLITICALLY INSENSITIVE, SPECULATIVE OR UNVERIFIABLE STATEMENTS.